SlideShare a Scribd company logo
1 of 35
Download to read offline
Tuyển tập đề chọn đội tuyển VMO
Nguyễn Minh Tuấn, Huỳnh Kim Linh, Nguyễn Nhất Huy, Đoàn Duy Tùng
Ngày 12 tháng 10 năm 2020
Mục lục
1 Đề thi tỉnh Phú Thọ. 5
A Phú Thọ ngày thi thứ nhất. 5
B Phú Thọ ngày thi thứ hai. 5
2 Đề thi tỉnh Nam Định. 6
A Nam Định ngày thi thứ nhất. 6
B Nam Định ngày thi thứ hai. 6
3 Đề thi tỉnh Hưng Yên. 7
A Hưng Yên ngày thi thứ nhất. 7
B Hưng Yên ngày thi thứ 2. 8
4 Đề thi tỉnh Hà Tĩnh. 8
A Hà Tĩnh ngày thi thứ nhất. 8
B Hà Tĩnh ngày thi thứ hai. 9
5 Đề thi tỉnh Quảng Ninh. 9
A Ngày thi thứ nhất. 9
B Ngày thi thứ hai. 10
6 Đề thi tỉnh Lạng Sơn. 10
A Ngày thi thứ nhất. 10
1
Š Tuyển tập đề chọn đội tuyển các tỉnh trên cả nước VMO 2021
7 Đề thi học sinh giỏi Chuyên Lào Cai 11
8 Trải nghiệm VMO 2021 - Bring Math to Everyone - BM2E 12
A Vòng 1, ngày thi thứ nhất. 12
B Vòng 1, ngày thi thứ hai. 12
9 Đề thi tỉnh Sóc Trăng. 13
A Sóc Trăng ngày thi thứ nhất. 13
B Sóc Trăng ngày thi thứ hai. 13
10 Đề thi tỉnh Nghệ An. 14
A Ngày thi thứ nhất. 14
B Ngày thi thứ hai 14
11 Đề thi Chuyên Sư Phạm Hà Nội 2020 14
A Ngày thi thứ nhất. 14
B Ngày thi thứ hai. 15
12 Đề thi tỉnh Vũng Tàu 15
A Ngày thi thứ nhất. 15
13 Đề thi phổ thông năng khiếu 16
A Ngày thi thứ nhất. 16
B Ngày thi thứ hai. 17
14 Đề thi tỉnh Ninh Bình. 18
15 Đề thi thành phố Cần Thơ 18
16 Đề thi tỉnh Bắc Ninh. 19
A Ngày thi thứ nhất. 19
B Ngày thi thứ hai. 20
® Chinh phục olympic toán 2 h Tạp chí và tư liệu toán học
Š Tuyển tập đề chọn đội tuyển các tỉnh trên cả nước VMO 2021
17 Đề thi tỉnh Lâm Đồng. 20
A Ngày thi thứ nhất. 20
18 Đề thi tỉnh Bắc Giang 21
19 Đề thi tỉnh Đồng Tháp. 21
20 Đề thi thành phố Hà Nội. 22
21 Đề thi tỉnh Hà Nam. 23
22 Đề thi tỉnh Kiên Giang. 23
A Ngày thi thứ nhất. 23
B Ngày thi thứ hai. 24
23 Đề thi tỉnh Quảng Ngãi. 25
24 Đề thi tỉnh Tây Ninh. 25
A Ngày thi thứ nhất. 25
B Ngày thi thứ hai. 26
25 Đề thi tỉnh Quảng Nam. 26
26 Đề thi tỉnh Quảng Trị. 27
27 Đề thi chọn đội tuyển THPT Chuyên Nguyễn Du. 27
A Ngày thi thứ nhất. 27
B Ngày thi thứ hai. 28
28 Đề thi tỉnh Phú Yên. 28
29 Đề chọn đội tuyển chuyên Đại học Vinh. 29
A Ngày thi thứ nhất. 29
B Ngày thi thứ hai. 30
30 Đề thi tỉnh Ninh Thuận. 30
® Chinh phục olympic toán 3 h Tạp chí và tư liệu toán học
Š Tuyển tập đề chọn đội tuyển các tỉnh trên cả nước VMO 2021
31 Đề thi tỉnh Đồng Nai. 31
32 Đề thi tỉnh Quảng Ngãi. 31
A Ngày thi thứ nhất. 31
B Ngày thi thứ hai. 32
33 Đề thi tỉnh Vĩnh Long. 33
A Ngày thi thứ nhất. 33
B Ngày thi thứ hai. 33
34 Đề thi tỉnh Quảng Bình. 34
35 Đề thi tỉnh Bình Dương 34
36 Đề thi tỉnh Yên Bái. 35
® Chinh phục olympic toán 4 h Tạp chí và tư liệu toán học
Š Tuyển tập đề chọn đội tuyển các tỉnh trên cả nước VMO 2021
Tóm tắt nội dung
Các đề vẫn đang tiếp tục được cập nhật trong link của bài viết này!
1 Đề thi tỉnh Phú Thọ.
A Phú Thọ ngày thi thứ nhất.
L Câu 1. Cho a, b ∈ R, a 6= b. Giải hệ phương trình





3x + z = 2y + (a + b)
3x2
+ 3xz = y2
+ 2 (a + b) y + ab
x3
+ 3x2
z = y2
(a + b) + 2yab.
L Câu 2. Cho dãy số thực dương (an)n>1 thỏa mãn điều kiện
a1 + a2 + ... + an + an+1 + an+2 < 4an+1, ∀n ∈ N∗
.
Chứng minh rằng a1 + a2 + ... + an 6 an+1, ∀n ∈ N∗
.
L Câu 3. Giả sử O, I lần lượt là tâm đường tròn ngoại tiếp, nội tiếp tam giác ABC với bán kính
R, r tương ứng. Gọi P là điểm chính giữa cung
_
BAC, QP là đường kính của (O), D là giao điểm
của PI và BC, F là giao điểm của đường tròn ngoại tiếp tam giác AID với đường thẳng PA. Lấy
E trên tia DP sao cho DE = DQ.
1 Chứng minh rằng [
IDF = 90◦
.
2 Giả sử [
AEF = [
APE, chứng minh rằng sin2 [
BAC =
2r
R
.
L Câu 4. Trên mặt phẳng tọa độ Oxy, cho S là tập hợp các điểm (x; y) thỏa mãn đồng thời hai
điều kiện
x, y ∈ N; 0 6 y 6 x 6 2020.
1 Tính số phần tử của S.
2 Hỏi có bao nhiêu tập con A gồm 2020 phần tử của S sao cho A không chứa hai điểm (x1; y1) ;
(x2; y2) thỏa mãn (x1 − x2) (y1 − y2) = 0?
B Phú Thọ ngày thi thứ hai.
L Câu 5. Cho đường tròn tâm O với hai điểm B, C cố định nằm trên đường tròn đó. Điểm A thay
đổi trên đường tròn (O) sao cho tam giác ABC nhọn. Gọi D là điểm đối xứng với A qua O, tiếp
tuyến với đường tròn (O) tại D cắt đường thẳng BC tại điểm K. Đường thẳng KO cắt AB, AC lần
lượt tại E và F.
1 Chứng minh DF song song với AB.
2 Gọi I là tâm đường tròn ngoại tiếp tam giác AEF. Chứng minh rằng AI luôn đi qua một điểm
cố định khi A thay đổi.
L Câu 6. Xét a, b, c ∈ Z, 1 6 a < b < c thỏa mãn hệ phương trình đồng dư





(b + 1) (c + 1) ≡ 1(moda)
(c + 1) (a + 1) ≡ 1(modb)
(a + 1) (b + 1) ≡ 1(modc).
® Chinh phục olympic toán 5 h Tạp chí và tư liệu toán học
Š Tuyển tập đề chọn đội tuyển các tỉnh trên cả nước VMO 2021
1 Chứng minh rằng hệ phương trình có vô hạn nghiệm (a; b; c) .
2 Giả sử gcd (a, b) = 1. Tìm tất cả các nghiệm của hệ phương trình đó.
L Câu 7. Cho hàm số f : R → [0; +∞) thỏa mãn
f2
(x + y) + f2
(x − y) = 2f2
(x) + 2f2
(y), ∀x, y ∈ R.
Chứng minh rằng f (x + y) 6 f(x) + f(y), ∀x, y ∈ R, (ở đây f2
(x) = (f(x))2
).
2 Đề thi tỉnh Nam Định.
A Nam Định ngày thi thứ nhất.
L Câu 1. Giải hệ phương trình
(
x3
+ y2
+ 3x = x2
+ x + 3

y
p
3y + 4 + 6x + 21 =
p
x − 2y + 5 + 3xy.
.
L Câu 2. Cho dãy số (xn) xác định bởi x1 =
1
2
và xn+1 = x2
n − xn + 1, ∀n ∈ N∗
.
1 Chứng minh rằng xn 6
2n − 1
2n
, ∀n ∈ N∗
.
2 Tìm giới hạn lim (xn+1 + x1x2
2x3
3 . . . xn
n).
L Câu 3. Tìm tất cả các hàm số f : R → R thỏa mãn f(x+y)·f(x−y)+y2
= f (x2
) , ∀x, y ∈ R.
L Câu 4. Trên đường tròn (O) lấy hai điểm A, M sao cho AM không là đường kính. Điểm I nằm
trên đoạn OA (I không trùng với các điểm O, A). Đường tròn (I, IA) và đường tròn đường kính IM
cắt nhau tại hai điểm B, C. Các đường thẳng MB, MI, MC cắt đường tròn (O) tại các điểm thứ
hai lần lượt là D, E, F sao cho tia MA nằm giữa hai tia MD, ME. Đường thẳng DF cắt các đường
thẳng ME, MA, AE lần lượt tại T, S, Q.
1 Chứng minh rằng ME là phân giác của 
DMF và SD.SF = ST.SQ.
2 Chứng minh rằng
AD
AF
=
BD
CF
và ba điểm B, C, Q thẳng hàng.
B Nam Định ngày thi thứ hai.
L Câu 5.
1 Có bao nhiêu cách chọn ra hai số phân biệt từ 100 số nguyên dương đầu tiên sao cho hai số
này hơn kém nhau không quá 10 đơn vị?
2 Cho tập hợp X =
n
22k
|k ∈ N; 0 6 k 6 2020
o
. Xét tất cả các tập hợp con khác rỗng của X.
Với mỗi tập hợp con như vậy, ta tính tích của mọi phần tử trong tập hợp đó (nếu tập hợp con
chỉ có một phần tử thì quy ước tích chính là phần tử đó). Tính tổng của tất cả các tích thu
được.
L Câu 6. Xét x, y, z ∈ (0; 1]. Chứng minh rằng
36
2
√
x +
√
y
 √
y + 2
√
z
 + 27 (x − 1) (y − 1) (z − 1)  4
Tìm tất cả các bộ giá trị (x; y; z) để dấu bằng trong bất đẳng thức trên xảy ra.
® Chinh phục olympic toán 6 h Tạp chí và tư liệu toán học
Š Tuyển tập đề chọn đội tuyển các tỉnh trên cả nước VMO 2021
L Câu 7. Với P(x) = x2
− 2020x + 1, ta định nghĩa
(
P1(x) = P(x)
Pn+1(x) = P (Pn(x)) , ∀n ∈ N∗
.
1 Chứng minh rằng với mỗi số thực k thuộc (0; 1), đa thức P2(x) − α có đúng bốn nghiệm thực
phân biệt.
2 Tìm tất cả các số thực dương k sao cho Với mọi số nguyên dương n, đa thức Pn(x) − kx luôn
có hai nghiệm thực mà hiệu của chúng lớn hơn
√
2019 × 2023.
L Câu 8. Cho tam giác ABC nội tiếp đường tròn (O). P là điểm bất kì nằm trong tam giác ABC
nhưng không thuộc đường thẳng AO. Đường thẳng AP cắt đường tròn (O) tại điểm D (khác A).
Kẻ các đường kính DE, AF của đường tròn (O). Các đường thẳng EP, FP lần lượt cắt đường tròn
(O) tại các điểm G, H (khác E, F). Gọi K là giao điểm của các đường thẳng AH và DG, L là hình
chiếu vuông góc của K trên đường thẳng OP. Giả sử rằng hai đường thẳng KL và AD cắt nhau.
1 Chứng minh rằng bốn điểm A, L, K, D cùng thuộc một đường tròn (gọi đường tròn này là (S)).
2 Chứng minh rằng các đường thẳng OP và EF cắt nhau tại điểm T thuộc đường tròn (S).
L Câu 9.
1 Cho p là số nguyên tố và n là số nguyên dương lớn hơn 1. Chứng minh rằng mọi ước nguyên
tố của số A = 1 + n + n2
+ ... + np−1
hoặc là p, hoặc chia cho p dư 1.
2 Tìm tất cả các số nguyên tố p sao cho tồn tại số nguyên dương n mà số B = 2+n+n2
+...+np−1
là lũy thừa bậc 5 của một số nguyên dương.
3 Đề thi tỉnh Hưng Yên.
A Hưng Yên ngày thi thứ nhất.
L Câu 1. Cho dãy số (an) xác định bởi a0 = 2020 và an+1 =
a2
n
1 + an
, ∀n  0.
1 Chứng minh rằng dãy số (an) có giới hạn và tìm giới hạn đó.
2 Tính [a1000] ([x] phần nguyên của số thực x).
L Câu 2. Cho P(x) là một đa thức bậc ba, xét đa thức
Q(x) = x3
+ 2x + 1 − P(x)

2x3
− 6x2
+ 5 − P(x)

1 Giả sử Q(x) 6 0, ∀x ∈ R và P(0) = 4, tính Q (−1).
2 Hỏi có tồn tại hay không đa thức P(x) để Q(x)  0, ∀x ∈ R.
L Câu 3. Cho tam giác ABC có đường tròn nội tiếp (I) tiếp xúc với các cạnh BC, CA, AB lần
lượt tại các điểm X, Y, Z. Các đường phân giác trong và ngoài tại góc A của tam giác ABC cắt BC
tại E, F. Các tiếp tuyến kẻ từ E, F đến (I) cắt nhau tại điểm D (D khác E, F). Trên BI lấy điểm
K sao cho DK ⊥ AI.
1 Giả sử BC cố định và A thay đổi, chứng minh K thuộc một đường tròn cố định.
2 Gọi M, N là các tiếp điểm của (I) với các tiếp tuyến DE, DF. Đường thẳng qua D song song
với AB cắt AC, MN tại P, Q. Đường thẳng QZ cắt đường tròn (I) tại T. Chứng minh PT
tiếp xúc với đường tròn (I).
® Chinh phục olympic toán 7 h Tạp chí và tư liệu toán học
Š Tuyển tập đề chọn đội tuyển các tỉnh trên cả nước VMO 2021
L Câu 4. Cho số nguyên tố p  5. Đặt n = 4p
− 1.
1 Chứng minh rằng n có ít nhất 3 ước nguyên tố phân biệt và 2n
≡ 8 (modn).
2 Với a là một số tự nhiên, chứng minh rằng 2 + a + a2
+ ... + ap−1
không là số chính phương.
B Hưng Yên ngày thi thứ 2.
L Câu 1. Tìm tất cả các hàm số f : R → R thỏa mãn điều kiện
f(x + 2020.f(x)) = f(x) + 2020xf(y)
với mọi x, y ∈ R.
L Câu 2. Cho tam giác ABC ngoại tiếp đường tròn (I) và có trực tâm H. Gọi D, E, F lần lượt là
các tiếp điểm của BC, CA, AB với đường tròn (I). Gọi S, T lần lượt là trung điểm của AH, BH và
gọi M là điểm đối xứng với S qua FE, gọi N là điểm đối xứng với T qua FD. Chứng minh rằng
đường thẳng MN đi qua trực tâm của tam giác DEF.
L Câu 3. Cho bảng ô vuông ABCD kích thước 2021 × 2021 gồm 20212
ô vuông đơn vị, mỗi ô
vuông đơn vị được tô bởi một trong ba màu đen, trắng hoặc xám. Một cách tô màu được gọi là đối
xứng nếu mỗi ô vuông đơn vị có tâm trên đường chéo AC được tô màu xám và mỗi cặp ô vuông đơn
vị đối xứng qua AC được tổ cùng màu đen hoặc cùng màu trắng. Người ta điền vào mỗi ô xám số 0,
mỗi ô trắng một số nguyên dương và mỗi ô đen một số nguyên âm. Một cách điền số như vậy được
gọi là k - cân đối (với k là số nguyên dương) nếu thỏa mãn các điều kiện sau
1 Mỗi cặp ô vuông đơn vị đối xứng qua AC được điền cùng một số nguyên thuộc đoạn [−k; k].
2 Nếu một hàng và một cột giao nhau tại ô đen thì tập các số nguyên dương được điền trên hàng
đó và tập số nguyên dương được điền trên cột đó không giao nhau, nếu một hàng và một cột
giao nhau tại ô trắng thì tập các số nguyên âm được điền trên hàng đó và tập các số nguyên
âm được được điền trên cột đó không giao nhau. Tìm giá trị nhỏ nhất của k để với mọi cách
tô màu đối xứng, luôn tồn tại cách điền k - cân đối.
4 Đề thi tỉnh Hà Tĩnh.
A Hà Tĩnh ngày thi thứ nhất.
L Câu 1. Cho phương trình xn
= x + 1. Chứng minh rằng với mỗi n ∈ N, n  2, phương trình có
nghiệm dương duy nhất, ký hiệu là xn.
1 Tính giới hạn của dãy số (un) với un = n (xn − 1).
2 Tìm số thực k sao cho dãy số vn = nk
(xn+1 − xn) có giới hạn hữu hạn khác 0.
L Câu 2. Tìm tất cả các hàm số f : R → R thỏa mãn f(y−f(x)) = f(f(x))−2yf(x)+f(y)∀x, y ∈
R.
L Câu 3. Cho tam giác nhọn ABC có AB  AC  BC và nội tiếp đường tròn (O; R). Đường
thẳng d thay đổi nhưng luôn vuông góc với đoạn thẳng OA và cắt cạnh AB, AC lần lượt tại M, N.
Gọi K là giao điểm của đường thẳng BN và CM; P là giao điểm của đường thẳng AK và BC; I là
trung điểm của BC.
1 Chứng minh tứ giác MNIP nội tiếp được trong một đường tròn.
2 Gọi H là trực tâm tam giác AMN. Chứng minh rằng đường thẳng HK luôn đi qua một điểm
cố định khi đường thẳng d thay đổi.
L Câu 4. Tìm tất cả các số nguyên tố m, n sao cho
7m
+ 7n
mn
là một số nguyên.
® Chinh phục olympic toán 8 h Tạp chí và tư liệu toán học
Š Tuyển tập đề chọn đội tuyển các tỉnh trên cả nước VMO 2021
B Hà Tĩnh ngày thi thứ hai.
L Câu 5. Với a, b ∈ Z, xét hai dãy đa thức
(
P0(x) = x + a, Pn+1(x) = P2
n (x) + (−1)n
n
Q0(x) = x + b, Qn+1(x) = −Q2
n(x) + (−1)n
n
, ∀n  0, n ∈ Z.
1 Cho a 6= b, a − b 6= ±1. Hỏi đa thức f(x) = P0
2020(x).Q0
2020(x) có bao nhiêu nghiệm phân biệt?
2 Tìm điều kiện của a, b để tồn tại nsao cho Pn(x) + Qn(x) chia hết cho x + 2.
L Câu 6. Cho tam giác ABC không cân, đường tròn (I) nội tiếp tam giác tiếp xúc với các cạnh
BC, CA, AB lần lượt tại D, E, F. Đường thẳng AI cắt đường tròn (I) lần lượt tại M, Hvà cắt EF
tại N(M nằm giữa A và I). Tiếp tuyến tại I của đường tròn ngoại tiếp tam giác IMD cắt DN tại
P. Trên đường thẳng PM lấy Q sao cho DQ vuông góc với EF.
1 Chứng minh rằng PH song song với AD.
2 Chứng minh đường thẳng DA đi qua trung điểm của đoạn thẳng MQ.
L Câu 7. Cho bảng vuông n × n ô vuông (n  2) với các ô vuông được tô bằng 2 màu đen hoặc
trắng (mỗi ô chỉ tô bởi một màu). Biết rằng cứ mỗi bước, ta chỉ được thay đổi màu của toàn bộ các
ô trong một hàng hoặc một cột (ô trắng thành đen và ô đen thành trắng).
1 Giả sử trong bảng có đúng 1 ô được tô đen. Hỏi sau một số bước đổi màu các hàng hoặc cột
nào đó thì bảng toàn ô trắng được hay không?
2 Có tất cả bao nhiêu cấu hình ban đầu sao cho sau hữu hạn bước đổi màu hàng hoặc cột thì
bảng gồm toàn ô trắng?
Ví dụ. Cấu hình H1 là 1 cấu hình thỏa mãn với n = 3.
5 Đề thi tỉnh Quảng Ninh.
A Ngày thi thứ nhất.
L Câu 1. Cho các số thực a, b, c ∈ [1; 2], chứng minh rằng
9 6 (a + b + c)

1
a
+
1
b
+
1
c

6 10
L Câu 2. Với mỗi số nguyên dương n, xét đa thức Pn(x) = xn
+ (x − 1)n
− (x + 1)n
.
1 Chứng minh rằng với mỗi n nguyên dương, đa thức Pn(x) có duy nhất một nghiệm dương, kí
hiệu là rn và rn  rn+1, ∀n ∈ N∗
.
2 Tính lim
rn
n
.
® Chinh phục olympic toán 9 h Tạp chí và tư liệu toán học
Š Tuyển tập đề chọn đội tuyển các tỉnh trên cả nước VMO 2021
L Câu 3. Cho tam giác ABC nội tiếp đường tròn (C) có AB  AC. Các điểm M, N lần lượt nằm
trên các cạnh AB, BC sao cho AM = CN. Gọi K là giao điểm của các đường thẳng MN và AC, P
và Q lần lượt là tâm đường tròn nội tiếp tam giác AMK và tâm đường tròn bàng tiếp góc K của
tam giác CNK. Gọi S là điểm chính giữa cung ABC của (C). Chứng minh rằng SP = SQ.
L Câu 4.
1 Cho các số nguyên dương a, n với a  2. Chứng minh rằng k | ϕ(ak
− 1), trong đó ϕ là phi
hàm Euler.
2 Cho số nguyên m  2 và p là một số nguyên tố không là ước của m nhưng p là ước của ϕ(m).
Chứng minh rằng tồn tại số nguyên tố q sao cho q ≡ 1 (mod p).
3 Cho p là một số nguyên tố. Chứng minh rằng có vô hạn số nguyên tố có dạng pk + 1.
B Ngày thi thứ hai.
L Câu 5. Tìm tất cả các hàm số f : R → R thỏa mãn f(x+y)−f(x)f(y) = f(xy)−2xy−1, ∀x, y ∈
R.
L Câu 6. Cho P(x) là đa thức bậc 4 với hệ số thực, có đúng 4 nghiệm dương phân biệt. Chứng
minh rằng phương trình sau cũng có 4 nghiệm dương phân biệt
1 − 3x
x4
· P(x) +

1 −
1 − 3x
x4

· P0
(x) − P00
(x) = 0
L Câu 7. Cho tam giác ABC nội tiếp đường tròn (O), điểm D cố định trên cung BC không chứa
A nhưng phải là điểm chính giữa cung đó. Điểm P di động trên đoạn AD và P 6= A, D. Điểm Q
đẳng giác với P trong tam giác ABC.
1 Gọi G là giao điểm của DQ và BC. Chứng minh rằng PG k AQ.
2 Kẻ dây cung DE của đường tròn (O) vuông góc với BC. K là trung điểm DE, R là hình chiếu
vuông góc của Q lên BC. Chứng minh rằng đường thẳng qua R và vuông góc với PK luôn đi
qua một điểm cố định khi P di động trên AD.
L Câu 8. Tìm tất cả các số nguyên dương n  60 sao cho tập hợp M = {n; n + 1; n + 2; ...; 60} có
thể viết được thành hợp của các tập con đôi một rời nhau thỏa mãn tính chất trong mỗi tập con
có một phần tử bằng tổng tất cả các phần tử còn lại trong tập con đó .
6 Đề thi tỉnh Lạng Sơn.
A Ngày thi thứ nhất.
L Câu 1.
1 Giải phương trình n
q
(x − 1)2
− 4 n
q
(x + 1)2
= −3 n
√
x2 − 1, với n ∈ N∗
, n  2.
2 Giải hệ phương trình

x3
− 2y3
= 4x2
y − 5xy2
√
x + y +
√
x − y = 2
.
L Câu 2. Cho các số thực dương a, b, c thỏa mãn a + b + c = 1. Chứng minh rằng
1
a
b + c
+
b
c + a
+
c
a + b
 3 −
9
2
(ab + bc + ca).
® Chinh phục olympic toán 10 h Tạp chí và tư liệu toán học
Š Tuyển tập đề chọn đội tuyển các tỉnh trên cả nước VMO 2021
2 27abc 6 a3
+ b3
+ c3
+ 24abc 6 1.
L Câu 3. Cho tam giác ABC nhọn và nội tiếp đường tròn (O) với các đường cao BE, CF(E ∈
CA, F ∈ AB). Gọi D = AO ∩ BC. Tiếp tuyến tại B và C của (O) cắt nhau tại S. Gọi P, Q lần lượt
là giao điểm của SB, SC với EF.
1 Chứng minh rằng
DB
DC
=
SQ
SP
=
sin

2 [
ACB

sin

2 [
ABC
.
2 Chứng minh rằng SD đi qua trung điểm của PQ.
3 Chứng minh rằng đường tròn ngoại tiếp tam giác SPQ tiếp xúc với (O).
L Câu 4. Cho dãy số (un) xác định bởi



u0 = 1
un+1 = un +
1
u2
n
, (n ∈ N)
.
1 Chứng minh rằng u3
n  1 + 3n với mọi n ∈ N∗
.
2 Tìm lim
u3
n
n
.
7 Đề thi học sinh giỏi Chuyên Lào Cai
L Câu 1. Cho dãy số (un) xác định bởi
(
u1 =
1
2
un+1 = 1 + un − u2
n, ∀n ∈ N∗
1 Chứng minh rằng
1
2
6 un 6
5
4
, ∀n ∈ N∗
.
2 Chứng minh dãy (un) có giới hạn hữu hạn. Tìm giới hạn hữu hạn đó.
L Câu 2. Cho đa thức hai đa thức Q(x) ∈ R[x] và P(x) = a + bx + cx2
+ x3
Q(x), trong đó a, b, c
là các số thực thỏa mãn ac 6= 0. Chứng minh rằng nếu tất cả các nghiệm của P(x) đều là số thực,
thì b2
 2ac.
L Câu 3. Cho tam giác ABC nhọn không cân nội tiếp đường tròn (O) có các đường cao BE, CF
cắt nhau tại H. Gọi I là trung điểm của BC. Tia IH cắt (O) tại T. Trên đường thằng EF lấy điểm
D sao cho HD k BC.
1 Chứng minh rằng DT tiếp xúc với đường tròn ngoại tiếp tam giác HEF.
2 Gọi M, N lần lượt là giao điểm của EF với các đường tròn (IBT), (ICT) thỏa mãn M khác
phía E đối với F và N khác phía F đối với E. Gọi P là giao điểm thứ hai của AH với (O).
Chứng minh rằng BM, CN, TP đồng quy.
L Câu 4. Cho p là số nguyên tố lớn hơn 3. Đặt
1 −
1
2
+
1
3
− . . . +
1
p − 2
−
1
p − 1
=
a
(p − 1)!
Chứng minh rằng a ≡
2 − 2p
p
(mod p).
L Câu 5. Cho tập hợp T = {1; 2; ...; 2020}. Có bao nhiêu cách chọn ra 10 số từ tập T sao cho hai
số bất kì trong các số được chọn luôn hơn kém nhau ít nhất 7 đơn vị.
® Chinh phục olympic toán 11 h Tạp chí và tư liệu toán học
Š Tuyển tập đề chọn đội tuyển các tỉnh trên cả nước VMO 2021
8 Trải nghiệm VMO 2021 - Bring Math to Everyone - BM2E
A Vòng 1, ngày thi thứ nhất.
L Câu 1. Cho α = 3 +
√
10. Các dãy số (xn)∞
n=1 , (yn)∞
n=1 được xác định như sau xn =
5α + 1
αn + α
và
yn = (1 + x1) (1 + x2) . . . (1 + xn) với mọi n = 1, 2, 3, .... Chứng minh rằng dãy (yn) có giới hạn hữu
hạn và tìm giới hạn đó.
L Câu 2. Tìm tất cả các hàm số f : R → R sao cho với mọi số thực x, y đẳng thức sau được thỏa
mãn
f(f(x) + y) + 1 = f (x2
+ y) + 2f(x) + 2y
L Câu 3. Cho tam giác nhon ABC nội tiếp đường tròn (C), AB  AC. Gọi P, Q là trung điểm
các cung nhỏ và cung lớn BC. M là chân đường vuông góc hạ từ Q xuống AB. Chứng minh rằng
đường tròn ngoại tiếp tam giác AMC đi qua trung điểm của đoạn thẳng AP.
L Câu 4. Trên bảng có các số 1, 2, 3, ..., 14, 15. An và Bình thực hiện quá trình sau: An chọn hai
số bất kỳ trên bảng (ví dụ là x, y) xóa chúng đi và viết lại tổng của hai số này lên bảng. Trong lúc
đó Bình viết giá trị của xy(x + y) và vở của bạn ấy. Hai bạn cứ làm như vậy cho đến khi trên bảng
chỉ còn lại một số thì dừng lại. Lúc đó Bình cộng tất cả các số mà bạn ấy đã viết vào vở, gọi tổng
thu được là S.
1 Chứng minh rằng S là một hằng số, tức là giá trị của S không phụ thuộc và thứ tự mà An và
Bình thực hiện.
2 Hãy tìm giá trị của S.
B Vòng 1, ngày thi thứ hai.
L Câu 5. Với mỗi số thực x, ký hiệu [x], là số nguyên lớn nhất không vượt quá x, gọi là phần
nguyên của x. Ta cũng ký hiệu {x} = x − [x] và gọi là phần lẻ của x. Xét tập hợp X các số nguyên
dương không lớn hơn 2020 và không phải là lập phương của một số nguyên.
1 Chứng minh rằng với mọi n thuộc X ta có { 3
√
n} 
1
500
.
2 Chứng minh rằng tồn tại n thuộc X sao cho { 3
√
n} 
1
432
.
L Câu 6. Một số nguyên dương được gọi là siêu hợp số nếu nó có thể phân tích thành tích của 3
thừa số nguyên dương  1. Ví dụ 12 là siêu hợp số vì 12 = 2.2.3. Một số nguyên dương được gọi là
mịn nếu mọi ước nguyên tố của nó đều nhỏ hơn hay bằng 19. Chứng minh rằng từ một tập hợp gồm
46 số mịn, luôn tìm được 2 số có tổng là một siêu hợp số.
L Câu 7. Cho tam giác ABC nhọn, không cân có các đường cao AD, BE, CF và (M) là đường
tròn đường kính BC. Đường thẳng qua A song song với EF cắt DE, DF theo thứ tự tại K, L.
1 Đường tròn ngoại tiếp tam giác AEF cắt KL, AM tại R, S. Chứng minh rằng ba đường thẳng
BK, CL và RS đồng quy.
2 Gọi (I), (J) theo thứ tự là đường tròn nội tiếp của các tam giác ADK, ADL. Chứng minh rằng
ba đường tròn (I), (J) và (M) có một tiếp tuyến chung.
® Chinh phục olympic toán 12 h Tạp chí và tư liệu toán học
Š Tuyển tập đề chọn đội tuyển các tỉnh trên cả nước VMO 2021
9 Đề thi tỉnh Sóc Trăng.
A Sóc Trăng ngày thi thứ nhất.
L Câu 1. Giải hệ phương trình sau trên tập số thực
(
x2021
− y2022
= y4042
− xy2020
x2
− 2y2
= 2
√
2x − 1
.
L Câu 2.
1 Tìm nghiệm nguyên của phương trình
x4
+ y4
− z4
+ 2z3
+ 2x2
y2
+ 3x2
+ 4y2
+ 21z2
− 22z − 120 = 0
2 Có bao nhiêu số tự nhiên có 3 chữ số khác nhau và chia hết cho 3 ?
L Câu 3. Cho hình vuông ABCD và một đường thẳng ∆ đi qua A, Gọi E, F lần lượt là hình chiếu
của B, D trên ∆.
1 Chứng minh DE⊥CF.
2 Gọi G, H lần lượt là giao điểm của BF với EC và ED, I là giao điểm của CH với ∆. Chứng
minh bốn điểm C, F, G, I cùng nằm trên một đường tròn.
L Câu 4. Cho các số thực dương a, b, c thỏa mãn
1
a
+
1
b
+
1
c
= 2034. Tìm giá trị lớn nhất của biểu
thức
P =
1
4a + 10b + 2020c
+
1
4c + 10a + 2020b
+
1
4b + 10c + 2020a
B Sóc Trăng ngày thi thứ hai.
L Câu 1. Cho dãy số (xn) thỏa
x1 = 2020, xn+1 =
xn
2021
+ xn + 25
xn
2020 − xn + 11
∀n ∈ N∗
Đặt un =
1
x1
2020 + 6
+
1
x2
2020 + 6
+
1
x3
2020 + 6
....... +
1
xn
2020 + 6
=
n
X
i=1
1
xi
2020 + 6
.
Tính lim un.
L Câu 2. Cho đa giác đều có 1000 đỉnh A1A2A3A4....An nội tiếp đường tròn (O).
1 Có bao nhiêu hình chữ nhật có các đỉnh là đỉnh của đa giác và hình chữ nhật đó có kích thước
khác nhau?
2 Cho quy tắc sau: “Trên đường tròn (O) đánh dấu đỉnh A, đi theo chiều tăng của thứ tự các
đỉnh lần lượt đánh dấu các đỉnh cách đỉnh vừa được đánh dấu 15 đỉnh, đến gặp vị trí đỉnh đã
được đánh dấu thì kết thúc”. Hỏi khi quy tắc này kết thúc, trên đường tròn (O) còn lại bao
nhiêu đỉnh chưa đámh dấu, các đỉnh này có lập thành đa giác đều hay không?
L Câu 3. Cho tam giác ABC có ba góc nhọn và các đường cao AE, BF, CI.
1 Cho ∠BAC = 60◦
và AD là phân giác trong của góc A(D ∈ BC). Chứng minh
AD
AB
+
AD
AC
=
√
3.
2 Gọi M, Q lần lượt là hình chiếu vuông góc của E trên AB, AC; N, R lần lượt là hình chiếu
vuông góc của F trên AB, BC; P, S lần lượt là hình chiếu vuông góc của I trên AC, BC.
Chứng minh các điểm M, N, P, Q, R, S cùng nằm trên một đường tròn.
® Chinh phục olympic toán 13 h Tạp chí và tư liệu toán học
Š Tuyển tập đề chọn đội tuyển các tỉnh trên cả nước VMO 2021
10 Đề thi tỉnh Nghệ An.
A Ngày thi thứ nhất.
L Câu 1. Cho α là số thực thuộc khoảng (1; 2). Xét dãy số dương (un) thỏa mãn
uα
n  u1 + u2 + ... + un−1, ∀n  2
Chứng minh rằng tồn tại hằng số C sao cho un  Cn, ∀n ∈ N∗
.
L Câu 2. Tìm tất cả các đa thức hệ số nguyên P(x) thỏa mãn a2
+b2
−c2
là ước của P(a)+P(b)−
P(c) với mọi bộ số tự nhiên a, b, c mà a2
+ b2
− c2
6= 0.
L Câu 3. Cho tứ giác lồi ABCD có AB = BD = DA, CB  CD, các đường chéo AC và BD cắt
nhau tại O, Gọi K là trung điểm của AO. Trên tia đối của tia BA lấy điểm E sao cho BE = DO,
trên tia đối của tia DA lấy điểm F sao cho DF = BO. Các đường thẳng KE, KF lần lượt cắt các
đường thẳng CB, CD tại M, N. Các đường thẳng BD, MN cắt nhau tại I. Chứng minh AK vuông
góc với KI.
L Câu 4. Cho bảng ô vuông với m hàng, n cột m, n ∈ N∗
. Ta tô màu k ô vuông của bảng thỏa
mãn: với 4 ô vuông bất kỳ mà tâm của chúng là 4 đỉnh của một hình chữ nhật (có cạnh song song
với cạnh của bảng ô vuông) thì chỉ cho phép tối đa 2 ô trong đó được tô màu. Tìm giá trị lớn nhất
có thể của k.
B Ngày thi thứ hai
L Câu 5. Cho tập hợp A gồm hữu hạn các số thực dương. B là tập hợp gồm các số có dạng
a + b
c + d
với
a, b, c, d là các số thuộc tập hợp A. Chứng minh rằng bất đẳng thức sau luôn đúng : |B|  2|A|2
−1.
L Câu 6. Cho tam giác ABC cân tại A nội tiếp (O). Gọi I là tâm nội tiếp tam giác ABC. BI, CI
cắt lại (O) tại M, N. Điểm D bất kỳ nằm trên cung BC không chứa A, DB  DC,D khác B, C.
AD cắt BI, CI tại E, F. DM, DN cắt CI, BI tại P, Q.
1 Chứng minh rằng D, I, P, Q cùng nằm trên một đường tròn. Giả sử đường tròn đó là (ω).
2 Chứng minh rằng CE, BF cắt nhau tại một điểm thuộc đường tròn (ω).
L Câu 7. Tồn tại hay không một tập hợp M thỏa mãn
1 M là tập hợp gồm 2020 số tự nhiên phân biệt.
2 Tổng tất cả các phần tử của các tập con bất kỳ khác rỗng của M đều là một số tự nhiên có
dạng mk
với m, k là số tự nhiên, k  2.
11 Đề thi Chuyên Sư Phạm Hà Nội 2020
A Ngày thi thứ nhất.
L Câu 1. Cho dãy số (an) được xác định bởi a1 = 1 và an+1 =
an
√
an + 1
với n = 1, 2, 3, ... Chứng
minh rằng lim
n→+∞
(nan) = 0.
L Câu 2. Cho số thực q. Biết rằng tồn tại một cấp số cộng (an) tăng ngặt sao cho với mỗi số
nguyên dương k thì qk
là một số hạng của (an). Chứng minh rằng q là số nguyên.
® Chinh phục olympic toán 14 h Tạp chí và tư liệu toán học
Š Tuyển tập đề chọn đội tuyển các tỉnh trên cả nước VMO 2021
L Câu 3. Cho tam giác ABC không cân, có trọng tâm G và đường tròn nội tiếp (I) tiếp xúc với
cạnh BC tại D. Gọi M, N, P lần lượt là trung điểm của các cạnh BC, CA, AB. Qua D, kẻ đường
thẳng song song với AI cắt IM tại S.
1 Chứng minh rằng AS = ID.
2 Gọi X, Y là các điểm theo thứ tự thuộc MN, MP sao cho PX, NY là các tiếp tuyến của đường
tròn (I). Chứng minh rằng IG vuông góc với XY .
L Câu 4. Có 2020 tấm thẻ đỏ và 2020 tấm thẻ xanh. Trên mỗi tấm thẻ, người ta viết một số
nguyên dương không vượt quá 2020 (hai tấm thẻ khác nhau có thể viết cùng một số). Chứng minh
rằng có thể chọn ra một bộ thẻ đỏ và một bộ thẻ xanh sao cho tổng các số viết trên mỗi tấm thẻ ở
hai bộ là bằng nhau (chú ý rằng một bộ có thể chỉ gồm 1 tấm thẻ).
B Ngày thi thứ hai.
L Câu 5. Tìm tất cả các hàm số f : R → R thỏa mãn f(x) = x.f(x2
) với mọi x ∈ [0; 1].
L Câu 6. Cho dãy số (xn) được xác định bới x0 = 1, x1 = 3 và xn+2 = 6xn+1 − xn với mọi n ∈ N.
1 Chứng minh rằng với mỗi số nguyên dương n thì (x2
n − 1)2
+ 1 là hợp số.
2 Cho k là số nguyên dương và p là một ước nguyên tố của x2k . Chứng minh rằng nếu p − 1 chia
hết cho 4 thì p − 1 chia hết cho 2k+2
.
L Câu 7. Cho tam giác ABC có AB  AC, nội tiếp trong đường tròn (O). Gọi M, N theo thứ
tự là trung điểm BC, OA. Xét D là một điểm thuộc đoạn thẳng MC, không trùng với hai đầu mút
và E, F lần lượt là hình chiếu của B, C lên AD. Gọi G là giao điểm của đường thẳng qua E, song
song AB và đường thẳng qua F, song song AC. Gọi I là tâm đường tròn ngoại tiếp tam giác GEF.
Chứng minh rằng NI = NM.
L Câu 8. Một hội nghị có n người tham dự (n  5). Biết rằng tồn tại số nguyên dương m sao cho
trong n − 2 người bất kỳ tham dự hội nghị, luôn có đúng 3m
cặp quen nhau. Tìm tất cả các giá trị
có thể có của n.
12 Đề thi tỉnh Vũng Tàu
A Ngày thi thứ nhất.
L Câu 1.
1 Giải hệ phương trình





1
√
x
−
3
7x − y
=
1
2
1
√
y
+
6
7x − y
= 2
.
2 Cho các số thực dương x, y, z. Tìm giá trị lớn nhất của biểu thức
P =
x
p
(2x + y)(2x + z)
+
y
p
(2y + x)(2y + z)
+
z
p
(2z + x)(2z + y)
L Câu 2. Cho a là một số thực và dãy số (xn) xác định bởi
x1 = a; xn+1 =





√
xn + 3 −
(2n + 1)xn − n
n + 1
nếu xn 6 1
xn +
n
xn
nếu xn  1
, n ∈ N∗
® Chinh phục olympic toán 15 h Tạp chí và tư liệu toán học
Š Tuyển tập đề chọn đội tuyển các tỉnh trên cả nước VMO 2021
1 Khi a = 2. Chứng minh rằng lim
xn
n
= 1 và xác định tất cả các số thực β sao cho dãy số (yn)
xác định bởi yn = nβ √
xn+1 −
√
xn

, ∀n ∈ N∗
có giới hạn hữu hạn khác 0.
2 Tìm tất cả các giá trị a  0 sao cho dãy số
xn
n

có giới hạn.
L Câu 3. Cho tam giác ABC có 3 góc nhọn ngoại tiếp đường tròn tâm (I) với AB  AC. Gọi
D, E, F lần lượt là tiếp điểm của (I) với BC, CA, AB. Các đường thằng ID và EF cắt nhau tại J.
Đường thằng AJ cắt đường tròn tâm (I) tại các điểm K, L với K nằm giữa A và L. Đường thẳng
qua A song song với BC cắt ID, EF lần lượt tại N và S. Đường thẳng qua K và song song với BC
cắt (I) tại điểm X(X 6= K). Đường thằng qua L song song với BC cắt (I) tại điểm Y (Y 6= L). Các
đường thẳng AX, AY cắt BC lần lượt tại Q và P.
1 Chứng minh rằng ND là phân giác của 
ENF và AJ đi qua trung điểm M của BC.
2 Chứng minh rằng M là trung điểm của PQ.
L Câu 4. Tìm tất cả các hàm số f : (0; +∞) → R thỏa mãn
f(x) + f(y) =
q
x
y
+
py
x

f(
√
xy); ∀x, y  0
L Câu 5.
1 Cho số nguyên tố p  7. Đặt 1+
1
2
+. . .+
1
p2 − 1
=
A
B
với A, B là các số nguyên dương nguyên
tố cùng nhau. Chứng minh rằng B không chia hết cho p và A chia hết cho p.
2 Trong không gian cho N điểm, trong đó không có 3 điểm nào thẳng hàng (N là số nguyên
dương lớn hơn 3). Tất cả các cặp điểm trên được nối với nhau bởi

N
2

đoạn thẳng. Mỗi đoạn
thẳng được tô một trong hai màu xanh hoặc đỏ và thỏa mãn 2 điều kiện sau
(a) Không có tam giác nào có đúng 1 cạnh xanh.
(b) Không có 13 điểm nào mà tất cả các đoạn nối được tô cùng màu.
Chứng minh rằng N 6 144.
13 Đề thi phổ thông năng khiếu
A Ngày thi thứ nhất.
L Câu 1. Với mỗi số nguyên dương n, tìm số thực Mn  0 lớn nhất sao cho với mọi số dương
x1, x2, ..., xn đều có
n
X
k=1
1
x2
k
+
1
n
X
k=1
xk
!2  Mn






n
X
k=1
1
xk
+
1
n
X
k=1
xk






2
L Câu 2. Cho 2021 số nguyên khác 0. Biết rằng tổng của một số bất kỳ trong chúng với tích của
tất cả 2020 số còn lại luôn âm
® Chinh phục olympic toán 16 h Tạp chí và tư liệu toán học
Š Tuyển tập đề chọn đội tuyển các tỉnh trên cả nước VMO 2021
1 Chứng minh rằng với mọi cách chia 2021 số này thành hai nhóm và nhân các số cùng nhóm
với nhau thì tổng của hai tích thu được cũng luôn âm.
2 Một bộ số thỏa mãn điều kiện đề bài có thể có nhiều nhất bao nhiêu số âm?
L Câu 3. Cho 2 hàm số f : R → R và g : R → R, thỏa g(2020)  0 và
(
f (x − g (y)) = f (−x + 2g (y)) + xg (y) − 6
g (y) = g (2f (x) − y)
, ∀x, y ∈ R
1 Chứng minh rằng g là hàm hằng.
2 Chứng minh đồ thị hàm số h(x) = f(x) − x nhận đường thẳng x = 1 làm trục đối xứng.
L Câu 4. Tam giác ABC nhọn nội tiếp đường tròn (O) có H là trực tâm. AH, BH, CH lần lượt cắt
BC, CA, AB tại D, E, F và I, M, N là trung điểm của BC, HB, HC. BH, CH cắt (O) tại L, K(L 6=
B, K 6= C); KL cắt MN tại G.
1 Trên EF lấy điểm T sao cho AT vuông góc với IH. Chứng minh GT vuông góc với OH.
2 DE, DF lần lượt cắt MN tại P, Q. Gọi S là giao điểm của BQ với CP. Chứng minh rằng HS
qua trung điểm của EF.
B Ngày thi thứ hai.
L Câu 5. Cho số nguyên dương n  1. Chứng minh rằng với mọi số thực a ∈

0;
1
n

và mọi đa
thức P(x) có bậc 2n + 1 thỏa mãn điều kiện P(0) = P(1) = 0 luôn tồn tại các số thực x1, x2 thuộc
[0; 1] sao cho P (x1) = P (x2) và x2 − x1 = a.
L Câu 6. Giải phương trình sau trên tập số nguyên dương
(x2
+ 3)
3x+1
h
(x2
+ 3)
3x+1
+ 1
i
+ x2
+ y = x2
y
L Câu 7. Cho các số nguyên n  k  t  0 và X = {1, 2, . . . , n}. Gọi F là họ các tập con có k
phần tử của tập hợp X sao cho với mọi F, F0
∈ F thì |F ∩ F0
|  t. Giả sử không có tập con có t
phần tử nào được chứa trong tất cả các tập F ∈ F.
1 Chứng minh rằng tồn tại một tập hợp B ⊂ X sao cho |B|  3k và |B ∩ F|  t + 1 với mọi
F ∈ F.
2 Chứng minh rằng |F| 

3k
t + 1

n
k − t − 1

.
L Câu 8. Cho tam giác ABC nội tiếp trong đường tròn (O) với B, C cố định và A thay đổi trên
cung lớn BC. Dựng hình bình hành ABDC và AD cắt (BCD) ở K.
1 Gọi R1, R2 lần lượt là bán kính đường tròn ngoại tiếp (KBC), (KAC). Chứng minh rằng tích
R1R2 không đổi.
2 Ký hiệu (T), (T0
) lần lượt là các đường tròn cùng đi qua K, tiếp xúc với BD ở B và tiếp xúc
với CD ở C. Giả sử (T), (T0
) cắt nhau ở L 6= K. Chứng minh rằng AL luôn đi qua một điểm
cố định.
® Chinh phục olympic toán 17 h Tạp chí và tư liệu toán học
Š Tuyển tập đề chọn đội tuyển các tỉnh trên cả nước VMO 2021
14 Đề thi tỉnh Ninh Bình.
L Câu 1.
1 Giải phương trình (2x − 4)
√
3x − 2 +
√
x + 3 = 5x − 7 +
√
3x2 + 7x − 6.
2 Cho các số thực dương x, y. Chứng minh rằng
(
√
x +
√
y)

1
√
x + 3y
+
1
√
y + 3x

6 2
L Câu 2. Cho dãy số (un) được xác định bởi công thức un =

22n+1
+ n2
+ n + 2
2n+1 + 2

, n ∈ N, trong
đó [x] là số nguyên lớn nhất không vượt quá x và {x} = x − [x].
1 Tính sáu số hạng đầu của dãy (un).
2 Tính giới hạn của dãy số (un).
3 Có bao nhiêu số hạng của dãy số (un) với n 6 86 thỏa mãn
2526 · 2n−99
2n + 1
6 un 6
23
65
.
L Câu 3. Cho tam giác ABC nội tiếp đường tròn tâm (O). Đường tròn ngoại tiếp tam giác OBC
có tâm S, cắt đường thẳng AB tại điểm X khác B và cắt đường tròn Euler của tam giác ABC tại
hai điểm D, E. Gọi K, L theo thứ tự là các điểm đối xứng của S qua AB, AC. Chứng minh rằng
1 XO⊥AC.
2 Đường thằng KL đi qua tâm đường tròn Euler của tam giác ABC và hai đường thẳng AD, AE
đối xứng nhau quá đường phân giác trong của [
BAC.
L Câu 4.
1 Cho số nguyên tố p, số nguyên dương a thỏa mãn 1  a  p + 1 và q là ước nguyên tố của
A = 1 + a + · · · + ap−1
. Chứng minh rằng q − 1 chia hết cho p.
2 Cho số nguyên dương n. Có bao nhiêu số tự nhiên chia hết cho 3, có n chữ số và các chữ số
đều thuộc tập A = {3; 4; 5; 6; 9}.
15 Đề thi thành phố Cần Thơ
L Câu 1. Cho dãy số xác định bởi



u1 = 1
un+1 =
u2
n + 2n
3un
, n ∈ N∗
.
1 Chứng minh rằng
√
n − 1 6 un 6
√
n, ∀n ∈ N∗
.
2 Tìm lim
n→+∞
un
√
n
.
L Câu 2. Cho x1, x2, ..., x2020 là các số thực dương. Chứng minh rằng
1 + x2
1
1 + x1x2
+
1 + x2
2
1 + x2x3
+ ... +
1 + x2
2020
1 + x2020x1
 2020.
® Chinh phục olympic toán 18 h Tạp chí và tư liệu toán học
Š Tuyển tập đề chọn đội tuyển các tỉnh trên cả nước VMO 2021
L Câu 3. Cho hàm số f xác định trên tập số thực R thỏa mãn
(
f(xy) = xf(y) + yf(x)
f(x + y) = f x2021

+ f y2021

, ∀x, y ∈ R.
1 Chứng minh rằng f là hàm cộng tính trên R.
2 Tính f
√
2020

.
L Câu 4. Cho tam giác ABC có đường tròn nội tiếp (I) tiếp xúc với ba cạnh BC, CA, AB lần lượt
tại D, E, F. Gọi K, M, Nlần lượt là giao điểm của EF, FD, DE với BC, CA, AB. Đường tròn đường
kính KD, EM, FN lần lượt cắt (I) tại A1, B1, C1.
1 Chứng minh rằng các đường tròn đường kính KD, EM, FN đồng trục.
2 Chứng minh rằng DA1, EB1, FC1 đồng quy tại một điểm J.
3 Gọi T là trung điểm của EF. Chứng minh tiếp tuyến tại B và tại C của đường tròn ngoại tiếp
tam giác IBC cắt nhau tại một điểm thuộc JT.
L Câu 5. Cho n là số nguyên dương, n không có ước là số chính phương khác 1. Tồn tại hay không
hai số nguyên dương nguyên tố cùng nhau x, y để xn
+ yn
chia hết cho (x + y)3
?
L Câu 6. Một tàu du lịch có 100 khoang với sức chứa trong các khoang lần lượt là 101, 102, ..., 200
người. Hiện tại trên tàu đang có tổng cộng n người. Bây giờ, có một khách VIP đến và người thuyền
trưởng muốn cấp cho anh ta một khoang cá nhân. Với mục đích đó, người thuyền trưởng muốn chọn
hai khoang A và B,sau đó chuyển tất cả khách từ khoang A sang khoang B mà không vượt quá khả
năng chứa của khoang B. Xác định n lớn nhất mà người thuyền trưởng có thể chắc chắn đạt được
mục tiêu của mình cho dù lúc đầu n người này phân bố ở các khoang tàu như thế nào chăng nữa.
16 Đề thi tỉnh Bắc Ninh.
A Ngày thi thứ nhất.
L Câu 1. Cho số thực a và xét dãy số (xn) thỏa mãn
x1 = x2 = 1, x3 = 0, xn+3 =
x2
n+2 + x2
n+1 + x2
n
6
+ a, ∀x ∈ N∗
1 Chứng minh rằng với a = 0 thì dãy (xn) hội tụ.
2 Tìm số thực a lớn nhất sao cho dãy (un) hội tụ.
L Câu 2. Cho tập S = {p1, p2, ..., p2021} gồm 2021 số nguyên tố phân biệt và P(x) là đa thức với
các hệ số nguyên sao cho với mỗi số nguyên dương n đều tồn tại pi trong tập S là ước của P(n).
Chứng minh rằng tồn tại pk trong tập S sao cho pk là ước của P(2021).
L Câu 3. Cho tam giác ABC nhọn, không cân, nội tiếp đường tròn (O), có đường cao AD. Trên
các cạnh AC, AB lần lượt lấy các điểm E, F sao cho AD, BE, CF đồng quy tại H. Giả sử tứ giác
EFBC nội tiếp.
1 Chứng minh rằng H là trực tâm của tam giác ABC.
2 Đường thẳng vuông góc với OD tại D cắt AB tại K. Chứng minh rằng

DHK + 
AHC = 180◦
L Câu 4. Tìm tất cả các hàm số f : R → R thỏa mãn f (x2020
+ f (y)) = y+(f (x))2020
, ∀x, y ∈ R.
® Chinh phục olympic toán 19 h Tạp chí và tư liệu toán học
Š Tuyển tập đề chọn đội tuyển các tỉnh trên cả nước VMO 2021
B Ngày thi thứ hai.
L Câu 5. Cho dãy số (an)+∞
n=0 xác định như sau
a0 = 0, a1 = 1, an+2 = 2an+1 − pan, ∀n ∈ N
Biết rằng p là số nguyên tố và trong dãy có số hàng bằng −1. Tìm tất cả các giá trị có thể của p.
L Câu 6. Cho đường tròn (O) bán kính R tiếp xúc với đường thẳng d tại điểm T cho trước. Một
điểm M di động trên (O), tiếp tuyến của (O) tại M cắt d tại P. Gọi (C) là đường tròn tâm J qua
M và tiếp xúc với d tại P và I là điểm đối xứng với P qua J.
1 Chứng minh OI = IP và (C) tiếp xúc với đường tròn cố định.
2 Tìm quỹ tích tâm J của đường tròn (C) khi M di động trên (O).
L Câu 7. Một thành phố phát động phong trào đi bộ cho người dân. Thống kê điều tra cho thấy,
trong tháng 9 dương lịch, mỗi ngày có ít nhất 84% tổng số người dân đi bộ. Trong một ngày, ta gọi
hai người dân là một cặp chăm chỉ nếu trong ngày đó có ít nhất một trong hai người đi bộ.
Chứng tỏ rằng, luôn tìm được hai người là cặp chăm chỉ trong tất cả các ngày của tháng 9. (Giả
sử rằng, số người dân là số nguyên lớn hơn 2).
17 Đề thi tỉnh Lâm Đồng.
A Ngày thi thứ nhất.
L Câu 1. Chứng minh rằng (x2
+ 2) (y2
+ 2) (z2
+ 2)  9 (xy + yz + zx) , ∀x, y, z  0.
L Câu 2. Cho hàm số f(x) =
ex
(x + 1)2 .
1 Chứng minh rằng phương trình f(x) = x có duy nhất một nghiệm trong

1
2
, 1

.
2 Chứng minh dãy số (un) xác định bởi u1 = 1, un+1 = f (un) , ∀n ∈ N∗
có giới hạn.
L Câu 3. Cho tam giác ABC nhọn, không cân, nội tiếp trong đường tròn (O). Gọi H là hình chiếu
của A lên BC và D, E, M lần lượt là trung điểm HB, HC, BC. Đường tròn (ABE) tâm I cắt AC
tại S và đường tròn (ACD) tâm J cắt AB tại R.
1 Chứng minh rằng BC = 4IJ.
2 Trung tuyến đỉnh H của tam giác AHM cắt RS tại T, chứng minh rằng các đường thẳng
AT, BS, CR đồng quy.
L Câu 4. Cho số a = 2019·2020·2021 và số nguyên dương n  3. Người ta xếp n số nguyên dương
nào đó lên một đường tròn thỏa mãn đồng thời hai điều kiện sau
(i) Hai số nằm cạnh nhau có tích không chia hết cho a.
(ii) Hai số không nằm cạnh nhau có tích chia hết cho a.
1 Tìm một bộ các số nguyên dương thỏa mãn cách xếp trên.
2 Tìm giá trị lớn nhất của n.
® Chinh phục olympic toán 20 h Tạp chí và tư liệu toán học
Š Tuyển tập đề chọn đội tuyển các tỉnh trên cả nước VMO 2021
L Câu 5. Cho tập S = {1, 2, . . . , n} với n là số nguyên dương. Gọi An là tập hợp các hoán vị
(a1, a2, . . . , an) của tập S thỏa mãn điều kiện 2 (a1 + a2 + · · · + ak) chia hết cho k với mọi k =
1, 2, . . . , n.
1 Chứng minh rằng an − 1 chia hết cho n − 1 khi n chẵn và n  3.
2 Tìm số phần tử của A2020.
18 Đề thi tỉnh Bắc Giang
L Câu 1. Cho các số thực a, b, c thỏa mãn 0 6 a 6 b 6 c và a + b + c = ab + bc + ca.
Chứng minh rằng
√
bc(a + 1)  2.
L Câu 2. Cho tam giác ABC nhọn nội tiếp đường tròn (O), đường cao AD, trực tâm H. Đường
tròn đường kính AH cắt (O) tại điểm Q khác A. Đường tròn đường kính HQ cắt (O) tại điểm K
khác Q. Gọi M là trung điểm BC.
1 Đường thẳng qua H vuông góc với MH cắt BC tại X. Chứng minh rằng XK tiếp xúc với
đường tròn ngoại tiếp tam giác KDM.
2 Đường thẳng KQ cắt đường tròn ngoại tiếp tam giác KDM tại N khác K. Chứng minh rằng
MN chia đôi AQ.
L Câu 3. Cho số thực a và dãy số (un)n1 xác định bởi u1 = a, un+1 = u2
n + un + a3
(n  1) .
1 Chứng minh rằng, với dãy a ∈

−
1
2
; 0

, dãy số hội tụ và tìm giới hạn đó.
2 Cho a = 2020. Chứng minh rằng u2
n + 20203
luôn có ít nhất n+4 ước số nguyên tố khác nhau.
L Câu 4.
1 Tìm tất cả các số tự nhiên k sao cho 2k + 1 và 4k + 1 đều là các số chính phương.
2 Với mỗi số tự nhiên k thỏa mãn đề bài, chứng minh rằng 35 | (k2
− 12k).
L Câu 5. Sắp đến ngày Tết Trung thu, tổ chức Smile Foundation của trường THPT Chuyên Bắc
Giang làm bánh gây quỹ từ thiện thường niên. Sản phẩm năm nay là một cặp bánh dẻo, bánh nướng
có tổng giá cặp bánh đó là 50000 đồng. Do số lượng có hạn nên mỗi bạn chỉ được mua đúng một
cặp. Để mua bánh các bạn học sinh trường chuyên phải xếp hàng. Biết rằng trong hàng có m + n
bạn, trong đó m bạn cầm tờ 50000 đồng và n bạn cầm tờ 100000 đồng (m, n ∈ N∗
, m  n). Hỏi có
bao nhiêu cách xếp hàng để không bạn nào phải chờ tiền trả lại, giả thiết rằng ban đầu ban tổ chức
không cầm theo đồng tiền nào.
19 Đề thi tỉnh Đồng Tháp.
L Câu 1. Với mỗi số nguyên dương n  2, xét số thực un  1 sao cho phương trình [unx] = x có
đúng n nghiệm nguyên (theo ẩn x và [unx] là phần nguyên của unx).
1 Chứng minh rằng [un] = 1, ∀n ∈, n  2.
2 Với mỗi cách xác định của dãy (un) thỏa điều kiện trên. Chứng minh rằng dãy (un) luôn có
giới hạn và tìm giới hạn ấy.
® Chinh phục olympic toán 21 h Tạp chí và tư liệu toán học
Š Tuyển tập đề chọn đội tuyển các tỉnh trên cả nước VMO 2021
L Câu 2.
1 Giải hệ phương trình

(x + 1) (y + 1) (z + 1) = 5
√
x +
√
y +
√
z
2
= x + 6
2 Xét số T = 3n
− 2n
, trong đó n là số nguyên dương, n  2.
(a) Chứng minh rằng không tồn tại n để T là bình phương của một số nguyên tố.
(b) Nếu T là lập phương của một số nguyên tố thì n là một số nguyên tố.
L Câu 3. Với mỗi m ∈ N∗
ta kí hiệu α (2m) = (m!)2
, α (2m + 1) = (m!) . ((m + 1)!). Cho đa thức
p(x) hệ số nguyên, có bậc lớn hơn hoặc bằng k (k ∈ N∗
) và có ít nhất k nghiệm nguyên phân biệt.
Xét số nguyên n (n 6= 0) sao cho đa thức q(x) = p(x) − n có ít nhất một nghiệm nguyên. Chứng
minh rằng |n|  α(k).
L Câu 4. Cho tam giác ABC, đường tròn nội tiếp (I) tiếp xúc với các cạnh BC, CA, AB tại
D, E, F.
1 Gọi S là giao điểm của EF với BC. Chứng minh SI vuông góc với AD.
2 Đường thẳng d thay đổi, đi qua S và cắt đường tròn (I) tại hai điểm phân biệt M, N. Các tiếp
tuyến tại M, N của (I) cắt nhau tại T. Chứng minh T thuộc một đường thẳng cố định.
3 Gọi K là giao điểm của ME và NF, G là giao điểm của MC và NB. Chứng minh K và G
cùng thuộc đường thẳng AD.
L Câu 5. Viết n số thực có tổng bằng n − 1 (n  1) quanh một đường tròn. Chứng minh rằng ta
có thể gắn nhãn cho các số đó theo chiều kim đồng hồ là x1, x2, . . . , xn sao cho
x1 + x2 + . . . + xk  k − 1, ∀1 6 k 6 n.
20 Đề thi thành phố Hà Nội.
L Câu 1. Cho hàm số y = x3
−
3
2
mx2
+ m3
có đồ thị (Cm). Tìm tất cả các giá trị của tham số m
để hàm số có hai điểm cực trị A, B sao cho tam giác ABO có diện tích bằng 32 (với O là gốc tọa
độ).
L Câu 2.
1 Giải phương trình x3
+ 1 =
√
4x − 3 +
√
2x − 1.
2 Giải hệ phương trình

y3
+ y = x2
+ 2 (1)
8y3
− 3y = 2x2
− 3
p
2x2 + y + 7 + 7 (2)
L Câu 3. Cho đa giác đều 30 đỉnh A1, A2, ..., A30. Hỏi có bao nhiêu tam giác có 3 đỉnh là 3 điểm
trong 30 điểm A1, A2, ..., A30 đồng thời không có cạnh nào là cạnh của đa giác.
L Câu 4. Cho hình lập phương ABCD.A0
B0
C0
D0
có cạnh bằng 1. Gọi M, N là hai điểm thay đổi
lần lượt trên các cạnh AB, A0
D0
sao cho đường thẳng MN tạo với mặt phẳng (ABCD) một góc
bằng 60◦
.
1 Tính độ dài đoạn thẳng MN.
2 Tìm giá trị lớn nhất của khoảng cách giữa hai đường thẳng MN và CC0
.
® Chinh phục olympic toán 22 h Tạp chí và tư liệu toán học
Š Tuyển tập đề chọn đội tuyển các tỉnh trên cả nước VMO 2021
L Câu 5. Cho dãy số (un) xác định bởi u1 = 6, un+1 =
1
2
(u2
n − 4un + 9) ; n = 1, 2, .....
1 Chứng minh dãy số là dãy số tăng.
2 Chứng minh
1
u1 − 1
+
1
u2 − 1
+ ... +
1
u2020 − 1

1
3
.
L Câu 6. Cho các số thực a, b, c thỏa mãn a2
+ b2
+ c2
= ab + bc + ca + 6. Tìm giá trị nhỏ nhất
của biểu thức P = (a − b) (b − c) (c − a) .
21 Đề thi tỉnh Hà Nam.
L Câu 1.
1 Cho hàm số f(x) = −x +
p
(a + x)(b + x) trong đó a, b là hai số thực dương khác nhau cho
trước. Chứng minh rằng với mỗi số thực s ∈ (0; 1) đều tồn tại duy nhất số thực dương x0 sao
cho f (x0) =

as
+ bs
2
1
s
.
2 Xếp 35 học sinh, trong đó có bốn bạn Dũng, Minh, Công, Đoàn thành một hàng ngang. Hỏi
có tất cả bao nhiêu cách xếp hàng, mà trong mỗi cách xếp hàng không có ba bạn nào trong
bốn bạn Dũng, Minh, Công, Đoàn đứng ở ba vị trí liên tiếp.
L Câu 2. Cho hàm số f(x) =
x3
− 3x2
+ 3x + 5
x + 1
.
1 Chứng minh đồ thị hàm số có ba điểm cực trị không thẳng hàng.
2 Gọi A, B, C là ba điểm cực trị của đồ thị hàm số. Tính diện tích tam giác ABC.
L Câu 3. Cho p là một số nguyên tố lẻ và a là một số nguyên sao cho (a, p) = 1. Chứng minh
phương trình x2
≡ a (mod p) có nghiệm khi và chỉ khi a
p−1
2 ≡ 1 (mod p).
L Câu 4. Tìm tất cả các hàm f : R → R thỏa mãn điều kiện
f (x2
) + 4y2
f(y) = (f(x − y) + y2
) .(f(x + y) + f(y)) với mọi x, y ∈ R.
L Câu 5. Cho tứ giác ABCD cố định có hai đường chéo AC, BD cắt nhau tại P. Đường trung
trực của các đoạn thẳng AC và BD cắt nhau tại K. Một đường thẳng d thay đổi đi qua K, cắt
đường tròn ngoại tiếp tam giác PAB tại Q, R. Chứng minh trực tâm tam giác PQR luôn nằm trên
một đường tròn cố định, khi đường thẳng d thay đổi.
22 Đề thi tỉnh Kiên Giang.
A Ngày thi thứ nhất.
L Câu 1. Cho dãy số (xn) được xác định như sau
x1 =
7
3
, xn+1 = x2
n + 2xn − 2, ∀n ∈ N∗
1 Tìm số hạng tổng quát của dãy số (xn).
® Chinh phục olympic toán 23 h Tạp chí và tư liệu toán học
Š Tuyển tập đề chọn đội tuyển các tỉnh trên cả nước VMO 2021
2 Tìm lim
n→+∞

1
1 + x1
+
1
(1 + x1)(1 + x2)
+ ... +
1
(1 + x1)(1 + x2)...(1 + xn)

.
L Câu 2. Tìm tất cả các hàm số liên tục f : R → R sao cho
8f(4x) − 10f(2x) + 3f(x) = 30x, ∀x ∈ R
L Câu 3. Trên tập hợp các số nguyên không âm, xét phương trình
x2
+ 2.3y
= x 2y+1
− 1

(1)
1 Tìm tất cả các cặp số nguyên không âm (x; y) thỏa mãn (1) mà y 6 5.
2 Chứng minh rằng không tồn tại cặp số nguyên không âm (x; y) với y  6 thỏa mãn phương
trình (1).
L Câu 4. Cho đường tròn (C1) và điểm B thuộc (C1). Điểm A khác B sao cho đường thẳng AB là
tiếp tuyến của (C1). Điểm C không thuộc (C1) sao cho đoạn thẳng AC cắt (C1) tại hai điểm phân
biệt. Gọi (C2) là đường tròn tiếp xúc với AC tại C và tiếp xúc với (C1) tại D(điểm B và D ở khác
phía so với bờ AC). Gọi I là tâm đường tròn ngoại tiếp tam giác BCD và ∆ là tiếp tuyến chung
của (C1), (C2) tại D.
1 Chứng minh rằng điểm I cách đều hai đường thẳng AB và ∆.
2 Chứng minh rằng tâm đường tròn ngoại tiếp tam giác BCD nằm trên đường tròn ngoại tiếp
tam giác ABC.
B Ngày thi thứ hai.
L Câu 5. Xét hệ phương trình









x2
= yz + at
y2
= zx + bt
z2
= xy + ct
x + y + z = a + b + c
với a, b, c, x, y, z, t ∈ R.
1 Giải hệ phương trình đã cho khi a = b = c = 1.
2 Giải hệ phương trình đã cho khi a, b, c là ba số thực được cho trước, khác không và đôi một
khác nhau.
L Câu 6. Trong mặt phẳng, cho đường tròn (O) cố định và hai điểm B, C phân biệt, cố định thuộc
đường tròn (O) sao cho BC không là đường kính của (O). Xét điểm A di động trên đường tròn (O)
sao cho tam giác ABC không là tam giác cân. Gọi H là trực tâm của tam giác ABC.
1 Chứng minh rằng vector
−
−
→
AH có hướng và độ dài không đổi.
2 Gọi d là tiếp tuyến của đường tròn (O) tại điểm A. Gọi ∆ là đường thẳng đi qua H và vuông
góc với d. Chứng minh rằng ∆ luôn luôn đi qua một điểm cố định.
L Câu 7. Cho n là số nguyên lớn hơn 1. Gọi Tn = {a1a2...an | ak ∈ {1; 2; 3}, 1 6 k 6 n}.
1 Tính trung bình cộng của tất cả các số thuộc Tn.
2 Hỏi có tất cả bao nhiêu số a = a1a2...an thuộc Tn mà |ak − ak+1| = 1 với mọi k thuộc tập hợp
{1; 2; 3; ...; (n − 1)} ?
® Chinh phục olympic toán 24 h Tạp chí và tư liệu toán học
Š Tuyển tập đề chọn đội tuyển các tỉnh trên cả nước VMO 2021
23 Đề thi tỉnh Quảng Ngãi.
L Câu 1. Tìm số hạng tổng quát của dãy số (un) biết



u1 =
1
2
; u2 =
2023
3
;
(n + 3) un+2 − 2(n + 2)2
un+1 + (n + 2) (n + 1)2
un = 0, n ∈ N, n  1
L Câu 2.
1 Cho n là một số nguyên dương có ít nhất 6 ước nguyên dương. Giả sử các ước nguyên dương
của n được sắp theo thứ tự sau 1 = d1  d2  ...  dk = n với k  6. Tìm tất cả số nguyên
dương n sao cho n = d2
5 + d2
6.
2 Cho p là số nguyên tố. Chứng minh rằng tồn tại các số nguyên x, y, z, n với 0  n  p thỏa
mãn x2
+ y2
+ z2
− np = 0.
L Câu 3. Cho tam giác ABC nội tiếp đường tròn (O). Một đường tròn (I) bất kì đi qua B, C và
lần lượt các cạnh CA, AB tại D, E. BD cắt CE tại F và G là hình chiếu vuông góc của I lên đường
thẳng AF.
1 Chứng minh rằng bốn điểm D, E, G, I cùng nằm trên một đường tròn và GA là tia phân giác
của góc 
DGE.
2 BD cắt GE tại H, CE cắt GD tại K. Đường thẳng DE cắt đường tròn (O) tại hai điểm M, N.
Chứng minh rằng đường tròn ngoại tiếp tam giác GHK tiếp xúc với đường tròn ngoại tiếp
tam giác GMN.
24 Đề thi tỉnh Tây Ninh.
A Ngày thi thứ nhất.
L Câu 1. Cho x, y, z là ba số thực dương x + y + z = 3. Tìm giá trị nhỏ nhất của từng biểu thức
sau
1 A =
x2
x + y
+
y2
y + z
+
z2
z + x
2 B =
1
x2 + 1
+
1
y2 + 1
+
1
z2 + 1
.
L Câu 2. Tìm tất cả các hàm số f : R → R thỏa mãn
f (yf (x + y) + f (x)) = 4x + 2y.f (x + y) , ∀x, y ∈ R
L Câu 3. Cho tam giác nhọn ABC không cân. Gọi H, O lần lượt là trực tâm, tâm đường tròn
ngoại tiếp tam giác ABC; D, E lần lượt là chân đường cao hạ từ các đỉnh A, B của tam giác ABC.
Các đường thằng OD và BE cắt nhau tại K. Các đường thẳng OE và AD cắt nhau tại L. Gọi M
là trung điểm cạnh AB. Chứng minh rằng ba điểm K, L, M thẳng hàng khi và chỉ khi bốn điểm
C, D, O, H cùng nằm trên một đường tròn.
L Câu 4. Bạn Dung và bạn Lan tham gia giải cờ vua nữ cấp trường (cùng một số bạn khác). Biết
rằng, trong số những bạn tham gia, 2 bạn bất kỳ sẽ đấu với nhau đúng 1 lần, người thắng sẽ được
1 điểm, hòa được 0,5 điểm và thua được 0 điểm. Sau khi giải đấu kết thúc, người ta thấy rằng: tổng
điểm của Dung và Lan là 4 điểm, trong khi tất cả các bạn còn lại đều có số điểm bằng nhau. Hơn
nữa, điểm của mỗi bạn Dung, Lan luôn ít hơn điểm của mỗi bạn khác.
® Chinh phục olympic toán 25 h Tạp chí và tư liệu toán học
Š Tuyển tập đề chọn đội tuyển các tỉnh trên cả nước VMO 2021
1 Hãy tính xêm có bao nhiêu bạn tham gia giải cờ vua nữ cấp trường này.
2 Nêu ra mô hình thi đấu cụ thể thỏa mãn đề bài (với đáp số tìm được trong câu 1/).
B Ngày thi thứ hai.
L Câu 5. Giải hệ phương trình trên tập số thực





2x = y 1 − x2

2y = z 1 − y2

2z = x 1 − z2

.
L Câu 6. Cho dãy số (un) thỏa mãn



u1 = 1
un+1 =
2u2
n + 5un + 2
u2
n + un + 1
; n  1, n ∈ N
Chứng minh dãy số (un) có giới hạn hữu hạn và tìm giới hạn đó.
L Câu 7. Tìm tất cả các số nguyên tố p, q sao cho pq | (2p
+ 2q
).
L Câu 8. Trong tam giác ABC, đường tròn nội tiếp tiếp xúc với các cạnh BC, CA, AB lần lượt
tại D, E, F. Một đường tròn (w) đi qua A và tiếp xúc với BC tại D cắt các đường thẳng BF, CE
tương ứng tại K và L. Đường thẳng đi qua E và song song với DL cắt đường thẳng đi qua F và
song song với DK tại P. Đặt R1, R2, R3, R4 lần lượt là độ dài bán kính đường tròn ngoại tiếp các
tam giác AFD, AED, FPD, EPD. Chứng minh rằng R1.R4 = R2.R3.
25 Đề thi tỉnh Quảng Nam.
L Câu 1. Giải hệ phương trình
(
2x2
− x − 3 + x
√
x2 + 1 = (y + 1)
p
y2 + 2y + 2
x2
+ y2
= x − 2y + 2
L Câu 2. Cho dãy (un) được xác định bởi
u1 = 1; un+1 =
p
u2
n + 2un + 3 −
p
u2
n − 2un + 3, ∀n ∈ N
Chứng minh rằng dãy số (un) có giới hạn hữu hạn và tìm giới hạn đó.
L Câu 3. Cho tam giác ABC nhọn, không cân AB  AC, nội tiếp đường tròn tâm O và ngoại
tiếp đường tròn tâm I. Đường thẳng qua I vuông góc với AI cắt đường thẳng BC tại S. Gọi M là
trung điểm BC, D là điểm chính giữa cung nhỏ BC của đường tròn (O) và J là điểm đối xứng với
I qua O.
1 Chứng minh tam giác DBI cân và tam giác DSJ vuông.
2 Gọi X là hình chiếu của điểm S trên đường thẳng OI và Y là giáo điểm thứ hai của MI và
đường tròn ngoại tiếp tam giác OMS. Chứng minh ba điểm A, X, Y thẳng hàng.
L Câu 4. Gọi α, β là các nghiệm của phương trình x2
−ax+a = 0, trong đó a là một số nguyên lớn
hơn 4. Chứng minh rằng với mọi số nguyên dương m, n thì [ma] 6= [nb] với [x] là ký hiệu số nguyên
lớn nhất không vượt quá x.
L Câu 5. Cho dãy đa thức hệ số thực {fn(x)} xác định bởi
f0 (x) = 2; f1 (x) = 2x; fn+2 (x) = 2x.fn+1 (x) + 1 − x2

fn (x) , ∀n ∈ N
Tìm tất cả các số tự nhiên n để fn(x) chia hết cho x2
+ 3.
® Chinh phục olympic toán 26 h Tạp chí và tư liệu toán học
Š Tuyển tập đề chọn đội tuyển các tỉnh trên cả nước VMO 2021
L Câu 6. Một số tự nhiên được gọi là số thân thiện nếu nó có thể phân tích được thành tích của
một số các số nguyên dương mà tổng của chúng bằng 2020. Hãy tìm số thân thiện lớn nhất.
L Câu 7. Cho các số thực a, b, c thỏa mãn điều kiện a2
+ b2
+ c2
= 3. Tìm giá trị lớn nhất và giá
trị nhỏ nhất của biểu thức
P = abc + 2 (ab + bc + ca) + 4 (a + b + c)
26 Đề thi tỉnh Quảng Trị.
L Câu 1.
1 Tìm tất cả các điểm cực đại và điểm cực tiểu của hàm số y = cos x − sin x.
2 Tìm m để phương trình |2x4
− 4x2
+ 1| − 2m = 0 có đúng 5 nghiệm phân biệt.
L Câu 2.
1 Chứng minh rằng

2020
1

+ 2

2020
2

+ ... + 1010

2020
1010

= 1010.22019
.
2 Tìm tất cả các cặp số thực (x; y) thỏa mãn |xy| 6 4 và (x − y)2
+ 20 = (x + y)(xy − 8).
L Câu 3.
1 Cho hình chóp S.ABC có đáy ABC là tam giác đều cạnh bằng a, tam giác SAB vuông cân tại
S và nằm trong mặt phẳng vuông góc với mặt phẳng đáy. Tính thể tích của khối chóp S.ABC
và khoảng cách giữa hai đường thẳng SB và AC theo a.
2 Cho tam giác ABC ngoại tiếp đường tròn (I). Gọi M, D, E lần lượt là trung điểm của
BC, IB, IC; F, G lần lượt là tâm đường tròn ngoại tiếp các tam giác ABD và ACE. Chứng
minh AM vuông góc FG.
L Câu 4. Cho dãy số (xn) được xác định bởi x1 =
√
2 và xn+1 =
√
2 − xn, ∀n  1. Chứng minh
dãy số (xn) có giới hạn hữu hạn và tìm giới hạn đó.
L Câu 5. Xét các số thực dương a, b, c có tổng bằng 3. Tìm giá trị nhỏ nhất của biểu thức
P =
2b + c
a
+
2c + a
b
+
2a + b
c
+
18abc
ab + bc + ca
27 Đề thi chọn đội tuyển THPT Chuyên Nguyễn Du.
A Ngày thi thứ nhất.
L Câu 1.
1 Giải phương trình x4
+ 2x3
+ 2x2
− 2x + 1 = (x3
+ x)
r
1 − x2
x
.
2 Giải hệ phương trình
(
xy + 2 = y
√
x2 + 2
y2
+ 2(x + 1)
√
x2 + 2x + 3 = 2x2
− 4x
.
L Câu 2. Cho các số thực dương x, y, z thỏa mãn x + y + z = 1. Chứng minh rằng
® Chinh phục olympic toán 27 h Tạp chí và tư liệu toán học
Š Tuyển tập đề chọn đội tuyển các tỉnh trên cả nước VMO 2021
1 + x
y + z
+
1 + y
z + x
+
1 + z
x + y
6 2

x
z
+
z
y
+
y
x

L Câu 3. Chứng minh rằng với mọi n ∈ Z+
, luôn tồn tại m ∈ N sao cho
√
2 − 1
n
=
√
m + 1 −
√
m
L Câu 4. Cho tứ giác lồi ABCD nội tiếp đường tròn (C). Gọi M, N, P lần lượt là giao điểm của
các cặp đường thẳng AB và CD, AD và BC, AC và BD. Gọi I1, I2, I3, I4 lần lượt là tâm đường tròn
bàng tiếp các tam giác ABN, BCM, CDN và ADM tương ứng với các đỉnh A, C, D và D.
1 Chứng minh các điểm I1, I2, I3, I4 đồng viên.
2 Gọi I là tâm đường tròn qua I1, I2, I3, I4. Chứng minh PI vuông góc với MN.
L Câu 5. Tìm tất cả các hàm số f : R → R thỏa mãn
f(x + f(y)) − f(f(x) − x) = f(y) − f(x) + 2x + 2y, ∀x, y ∈ R
B Ngày thi thứ hai.
L Câu 6. Cho (an) ; (bn) thỏa mãn









a1 = 2020; b1 =
1
2020
an+1 =
p
an + bn + 2
bn+1 =
p
2an + bn + 6
. Tính giới hạn (an) ; (bn) nếu có.
L Câu 7. Tìm các đa thức P(x), Q(x) ∈ R[x] khác đa thức không và có bậc bé nhất thỏa mãn
P (x2
) + Q(x) = P(x) + x5
Q(x) ∀x ∈ R
L Câu 8. Tìm tất cả n tự nhiên để A = 22
...2
|{z}
n số 2
−2 viết được thành a3
+ b3
+ c3
với a, b, c nguyên.
L Câu 9. Cho tam giác ABC(AC  AB). Lấy hai điểm M, N lần lượt trên AB và AC sao cho
MN song song với BC. Gọi P là giao điểm của hai đoạn thẳng BN và CM. Gọi A0
là điểm đối xứng
của A qua đường thẳng BC; (ω) là đường tròn ngoại tiếp tam giác AMN.
1 Gọi E là điểm thuộc đường tròn (ω) sao cho AE k MN. Chứng minh rằng E, P, A0
thẳng hàng.
2 Gọi F là giao điểm thứ hai của A0
P với đường tròn (ω) và I là tâm đường tròn ngoại tiếp tam
giác AA0
F. Chứng minh rằng IF tiếp xúc với đường tròn ngoại tiếp tam giác BFC.
L Câu 10. Cho tập hợp A = {1, 2, ..., 101}, tô màu ít nhất 50 phần tử của A sao cho nếu a, b ∈ A
(a, b không nhất thiết phân biệt) được tô màu và a + b ∈ A thì a + b cũng được tô màu. Gọi S là
tổng tất cả các số không được tô màu của A. Tìm giá trị lớn nhất của S.
28 Đề thi tỉnh Phú Yên.
L Câu 1. Giải phương trình x + 4
√
x + 3 + 2
√
3 − 2x = 11.
L Câu 2. Cho hệ phương trình





xyz + z = a
xyz2
+ z = b
x2
+ y2
+ z2
= 4
, (a, b ∈ R)
Tìm tất cả các giá trị của a, b để hệ phương trình có nghiệm duy nhất.
® Chinh phục olympic toán 28 h Tạp chí và tư liệu toán học
Š Tuyển tập đề chọn đội tuyển các tỉnh trên cả nước VMO 2021
L Câu 3.
1 Cho tam thức bậc hai f(x) = ax2
+ bx + c (a, b, c ∈ R; a 6= 0) có hai nghiệm x1, x2 thuộc đoạn
[0; 1]. Tìm giá trị lớn nhất của biểu thức A =
(a − b)(2a − b)
a(a − b + c)
.
2 Cho a, b, c là các số thực dương. Chứng minh rằng
a
b
+
b
c
+
c
a
+
9 3
√
abc
a + b + c
 6.
L Câu 4.
1 Cho điểm M tùy ý nằm bên trong tam giác ABC. Gọi S1, S2, S3 lần lượt là diện tích của các
tam giác MBC, MAC và MAB. Chứng minh rằng S1.
−
−
→
MA + S2
−
−
→
MB + S3
−
−
→
MC =
−
→
0 .
2 Trong mặt phẳng Oxy, cho parabol (P) : y = x2
+ px + q (q 6= 0). Biết rằng (P) cắt trục Ox
tại hai điểm phân biệt A, B và cắt trục Oy tại C. Chứng minh rằng khi p và q thay đổi, đường
tròn ngoại tiếp tam giác ABC luôn đi qua một điểm cố định.
L Câu 5. Cho dãy số (un) xác định bởi: u1 = 2; un+1 =
u2
n
2un − 1
,với n = 1, 2, 3, ...
1 Chứng minh rằng dãy số (un) giảm và bị chặn.
2 Hãy xác định số hạng tổng quát của dãy số (un).
L Câu 6. Tìm tất cả các hàm số f : R+
→ R thỏa mãn điều kiện
f

x + y
2020

=
f(x) + f(y)
2019
, ∀x, y ∈ R+
29 Đề chọn đội tuyển chuyên Đại học Vinh.
A Ngày thi thứ nhất.
L Câu 1. Cho dãy số (un) xác định bởi u1 = 2, un+1 = un +
n2
u2
n
, n = 1, 2, 3, ...
1 Chứng minh rằng lim
un
n
= 1.
2 Tính giới hạn lim (un − n).
L Câu 2. Cho các số thực dương x, y, z thỏa mãn x + y + z =
1
x
+
1
y
+
1
z
. Tìm giá trị nhỏ nhất của
P = 4
r
xy
z
+ 4
r
yz
x
+ 4
r
zx
y
L Câu 3. Cho đường tròn (O) cố định và AB là một dây cung cố định của đường tròn đó (AB
không đi qua O). Gọi I là trung điểm AB và M là một điểm di động trên cung lớn
_
AB (M khác A
và B). Lấy các điểm A0
, B0
lần lượt nằm trên các tia MA, MB sao cho 
MA0I = 
MB0I = 
AMB.
1 Gọi E là tâm của đường tròn (OAB). Chứng minh rằng ME vuông góc với A0
B0
.
2 Chứng minh rằng đường thẳng Euler của tam giác MA0
B0
luôn đi qua một điểm cố định khi
điểm M di động trên cung lớn
_
AB.
® Chinh phục olympic toán 29 h Tạp chí và tư liệu toán học
Š Tuyển tập đề chọn đội tuyển các tỉnh trên cả nước VMO 2021
L Câu 4. Cho bảng ô vuông kích thước 20 × 20, trong đó có n ô vuông được tô màu đen, các ô
vuông còn lại được tô màu trắng. Ta đặt lên bảng một tấm bìa hình O là hình vuông kích thước
3 × 3 khuyết ô vuông ở tâm sao cho các ô vuông của tấm bìa trùng khít lên các ô vuông của bảng.
1 Với n = 46, chứng minh với mọi cách tô n ô vuông màu đen ta luôn có thể đặt tấm bìa hình
O sao cho không có ô vuông đen nào của bảng bị che khuất.
2 Với n = 56, hỏi khẳng định ở ý 1) có còn đúng hay không?
B Ngày thi thứ hai.
L Câu 5. Cho hàm số f : (0; +∞) → (0; +∞) thỏa mãn f(x) + f(x + y) = f(f(x) + y) với mọi số
thực dương x, y.
1 Chứng minh rằng hàm số g(x) = f(x) − x là đơn ánh, nhận giá trị dương trên (0; +∞) và
f(x) + f(y) = 2f

x + y
2

với mọi số thực dương x, y.
2 Chứng minh rằng f là đơn ánh và tìm tất cả các hàm f thỏa mãn bài toán.
L Câu 6. Cho tam giác ABC không cân. Đường tròn (I; r) nội tiếp tam giác ABC tiếp xúc
BC, CA, AB lần lượt tại D, E, F. Gọi J là giao điểm của AD và EF. Lấy M, N trên (I; r) sao
cho M, J, N thẳng hàng và M nằm về phía nửa mặt phẳng chứa C bờ AD, N nằm về phía nửa mặt
phẳng chứa B bờ AD. Giả sử DM cắt AC tại P, DN cắt AB tại Q. Tiếp tuyến tại M, N của (I; r)
cắt nhau ở S. Gọi G là giao điểm của EF và BC và T là giao điểm của MN và AG.
1 Chứng minh P, Q, T thẳng hàng và S, A, G thẳng hàng.
2 Gọi K là giao điểm của SJ và BC, H là giao điểm của IK và TD.
Chứng minh rằng IH.IK = r2
.
L Câu 7. Chứng minh rằng n = 21 là số nguyên dương nhỏ nhất sao cho với mọi số nguyên dương
a, b, c thỏa mãn a|b4
, b|c4
, c|a4
ta đều có abc|(a + b + c)n
30 Đề thi tỉnh Ninh Thuận.
L Câu 1.
1 Cho số thực x ∈

0;
7
2

. Chứng minh rằng 15x +
28
x
 2x2
+ 36.
2 Cho ba số thực dương a, b, c thỏa mãn a2
+ b2
+ c2
= 12. Chứng minh rằng
15abc(a + b + c) + 28(ab + bc + ca)  132abc.
L Câu 2. Cho dãy số (un) được xác định bởi u0 = 0 và un+1 = −
u2
n
10
+ un +
1
2
, với mọi số tự nhiên
n. Chứng minh rằng dãy số (un) có giới hạn hữu hạn và tính giới hạn của nó?
L Câu 3. Một đa thức với hệ số nguyên được gọi là bất khả quy trên tập số nguyên nếu đa thức đó
không phân tích được thành tích hai đa thức với hệ số nguyên có bậc lớn hơn hoặc bằng 1. Chứng
minh rằng đa thức f(x) = x2021
+ 7x2020
+ 5 bất khả quy trên tập số nguyên.
® Chinh phục olympic toán 30 h Tạp chí và tư liệu toán học
Š Tuyển tập đề chọn đội tuyển các tỉnh trên cả nước VMO 2021
L Câu 4. Cho điểm A nằm ngoài đường tròn (O) tâm O. Dựng các tiếp tuyến AB và AC của (O),
trong đó B và C là các tiếp điểm. Gọi O0
là điểm cố định trên tia OA, vẽ đường tròn (O0
) có tâm là
O0
và đi qua hai điểm B, C. Một điểm M di động trên đường tròn (O) sao cho M không trùng với
B và C. Gọi D là giao điểm thứ hai của đường thẳng MA và đường tròn (O). Gọi E là giao điểm
thứ hai của đường thẳng MB và đường tròn (O0
). Gọi F là giao điểm thứ hai của đường thẳng BD
và đường tròn (O0
). Gọi N là giao điểm của MA và EF.
1 Chứng minh rằng N là trung điểm của đoạn EF.
2 Khi điểm M di động trên đường thẳng (O), chứng minh đường thẳng EF luôn đi qua một
điểm cố định.
L Câu 5. Bạn Hảo rất thích chơi với ánh sáng, cậu có một dãy gồm 10 bóng đèn xếp thứ tự từ 1
đến 10. Mỗi bóng đèn có một công tắc mà khi chạm vào nó, bóng đèn đang tắt sẽ chuyển sang trạng
thái sáng và ngược lại. Ban đầu tất cả các bóng đèn đều đang tắt. Trò chơi của bạn Hảo bao gồm
15 lần chạm vào công tắc, mỗi lần chỉ chạm một công tắc. Gọi N là số cách thực hiện 15 lần chạm
để được kết quả các bóng đèn 1,2,3,4,5 sáng và các bóng đèn 6,7,8,9,10 tắt. Gọi M là số cách thực
hiện 15 lần chạm sao cho trong các lần chạm này chỉ được chạm vào các công tắc của các bóng đèn
1,2,3,4,5 để được kết quả các bóng đèn 1,2,3,4,5 sáng và các bóng đèn 6,7,8,9,10 tắt. Hãy tính giá trị
của tỉ số
N
M
.
31 Đề thi tỉnh Đồng Nai.
L Câu 1. Cho dãy số (un) xác định bởi u1 = −2020 và un+1 = un +
1
2021n
với mọi n ∈ N∗
. Chứng
minh rằng tồn tại số nguyên dương n sao cho un  0.
L Câu 2. Tìm các số nguyên dương x và y thỏa mãn 7x
+ x4
+ 47 = y2
.
L Câu 3. Tìm các hàm số f : Z → Z thỏa mãn f(a2
f(a) + f(b)) = f(a)3
+ b, ∀a, b ∈ Z.
L Câu 4. Cho tam giác ABC cân tại A, lấy điểm D thuộc cạnh AB khác A và B, gọi (O) là đường
tròn ngoại tiếp tam giác BCD, tiếp tuyến của đường tròn (O) tại D cắt đường thẳng AC tại điểm
E, vẽ tiếp tuyến EFcủa đường tròn (O) tại tiếp điểm F khác D. Gọi I là giao điểm của hai đường
thẳng BF và CD, gọi K là giao điểm của hai đường thẳng AI và BC. Chứng minh BK = 2CK.
L Câu 5. Một tổ gồm có 5 học sinh được phân công trực nhật 6 ngày trong tuần từ thứ Hai đến
thử Bảy thỏa mãn các điều kiện sau: Mỗi ngày đều có từ 1 đến nhiều nhất là 2 học sinh trực và
trong cả tuần mỗi học sinh trực đúng 2 lần, mỗi lần trực 1 ngày. Tính số các cách phân công trực
nhật của tổ thỏa mãn các điều kiện đã cho.
32 Đề thi tỉnh Quảng Ngãi.
A Ngày thi thứ nhất.
L Câu 1. Giải hệ phương trình
(
2x +
√
2x + 1 +
p
2x + y + 2xy + 1 = 2y +
p
y + 1 + 1
3 3
p
3y + 1 + 1 = 4x2
y
L Câu 2. Cho dãy số (xn) xác định bởi



x1 = a(a  2)
xn+1 =
x2
n + 12xn
3x2
n + 4
, ∀n ∈ N∗
® Chinh phục olympic toán 31 h Tạp chí và tư liệu toán học
Š Tuyển tập đề chọn đội tuyển các tỉnh trên cả nước VMO 2021
1 Chứng minh rằng dãy số (xn) có giới hạn hữu hạn và tính giới hạn đó.
2 Đặt yn =
xn
2
+
xn−1
21
+ ... +
x1
2n
, ∀n ∈ N∗
. Tính lim yn?
L Câu 3. Cho tam giác ABC nhọn nội tiếp đường tròn ω tâm O. Các tiếp tuyến với ω tại B và C
cắt nhau tại T.
1 Gọi M là trung điểm của BC. Chứng minh [
BAT = 
CAM.
2 Đoạn thẳng OT cắt đường tròn ω tại X. Đoạn thẳng AX cắt đường tròn tâm bán kính TB
tại I. Gọi E, F lần lượt là hình chiếu vuông góc của I lên các cạnh AC, AB. Các đoạn thẳng
BE, CF cắt nhau tại Z. Chứng minh AZ, BC, OI đồng quy.
L Câu 4. Trong lễ khai mạc kỳ thi chọn đội tuyển thi HSGQG, để chuẩn bị cho việc thả bóng bay
lên trời thể hiện quyết tâm và khát vọng của học sinh, Ban tổ chức chọn 9 bạn đại diện cho 9 môn:
Toán, Tin, Lý, Hóa, Sinh, Văn, Anh, Sử, Địa xếp thành một hàng dọc. Sau đó, Ban tổ chức phát các
quả bóng bay gồm hai màu xanh và hồng cho 9 học sinh này (mỗi bạn một quả bóng) sao cho thỏa
mãn đồng thời hai điều kiện sau
1 Học sinh đứng cuối hàng nhận bóng bay màu xanh.
2 Nếu học sinh thứ i và thứ j nhận bóng bay khác màu thì học sinh thứ i + j nhận bóng bay
màu hồng (1 6 i  j 6 9, i + j 6 9).
Tìm tất cả các cách phát bóng bay có thể có của ban tổ chức.
B Ngày thi thứ hai.
L Câu 5. Một số nguyên dương n được gọi là “số tạo cấp số” nếu nó thỏa mãn đồng thời hai điều
kiện sau
1 n có ít nhất 4 ước nguyên dương d1, d2, .., dk(k  4) với d1  d2  ..  dk.
2 Các số d2 − d1, d3 − d2, .., dk − dk−1 theo thứ tự đó lập thành một cấp số nhân.
a) Tìm tất cả các số nguyên dương chẵn là số tạo cấp số.
b) Tìm tất cả các số nguyên dương là số tạo cấp số.
L Câu 6. Cho đa thức hệ số nguyên P(x) = anxn
+ an−1xn−1
+ ... + a1x + a0(an 6= 0).
1 P(x) được gọi là đa thức đặc biệt nếu các hệ số của nó thỏa mãn đồng thời hai điều kiện sau
(a) Nếu 0 6 i 6 n, 3 - i thì 3 | ai.
(b) 6 |
X
ai, trong đó
X
ai là tổng tất cả các hệ số ai mà 3 - i.
Chứng minh rằng với Q(x) là một đa thức hệ số nguyên bất kỳ thì [Q(x)]3
là một đa thức đặc
biệt.
2 P(x) được gọi là đa thức tổng lập phương nếu tồn tại m số nguyên b1, b2, ..., bm và m đa thức
hệ số nguyên P1(x), P2(x), ..., Pm(x) sao cho P(x) = b1[P1(x)]3
+ ... + bm[Pm(x)]3
với m ∈ N∗
.
Hỏi T(x) = 15x2
+ 9x + 2021 có phải là đa thức tổng lập phương không? Vì sao?
3 Chứng minh rằng mọi đa thức đặc biệt đều là đa thức tổng lập phương.
® Chinh phục olympic toán 32 h Tạp chí và tư liệu toán học
Š Tuyển tập đề chọn đội tuyển các tỉnh trên cả nước VMO 2021
L Câu 7. Cho đường tròn (O) và hai điểm B, C cố định trên (O), BC không là đường kính. Một
điểm A thay đổi trên (O) sao cho tam giác ABC nhọn, không cân. Đường tròn nội tiếp của tam giác
ABC tiếp xúc với các cạnh BC, CA, AB lần lượt tại D, E, F. Gọi I là tâm đường tròn nội tiếp của
tam giác ABC.
1 Chứng minh rằng các đường tròn ngoại tiếp tam giác AID, BIE, CIF có hai điểm chung.
2 Gọi G là giao điểm của CI và DE, H là điểm nằm trên đường thẳng EF sao cho IH vuông
góc với IC. Các đường thẳng GH và ID cắt nhau tại K, BK và CK lần lượt cắt đường thẳng
EF tại M và L. Gọi N là trung điểm của LM. Chứng minh rằng đường thẳng IN luôn đi qua
một điểm cố định.
33 Đề thi tỉnh Vĩnh Long.
A Ngày thi thứ nhất.
L Câu 1.
1 Giải phương trình 4
√
15 + x − 4
√
2 − x = 1, x ∈ R.
2 Giải hệ phương trình
(
x3
(−2 + 3y) = −8
x(y3
+ 2) = −6
(x, y ∈ R).
L Câu 2. Cho dãy số (un)(n = 1, 2, 3, ...) được xác định bởi



u1 = 0
un+1 =
un + 2018
−un + 2020
, (n = 1, 2, 3, ...)
Chứng minh dãy số (un)(n = 1, 2, 3, ...) có giới hạn hữu hạn và tính lim un.
L Câu 3.
1 Có bao nhiêu số tự nhiên gồm 4 chữ số thỏa mãn không có chữ số nào lặp lại đúng 3 lần ?
2 Cho tập hợp S = {1; 2; 3; 4; 5; 6; 7; 8; 9; 10}. Hỏi có bao nhiêu cách chia tập S thành ba tập
khác rỗng sao cho trong mỗi tập con đó không có hai số nguyên liên tiếp nào ?
L Câu 4. Cho tam giác ABC nhọn, các đường cao BK và CL cắt nhau tại H. Một đường thẳng
đi qua H cắt AB, AC lần lượt tại P, Q. Chứng minh rằng HP = HQ khi và chỉ khi MP = MQ, với
M là trung điểm của BC.
B Ngày thi thứ hai.
L Câu 5. Cho x, y, z là ba số thực dương thỏa mãn điều kiện x + y + z = 1. Chứng minh rằng
√
x + yz +
√
y + zx +
√
z + xy  1 +
√
xy +
√
yz +
√
zx.
L Câu 6.
1 Tìm tất cả các cặp số nguyên dương (x; y) với x, y nguyên tố cùng nhau và thỏa mãn phương
trình 2 (x3
− x) = y3
− y.
2 Cho 2020 tập hợp mà mỗi tập chứa đúng 45 phần tử. Biết rằng hai tập tùy ý trong các tập
này đều có đúng một phần tử chung. Chứng minh rằng tồn tại phần tử thuộc tất cả 2020 tập
họp đã cho.
® Chinh phục olympic toán 33 h Tạp chí và tư liệu toán học
Š Tuyển tập đề chọn đội tuyển các tỉnh trên cả nước VMO 2021
L Câu 7. Cho tam giác ABC nội tiếp trong một đường tròn và M là điểm chính giữa cung BC.
Gọi I, J, K theo thứ tự là chân các đường vuông góc kẻ từ M đến các đường thẳng AB, BC, CA; X
là giao điểm của BK và AJ; L là giao điểm của CX và IJ. Vẽ tia Jy vuông góc với MK cắt AL
tại T. Chứng minh CT vuông góc với IM.
L Câu 8. Tìm tất cả các hàm số f : R+
→ R+
thỏa mãn điều kiện f(x + y) + f(xy) = x + y + xy
với mọi x, y ∈ R+
.
34 Đề thi tỉnh Quảng Bình.
L Câu 1.
1 Cho dãy số thực (xn) xác định bởi



x1 = 1
xn+1 =
q
6 +
√
2xn + 3, ∀n ∈ N∗
.
Chứng minh rằng dãy
số (xn) có giới hạn hữu hạn. Tìm lim xn.
2 Cho dãy số thực (un) xác định bởi



u1 = 2, u2 = 1
un+2 =
r
6 +
1
2
p
3un+1 + 5un + 12, ∀n ∈ N∗
.
Tìm lim un.
L Câu 2. Trên các cạnh AB, AC của tam giác ABC lần lượt lấy hai điểm C1, B1. Hai đoạn thẳng
BB1 và CC1 cắt nhau tại X và hai đoạn thẳng B1C1 và AX cắt nhau tại P. Đường tròn ngoại tiếp
các tam giác BXC1, CXB1 cắt nhau tại điểm thứ hai Y và cắt cạnh BC lần lượt tại D và E.
1 Giả sử B1C1 k BC và gọi H, K lần lượt là hình chiếu vuông góc của Y lên AB và AC. Chứng
minh rằng
Y H
AB
=
Y K
AC
.
2 Giả sử B1E và C1D cắt nhau tại Q và đường thẳng B1D cắt đường thẳng C1E tại R. Chứng
minh ba điểm P, Q và R thẳng hàng.
L Câu 3. Cho tập hợp X có 2020 phẩn tử. Bạn An chia tập X thành 2 tập hợp A và B thỏa mãn
|A| = |B|; A ∩ B = ∅, bằng k cách khác nhau. Tìm giá trị nhỏ nhất của k sao cho với 2 phần tử
bất kỳ của X, luôn có ít nhất 1 cách trong k cách chia mà bạn An chia chúng vào 2 tập hợp khác
nhau.
L Câu 4. Gọi n là số nguyên dương thỏa mãn điều kiện 2n − 5 | 3(n! + 1).
1 Giả sử tồn tại n  4 thỏa mãn điều kiện trên. Chứng minh rằng 2n − 5 là số nguyên tố.
2 Tìm tất cả các số nguyên dương n thỏa mãn điều kiện trên.
35 Đề thi tỉnh Bình Dương
L Câu 1. Giải hệ phương trình sau :





x + 3y = x3
− 12
− y + 4z = y3
− 6
9z + 2x = z3
+ 32
.
L Câu 2. Với ba số thực a, b, c thỏa mãn các điều kiện abc 6= 0 và
1
|a|
+
1
|b|
+
1
|c|
6 3.
Hãy tìm giá trị nhỏ nhất của biểu thức
® Chinh phục olympic toán 34 h Tạp chí và tư liệu toán học
Š Tuyển tập đề chọn đội tuyển các tỉnh trên cả nước VMO 2021
P = [a2
+ 2 (b2
+ c2
)] [b2
+ 2 (c2
+ a2
)] [c2
+ 2 (a2
+ b2
)]
L Câu 3. Tìm tất cả các số nguyên dương n và k sao cho (n + 1)n
= 2nk
+ 3n + 1.
L Câu 4. Cho dãy số (un) thỏa mãn









u1 =
1
2
un+1 =
1
2
un +
r
u2
n +
1
4n
!
. Tính un theo n và tìm lim
n→∞
un.
L Câu 5.
1 Cho n ∈ N, n  2. Xét n số thực phân biệt x1, ..., xn. Đặt S = {xi + xj; 1 6 i 6= j 6 n}. Hỏi S
có ít nhất bao nhiêu phần tử ?
2 Trong mặt phẳng tọa độ, cho 2021 điểm phân biệt, mỗi điểm có hoành độ và tung độ đều là
các số nguyên không âm không vượt quá 807. Chứng minh rằng có thể tìm được 4 điểm tạo
thành 4 đỉnh của một hình thang cân (Hình chữ nhật được coi là một hình thang cân).
L Câu 6. Cho tam giác ABC có trực tâm H. Điểm P di chuyển trên cạnh BC. Lấy các điểm M, N
sao cho PM ⊥ AB, MB ⊥ BC, PN ⊥ AC, NC ⊥ BC.
1 Chứng minh rằng đường thẳng MN đi qua H.
2 Chứng minh rằng đường thẳng đi qua P và vuông góc với MN luôn đi qua một điểm cố định.
36 Đề thi tỉnh Yên Bái.
L Câu 1. Tìm tất cả các bộ (x, y, z) gồm ba số thực đôi một phân biệt thỏa mãn hệ phương trình





x2
+ y2
= −x + 2y + 2z
y2
+ z2
= x − y + 3z
z2
+ x2
= −x + y + 3z
L Câu 2. Cho (xn)∞
n=1 là dãy số giảm ngặt và có giới hạn bằng 0. Xét dãy số (yn)∞
n=1, với
yn = x1 − x2 + x3 − x4 + ... + (−1)n+1
xn, ∀n ∈ N∗
Chứng minh rằng dãy số (yn)∞
n=1 có giới hạn hữu hạn.
L Câu 3. Cho p là một số nguyên tố khác 3 và a, b, c, d là các số nguyên. Với mỗi số nguyên dương
n ta đặt Sn = an
+ bn
+ cn
+ dn
. Chứng minh rằng nếu S1 và S3 cùng chia hết cho p thì S2021 cũng
chia hết cho p.
L Câu 4. Cho tứ giác ABCD không là hình thang nội tiếp đường tròn (ω) tâm O. Gọi O1 và O2
là tâm đường tròn nội tiếp các tam giác ABC và ABD. Đường thẳng O1O2 cắt các đoạn thẳng BC
và AD lần lượt tại E và F. Đường thẳng qua E vuông góc với BC và đường thẳng qua F vuông góc
với AD cắt nhau tại J. Chứng minh rằng
1 Điểm J nằm trên trung trực của đoạn thẳng EF.
2 Đường tròn tâm J bán kính JE tiếp xúc với đường tròn (ω).
L Câu 5. Cho một đa giác đều có 2020 đỉnh. Mỗi đỉnh của đa giác được gán một trong các số
nguyên dương 1, 2, 3, ..., 1009. Chứng minh rằng đa giác đó có bốn đỉnh A, B, C, D thỏa mãn đồng
thời hai điều kiện dưới đây
1 Hai đường thẳng AB và CD song song với nhau.
2 Nếu bốn đỉnh A, B, C, D lần lượt được gán các số a, b, c, d thì a + b = c + d.
® Chinh phục olympic toán 35 h Tạp chí và tư liệu toán học

More Related Content

Similar to đề thi ko đáp.pdf

39 đề luyện thi học sinh giỏi toán 9
39 đề luyện thi học sinh giỏi toán 939 đề luyện thi học sinh giỏi toán 9
39 đề luyện thi học sinh giỏi toán 9Jackson Linh
 
Toan pt.de019.2010(+17de)
Toan pt.de019.2010(+17de)Toan pt.de019.2010(+17de)
Toan pt.de019.2010(+17de)BẢO Hí
 
TUYỂN TẬP ĐỀ THI BỒI DƯỠNG HỌC SINH GIỎI TOÁN LỚP 8 NĂM HỌC 2022-2023 (30 ĐỀ ...
TUYỂN TẬP ĐỀ THI BỒI DƯỠNG HỌC SINH GIỎI TOÁN LỚP 8 NĂM HỌC 2022-2023 (30 ĐỀ ...TUYỂN TẬP ĐỀ THI BỒI DƯỠNG HỌC SINH GIỎI TOÁN LỚP 8 NĂM HỌC 2022-2023 (30 ĐỀ ...
TUYỂN TẬP ĐỀ THI BỒI DƯỠNG HỌC SINH GIỎI TOÁN LỚP 8 NĂM HỌC 2022-2023 (30 ĐỀ ...Nguyen Thanh Tu Collection
 
30 dehsg toan6
30 dehsg toan630 dehsg toan6
30 dehsg toan6Tuan Viet
 
Xuctu.com de thi_tuyen_sinh_10_lhp_tdn_chuyen_dhsp(hcm)
Xuctu.com de thi_tuyen_sinh_10_lhp_tdn_chuyen_dhsp(hcm)Xuctu.com de thi_tuyen_sinh_10_lhp_tdn_chuyen_dhsp(hcm)
Xuctu.com de thi_tuyen_sinh_10_lhp_tdn_chuyen_dhsp(hcm)Minh Đức
 
Tuyen tap de thi vao lop 10
Tuyen tap de thi vao lop 10Tuyen tap de thi vao lop 10
Tuyen tap de thi vao lop 10Nguyen Van Tai
 
30 de-toan-thi-vao-l10-chuyen
30 de-toan-thi-vao-l10-chuyen30 de-toan-thi-vao-l10-chuyen
30 de-toan-thi-vao-l10-chuyenngatb1989
 
32 de thi lop 10 dh khtn ha noi 1989 2005
32 de thi lop 10 dh khtn ha noi 1989 200532 de thi lop 10 dh khtn ha noi 1989 2005
32 de thi lop 10 dh khtn ha noi 1989 2005vutoanpvd
 
32 de thi lop 10 dh khtn ha noi 1989 2005
32 de thi lop 10 dh khtn ha noi 1989 200532 de thi lop 10 dh khtn ha noi 1989 2005
32 de thi lop 10 dh khtn ha noi 1989 2005Tam Vu Minh
 
Olympic 1996 cac nuoc
Olympic 1996 cac nuocOlympic 1996 cac nuoc
Olympic 1996 cac nuocToán THCS
 
Tuyển chọn 50 đề thi học sinh giỏi toán 9
Tuyển chọn 50 đề thi học sinh giỏi toán 9Tuyển chọn 50 đề thi học sinh giỏi toán 9
Tuyển chọn 50 đề thi học sinh giỏi toán 9Nhập Vân Long
 
50 de thi hsg toan 9
50 de thi hsg toan 950 de thi hsg toan 9
50 de thi hsg toan 9thanhgand
 
50dethihsgtoan9 140928111901-phpapp01
50dethihsgtoan9 140928111901-phpapp0150dethihsgtoan9 140928111901-phpapp01
50dethihsgtoan9 140928111901-phpapp01Minh Đức
 
De tsl10 toan hai duong chuyen 13-14
De tsl10 toan hai duong chuyen  13-14De tsl10 toan hai duong chuyen  13-14
De tsl10 toan hai duong chuyen 13-14Toan Isi
 
Tuyển tập đề thi vào lớp 10 môn Toán THPT - Thầy Thích
Tuyển tập đề thi vào lớp 10 môn Toán THPT - Thầy ThíchTuyển tập đề thi vào lớp 10 môn Toán THPT - Thầy Thích
Tuyển tập đề thi vào lớp 10 môn Toán THPT - Thầy ThíchBồi dưỡng Toán lớp 6
 

Similar to đề thi ko đáp.pdf (20)

39 đề luyện thi học sinh giỏi toán 9
39 đề luyện thi học sinh giỏi toán 939 đề luyện thi học sinh giỏi toán 9
39 đề luyện thi học sinh giỏi toán 9
 
Toan pt.de019.2010(+17de)
Toan pt.de019.2010(+17de)Toan pt.de019.2010(+17de)
Toan pt.de019.2010(+17de)
 
TUYỂN TẬP ĐỀ THI BỒI DƯỠNG HỌC SINH GIỎI TOÁN LỚP 8 NĂM HỌC 2022-2023 (30 ĐỀ ...
TUYỂN TẬP ĐỀ THI BỒI DƯỠNG HỌC SINH GIỎI TOÁN LỚP 8 NĂM HỌC 2022-2023 (30 ĐỀ ...TUYỂN TẬP ĐỀ THI BỒI DƯỠNG HỌC SINH GIỎI TOÁN LỚP 8 NĂM HỌC 2022-2023 (30 ĐỀ ...
TUYỂN TẬP ĐỀ THI BỒI DƯỠNG HỌC SINH GIỎI TOÁN LỚP 8 NĂM HỌC 2022-2023 (30 ĐỀ ...
 
30 dehsg toan6
30 dehsg toan630 dehsg toan6
30 dehsg toan6
 
30 dehsg toan6
30 dehsg toan630 dehsg toan6
30 dehsg toan6
 
30 dehsg toan6
30 dehsg toan630 dehsg toan6
30 dehsg toan6
 
Xuctu.com de thi_tuyen_sinh_10_lhp_tdn_chuyen_dhsp(hcm)
Xuctu.com de thi_tuyen_sinh_10_lhp_tdn_chuyen_dhsp(hcm)Xuctu.com de thi_tuyen_sinh_10_lhp_tdn_chuyen_dhsp(hcm)
Xuctu.com de thi_tuyen_sinh_10_lhp_tdn_chuyen_dhsp(hcm)
 
Tuyen tap de thi vao lop 10
Tuyen tap de thi vao lop 10Tuyen tap de thi vao lop 10
Tuyen tap de thi vao lop 10
 
Dgnl dhqg ha noi 2022 de so 1
Dgnl dhqg ha noi 2022 de so 1Dgnl dhqg ha noi 2022 de so 1
Dgnl dhqg ha noi 2022 de so 1
 
30 de-toan-thi-vao-l10-chuyen
30 de-toan-thi-vao-l10-chuyen30 de-toan-thi-vao-l10-chuyen
30 de-toan-thi-vao-l10-chuyen
 
32 de thi lop 10 dh khtn ha noi 1989 2005
32 de thi lop 10 dh khtn ha noi 1989 200532 de thi lop 10 dh khtn ha noi 1989 2005
32 de thi lop 10 dh khtn ha noi 1989 2005
 
C2-14
C2-14C2-14
C2-14
 
32 de thi lop 10 dh khtn ha noi 1989 2005
32 de thi lop 10 dh khtn ha noi 1989 200532 de thi lop 10 dh khtn ha noi 1989 2005
32 de thi lop 10 dh khtn ha noi 1989 2005
 
Olympic 1996 cac nuoc
Olympic 1996 cac nuocOlympic 1996 cac nuoc
Olympic 1996 cac nuoc
 
Tuyển chọn 50 đề thi học sinh giỏi toán 9
Tuyển chọn 50 đề thi học sinh giỏi toán 9Tuyển chọn 50 đề thi học sinh giỏi toán 9
Tuyển chọn 50 đề thi học sinh giỏi toán 9
 
50 de thi hsg toan 9
50 de thi hsg toan 950 de thi hsg toan 9
50 de thi hsg toan 9
 
50dethihsgtoan9 140928111901-phpapp01
50dethihsgtoan9 140928111901-phpapp0150dethihsgtoan9 140928111901-phpapp01
50dethihsgtoan9 140928111901-phpapp01
 
De tsl10 toan hai duong chuyen 13-14
De tsl10 toan hai duong chuyen  13-14De tsl10 toan hai duong chuyen  13-14
De tsl10 toan hai duong chuyen 13-14
 
K10+11+12
K10+11+12K10+11+12
K10+11+12
 
Tuyển tập đề thi vào lớp 10 môn Toán THPT - Thầy Thích
Tuyển tập đề thi vào lớp 10 môn Toán THPT - Thầy ThíchTuyển tập đề thi vào lớp 10 môn Toán THPT - Thầy Thích
Tuyển tập đề thi vào lớp 10 môn Toán THPT - Thầy Thích
 

More from NguyenTanBinh4

Proofs Without Words by Roger B Nelsen.pdf
Proofs Without Words  by Roger B Nelsen.pdfProofs Without Words  by Roger B Nelsen.pdf
Proofs Without Words by Roger B Nelsen.pdfNguyenTanBinh4
 
ĐẦY ĐỦ NHẤT VỀ TỔ HỢP XÁC SUẤT.pdf
ĐẦY ĐỦ NHẤT VỀ TỔ HỢP XÁC SUẤT.pdfĐẦY ĐỦ NHẤT VỀ TỔ HỢP XÁC SUẤT.pdf
ĐẦY ĐỦ NHẤT VỀ TỔ HỢP XÁC SUẤT.pdfNguyenTanBinh4
 
phương trình hàm.pdf
phương trình hàm.pdfphương trình hàm.pdf
phương trình hàm.pdfNguyenTanBinh4
 
Techniques_of_Variational_Analysis.pdf
Techniques_of_Variational_Analysis.pdfTechniques_of_Variational_Analysis.pdf
Techniques_of_Variational_Analysis.pdfNguyenTanBinh4
 
SH_Lien_ND_Dinh ly thang du Trung Hoa_VP_2016_08_16.pdf
SH_Lien_ND_Dinh ly thang du Trung Hoa_VP_2016_08_16.pdfSH_Lien_ND_Dinh ly thang du Trung Hoa_VP_2016_08_16.pdf
SH_Lien_ND_Dinh ly thang du Trung Hoa_VP_2016_08_16.pdfNguyenTanBinh4
 
CÁC BÀI TOÁN XÁC SUẤT HAY VÀ KHÓ.pdf
CÁC BÀI TOÁN XÁC SUẤT HAY VÀ KHÓ.pdfCÁC BÀI TOÁN XÁC SUẤT HAY VÀ KHÓ.pdf
CÁC BÀI TOÁN XÁC SUẤT HAY VÀ KHÓ.pdfNguyenTanBinh4
 

More from NguyenTanBinh4 (7)

Proofs Without Words by Roger B Nelsen.pdf
Proofs Without Words  by Roger B Nelsen.pdfProofs Without Words  by Roger B Nelsen.pdf
Proofs Without Words by Roger B Nelsen.pdf
 
ĐẦY ĐỦ NHẤT VỀ TỔ HỢP XÁC SUẤT.pdf
ĐẦY ĐỦ NHẤT VỀ TỔ HỢP XÁC SUẤT.pdfĐẦY ĐỦ NHẤT VỀ TỔ HỢP XÁC SUẤT.pdf
ĐẦY ĐỦ NHẤT VỀ TỔ HỢP XÁC SUẤT.pdf
 
phương trình hàm.pdf
phương trình hàm.pdfphương trình hàm.pdf
phương trình hàm.pdf
 
Techniques_of_Variational_Analysis.pdf
Techniques_of_Variational_Analysis.pdfTechniques_of_Variational_Analysis.pdf
Techniques_of_Variational_Analysis.pdf
 
PhuongTrinhHam.pdf
PhuongTrinhHam.pdfPhuongTrinhHam.pdf
PhuongTrinhHam.pdf
 
SH_Lien_ND_Dinh ly thang du Trung Hoa_VP_2016_08_16.pdf
SH_Lien_ND_Dinh ly thang du Trung Hoa_VP_2016_08_16.pdfSH_Lien_ND_Dinh ly thang du Trung Hoa_VP_2016_08_16.pdf
SH_Lien_ND_Dinh ly thang du Trung Hoa_VP_2016_08_16.pdf
 
CÁC BÀI TOÁN XÁC SUẤT HAY VÀ KHÓ.pdf
CÁC BÀI TOÁN XÁC SUẤT HAY VÀ KHÓ.pdfCÁC BÀI TOÁN XÁC SUẤT HAY VÀ KHÓ.pdf
CÁC BÀI TOÁN XÁC SUẤT HAY VÀ KHÓ.pdf
 

Recently uploaded

Giáo trình nhập môn lập trình - Đặng Bình Phương
Giáo trình nhập môn lập trình - Đặng Bình PhươngGiáo trình nhập môn lập trình - Đặng Bình Phương
Giáo trình nhập môn lập trình - Đặng Bình Phươnghazzthuan
 
Access: Chuong III Thiet ke truy van Query.ppt
Access: Chuong III Thiet ke truy van Query.pptAccess: Chuong III Thiet ke truy van Query.ppt
Access: Chuong III Thiet ke truy van Query.pptPhamThiThuThuy1
 
Bài giảng môn Truyền thông đa phương tiện
Bài giảng môn Truyền thông đa phương tiệnBài giảng môn Truyền thông đa phương tiện
Bài giảng môn Truyền thông đa phương tiệnpmtiendhti14a5hn
 
TUYỂN TẬP ĐỀ THI GIỮA KÌ, CUỐI KÌ 2 MÔN VẬT LÍ LỚP 11 THEO HÌNH THỨC THI MỚI ...
TUYỂN TẬP ĐỀ THI GIỮA KÌ, CUỐI KÌ 2 MÔN VẬT LÍ LỚP 11 THEO HÌNH THỨC THI MỚI ...TUYỂN TẬP ĐỀ THI GIỮA KÌ, CUỐI KÌ 2 MÔN VẬT LÍ LỚP 11 THEO HÌNH THỨC THI MỚI ...
TUYỂN TẬP ĐỀ THI GIỮA KÌ, CUỐI KÌ 2 MÔN VẬT LÍ LỚP 11 THEO HÌNH THỨC THI MỚI ...Nguyen Thanh Tu Collection
 
Trắc nghiệm CHƯƠNG 5 môn Chủ nghĩa xã hội
Trắc nghiệm CHƯƠNG 5 môn Chủ nghĩa xã hộiTrắc nghiệm CHƯƠNG 5 môn Chủ nghĩa xã hội
Trắc nghiệm CHƯƠNG 5 môn Chủ nghĩa xã hộiNgocNguyen591215
 
bài thi bảo vệ nền tảng tư tưởng của Đảng.docx
bài thi bảo vệ nền tảng tư tưởng của Đảng.docxbài thi bảo vệ nền tảng tư tưởng của Đảng.docx
bài thi bảo vệ nền tảng tư tưởng của Đảng.docxTrnHiYn5
 
C6. Van de dan toc va ton giao ....pdf . Chu nghia xa hoi
C6. Van de dan toc va ton giao ....pdf . Chu nghia xa hoiC6. Van de dan toc va ton giao ....pdf . Chu nghia xa hoi
C6. Van de dan toc va ton giao ....pdf . Chu nghia xa hoidnghia2002
 
SD-05_Xây dựng website bán váy Lolita Alice - Phùng Thị Thúy Hiền PH 2 7 8 6 ...
SD-05_Xây dựng website bán váy Lolita Alice - Phùng Thị Thúy Hiền PH 2 7 8 6 ...SD-05_Xây dựng website bán váy Lolita Alice - Phùng Thị Thúy Hiền PH 2 7 8 6 ...
SD-05_Xây dựng website bán váy Lolita Alice - Phùng Thị Thúy Hiền PH 2 7 8 6 ...ChuThNgnFEFPLHN
 
30 ĐỀ PHÁT TRIỂN THEO CẤU TRÚC ĐỀ MINH HỌA BGD NGÀY 22-3-2024 KỲ THI TỐT NGHI...
30 ĐỀ PHÁT TRIỂN THEO CẤU TRÚC ĐỀ MINH HỌA BGD NGÀY 22-3-2024 KỲ THI TỐT NGHI...30 ĐỀ PHÁT TRIỂN THEO CẤU TRÚC ĐỀ MINH HỌA BGD NGÀY 22-3-2024 KỲ THI TỐT NGHI...
30 ĐỀ PHÁT TRIỂN THEO CẤU TRÚC ĐỀ MINH HỌA BGD NGÀY 22-3-2024 KỲ THI TỐT NGHI...Nguyen Thanh Tu Collection
 
ĐỀ CHÍNH THỨC KỲ THI TUYỂN SINH VÀO LỚP 10 THPT CÁC TỈNH THÀNH NĂM HỌC 2020 –...
ĐỀ CHÍNH THỨC KỲ THI TUYỂN SINH VÀO LỚP 10 THPT CÁC TỈNH THÀNH NĂM HỌC 2020 –...ĐỀ CHÍNH THỨC KỲ THI TUYỂN SINH VÀO LỚP 10 THPT CÁC TỈNH THÀNH NĂM HỌC 2020 –...
ĐỀ CHÍNH THỨC KỲ THI TUYỂN SINH VÀO LỚP 10 THPT CÁC TỈNH THÀNH NĂM HỌC 2020 –...Nguyen Thanh Tu Collection
 
xemsomenh.com-Vòng Thái Tuế và Ý Nghĩa Các Sao Tại Cung Mệnh.pdf
xemsomenh.com-Vòng Thái Tuế và Ý Nghĩa Các Sao Tại Cung Mệnh.pdfxemsomenh.com-Vòng Thái Tuế và Ý Nghĩa Các Sao Tại Cung Mệnh.pdf
xemsomenh.com-Vòng Thái Tuế và Ý Nghĩa Các Sao Tại Cung Mệnh.pdfXem Số Mệnh
 
20 ĐỀ DỰ ĐOÁN - PHÁT TRIỂN ĐỀ MINH HỌA BGD KỲ THI TỐT NGHIỆP THPT NĂM 2024 MÔ...
20 ĐỀ DỰ ĐOÁN - PHÁT TRIỂN ĐỀ MINH HỌA BGD KỲ THI TỐT NGHIỆP THPT NĂM 2024 MÔ...20 ĐỀ DỰ ĐOÁN - PHÁT TRIỂN ĐỀ MINH HỌA BGD KỲ THI TỐT NGHIỆP THPT NĂM 2024 MÔ...
20 ĐỀ DỰ ĐOÁN - PHÁT TRIỂN ĐỀ MINH HỌA BGD KỲ THI TỐT NGHIỆP THPT NĂM 2024 MÔ...Nguyen Thanh Tu Collection
 
Kiến thức cơ bản về tư duy số - VTC Net Viet
Kiến thức cơ bản về tư duy số - VTC Net VietKiến thức cơ bản về tư duy số - VTC Net Viet
Kiến thức cơ bản về tư duy số - VTC Net VietNguyễn Quang Huy
 
Đề thi tin học HK2 lớp 3 Chân Trời Sáng Tạo
Đề thi tin học HK2 lớp 3 Chân Trời Sáng TạoĐề thi tin học HK2 lớp 3 Chân Trời Sáng Tạo
Đề thi tin học HK2 lớp 3 Chân Trời Sáng Tạowindcances
 
60 CÂU HỎI ÔN TẬP LÝ LUẬN CHÍNH TRỊ NĂM 2024.docx
60 CÂU HỎI ÔN TẬP LÝ LUẬN CHÍNH TRỊ NĂM 2024.docx60 CÂU HỎI ÔN TẬP LÝ LUẬN CHÍNH TRỊ NĂM 2024.docx
60 CÂU HỎI ÔN TẬP LÝ LUẬN CHÍNH TRỊ NĂM 2024.docxasdnguyendinhdang
 
30 ĐỀ PHÁT TRIỂN THEO CẤU TRÚC ĐỀ MINH HỌA BGD NGÀY 22-3-2024 KỲ THI TỐT NGHI...
30 ĐỀ PHÁT TRIỂN THEO CẤU TRÚC ĐỀ MINH HỌA BGD NGÀY 22-3-2024 KỲ THI TỐT NGHI...30 ĐỀ PHÁT TRIỂN THEO CẤU TRÚC ĐỀ MINH HỌA BGD NGÀY 22-3-2024 KỲ THI TỐT NGHI...
30 ĐỀ PHÁT TRIỂN THEO CẤU TRÚC ĐỀ MINH HỌA BGD NGÀY 22-3-2024 KỲ THI TỐT NGHI...Nguyen Thanh Tu Collection
 
Danh sách sinh viên tốt nghiệp Đại học - Cao đẳng Trường Đại học Phú Yên năm ...
Danh sách sinh viên tốt nghiệp Đại học - Cao đẳng Trường Đại học Phú Yên năm ...Danh sách sinh viên tốt nghiệp Đại học - Cao đẳng Trường Đại học Phú Yên năm ...
Danh sách sinh viên tốt nghiệp Đại học - Cao đẳng Trường Đại học Phú Yên năm ...hoangtuansinh1
 
xemsomenh.com-Vòng Tràng Sinh - Cách An 12 Sao Và Ý Nghĩa Từng Sao.pdf
xemsomenh.com-Vòng Tràng Sinh - Cách An 12 Sao Và Ý Nghĩa Từng Sao.pdfxemsomenh.com-Vòng Tràng Sinh - Cách An 12 Sao Và Ý Nghĩa Từng Sao.pdf
xemsomenh.com-Vòng Tràng Sinh - Cách An 12 Sao Và Ý Nghĩa Từng Sao.pdfXem Số Mệnh
 
Bài tập nhóm Kỹ Năng Gỉai Quyết Tranh Chấp Lao Động (1).pptx
Bài tập nhóm Kỹ Năng Gỉai Quyết Tranh Chấp Lao Động (1).pptxBài tập nhóm Kỹ Năng Gỉai Quyết Tranh Chấp Lao Động (1).pptx
Bài tập nhóm Kỹ Năng Gỉai Quyết Tranh Chấp Lao Động (1).pptxDungxPeach
 
TÀI LIỆU BỒI DƯỠNG HỌC SINH GIỎI KỸ NĂNG VIẾT ĐOẠN VĂN NGHỊ LUẬN XÃ HỘI 200 C...
TÀI LIỆU BỒI DƯỠNG HỌC SINH GIỎI KỸ NĂNG VIẾT ĐOẠN VĂN NGHỊ LUẬN XÃ HỘI 200 C...TÀI LIỆU BỒI DƯỠNG HỌC SINH GIỎI KỸ NĂNG VIẾT ĐOẠN VĂN NGHỊ LUẬN XÃ HỘI 200 C...
TÀI LIỆU BỒI DƯỠNG HỌC SINH GIỎI KỸ NĂNG VIẾT ĐOẠN VĂN NGHỊ LUẬN XÃ HỘI 200 C...Nguyen Thanh Tu Collection
 

Recently uploaded (20)

Giáo trình nhập môn lập trình - Đặng Bình Phương
Giáo trình nhập môn lập trình - Đặng Bình PhươngGiáo trình nhập môn lập trình - Đặng Bình Phương
Giáo trình nhập môn lập trình - Đặng Bình Phương
 
Access: Chuong III Thiet ke truy van Query.ppt
Access: Chuong III Thiet ke truy van Query.pptAccess: Chuong III Thiet ke truy van Query.ppt
Access: Chuong III Thiet ke truy van Query.ppt
 
Bài giảng môn Truyền thông đa phương tiện
Bài giảng môn Truyền thông đa phương tiệnBài giảng môn Truyền thông đa phương tiện
Bài giảng môn Truyền thông đa phương tiện
 
TUYỂN TẬP ĐỀ THI GIỮA KÌ, CUỐI KÌ 2 MÔN VẬT LÍ LỚP 11 THEO HÌNH THỨC THI MỚI ...
TUYỂN TẬP ĐỀ THI GIỮA KÌ, CUỐI KÌ 2 MÔN VẬT LÍ LỚP 11 THEO HÌNH THỨC THI MỚI ...TUYỂN TẬP ĐỀ THI GIỮA KÌ, CUỐI KÌ 2 MÔN VẬT LÍ LỚP 11 THEO HÌNH THỨC THI MỚI ...
TUYỂN TẬP ĐỀ THI GIỮA KÌ, CUỐI KÌ 2 MÔN VẬT LÍ LỚP 11 THEO HÌNH THỨC THI MỚI ...
 
Trắc nghiệm CHƯƠNG 5 môn Chủ nghĩa xã hội
Trắc nghiệm CHƯƠNG 5 môn Chủ nghĩa xã hộiTrắc nghiệm CHƯƠNG 5 môn Chủ nghĩa xã hội
Trắc nghiệm CHƯƠNG 5 môn Chủ nghĩa xã hội
 
bài thi bảo vệ nền tảng tư tưởng của Đảng.docx
bài thi bảo vệ nền tảng tư tưởng của Đảng.docxbài thi bảo vệ nền tảng tư tưởng của Đảng.docx
bài thi bảo vệ nền tảng tư tưởng của Đảng.docx
 
C6. Van de dan toc va ton giao ....pdf . Chu nghia xa hoi
C6. Van de dan toc va ton giao ....pdf . Chu nghia xa hoiC6. Van de dan toc va ton giao ....pdf . Chu nghia xa hoi
C6. Van de dan toc va ton giao ....pdf . Chu nghia xa hoi
 
SD-05_Xây dựng website bán váy Lolita Alice - Phùng Thị Thúy Hiền PH 2 7 8 6 ...
SD-05_Xây dựng website bán váy Lolita Alice - Phùng Thị Thúy Hiền PH 2 7 8 6 ...SD-05_Xây dựng website bán váy Lolita Alice - Phùng Thị Thúy Hiền PH 2 7 8 6 ...
SD-05_Xây dựng website bán váy Lolita Alice - Phùng Thị Thúy Hiền PH 2 7 8 6 ...
 
30 ĐỀ PHÁT TRIỂN THEO CẤU TRÚC ĐỀ MINH HỌA BGD NGÀY 22-3-2024 KỲ THI TỐT NGHI...
30 ĐỀ PHÁT TRIỂN THEO CẤU TRÚC ĐỀ MINH HỌA BGD NGÀY 22-3-2024 KỲ THI TỐT NGHI...30 ĐỀ PHÁT TRIỂN THEO CẤU TRÚC ĐỀ MINH HỌA BGD NGÀY 22-3-2024 KỲ THI TỐT NGHI...
30 ĐỀ PHÁT TRIỂN THEO CẤU TRÚC ĐỀ MINH HỌA BGD NGÀY 22-3-2024 KỲ THI TỐT NGHI...
 
ĐỀ CHÍNH THỨC KỲ THI TUYỂN SINH VÀO LỚP 10 THPT CÁC TỈNH THÀNH NĂM HỌC 2020 –...
ĐỀ CHÍNH THỨC KỲ THI TUYỂN SINH VÀO LỚP 10 THPT CÁC TỈNH THÀNH NĂM HỌC 2020 –...ĐỀ CHÍNH THỨC KỲ THI TUYỂN SINH VÀO LỚP 10 THPT CÁC TỈNH THÀNH NĂM HỌC 2020 –...
ĐỀ CHÍNH THỨC KỲ THI TUYỂN SINH VÀO LỚP 10 THPT CÁC TỈNH THÀNH NĂM HỌC 2020 –...
 
xemsomenh.com-Vòng Thái Tuế và Ý Nghĩa Các Sao Tại Cung Mệnh.pdf
xemsomenh.com-Vòng Thái Tuế và Ý Nghĩa Các Sao Tại Cung Mệnh.pdfxemsomenh.com-Vòng Thái Tuế và Ý Nghĩa Các Sao Tại Cung Mệnh.pdf
xemsomenh.com-Vòng Thái Tuế và Ý Nghĩa Các Sao Tại Cung Mệnh.pdf
 
20 ĐỀ DỰ ĐOÁN - PHÁT TRIỂN ĐỀ MINH HỌA BGD KỲ THI TỐT NGHIỆP THPT NĂM 2024 MÔ...
20 ĐỀ DỰ ĐOÁN - PHÁT TRIỂN ĐỀ MINH HỌA BGD KỲ THI TỐT NGHIỆP THPT NĂM 2024 MÔ...20 ĐỀ DỰ ĐOÁN - PHÁT TRIỂN ĐỀ MINH HỌA BGD KỲ THI TỐT NGHIỆP THPT NĂM 2024 MÔ...
20 ĐỀ DỰ ĐOÁN - PHÁT TRIỂN ĐỀ MINH HỌA BGD KỲ THI TỐT NGHIỆP THPT NĂM 2024 MÔ...
 
Kiến thức cơ bản về tư duy số - VTC Net Viet
Kiến thức cơ bản về tư duy số - VTC Net VietKiến thức cơ bản về tư duy số - VTC Net Viet
Kiến thức cơ bản về tư duy số - VTC Net Viet
 
Đề thi tin học HK2 lớp 3 Chân Trời Sáng Tạo
Đề thi tin học HK2 lớp 3 Chân Trời Sáng TạoĐề thi tin học HK2 lớp 3 Chân Trời Sáng Tạo
Đề thi tin học HK2 lớp 3 Chân Trời Sáng Tạo
 
60 CÂU HỎI ÔN TẬP LÝ LUẬN CHÍNH TRỊ NĂM 2024.docx
60 CÂU HỎI ÔN TẬP LÝ LUẬN CHÍNH TRỊ NĂM 2024.docx60 CÂU HỎI ÔN TẬP LÝ LUẬN CHÍNH TRỊ NĂM 2024.docx
60 CÂU HỎI ÔN TẬP LÝ LUẬN CHÍNH TRỊ NĂM 2024.docx
 
30 ĐỀ PHÁT TRIỂN THEO CẤU TRÚC ĐỀ MINH HỌA BGD NGÀY 22-3-2024 KỲ THI TỐT NGHI...
30 ĐỀ PHÁT TRIỂN THEO CẤU TRÚC ĐỀ MINH HỌA BGD NGÀY 22-3-2024 KỲ THI TỐT NGHI...30 ĐỀ PHÁT TRIỂN THEO CẤU TRÚC ĐỀ MINH HỌA BGD NGÀY 22-3-2024 KỲ THI TỐT NGHI...
30 ĐỀ PHÁT TRIỂN THEO CẤU TRÚC ĐỀ MINH HỌA BGD NGÀY 22-3-2024 KỲ THI TỐT NGHI...
 
Danh sách sinh viên tốt nghiệp Đại học - Cao đẳng Trường Đại học Phú Yên năm ...
Danh sách sinh viên tốt nghiệp Đại học - Cao đẳng Trường Đại học Phú Yên năm ...Danh sách sinh viên tốt nghiệp Đại học - Cao đẳng Trường Đại học Phú Yên năm ...
Danh sách sinh viên tốt nghiệp Đại học - Cao đẳng Trường Đại học Phú Yên năm ...
 
xemsomenh.com-Vòng Tràng Sinh - Cách An 12 Sao Và Ý Nghĩa Từng Sao.pdf
xemsomenh.com-Vòng Tràng Sinh - Cách An 12 Sao Và Ý Nghĩa Từng Sao.pdfxemsomenh.com-Vòng Tràng Sinh - Cách An 12 Sao Và Ý Nghĩa Từng Sao.pdf
xemsomenh.com-Vòng Tràng Sinh - Cách An 12 Sao Và Ý Nghĩa Từng Sao.pdf
 
Bài tập nhóm Kỹ Năng Gỉai Quyết Tranh Chấp Lao Động (1).pptx
Bài tập nhóm Kỹ Năng Gỉai Quyết Tranh Chấp Lao Động (1).pptxBài tập nhóm Kỹ Năng Gỉai Quyết Tranh Chấp Lao Động (1).pptx
Bài tập nhóm Kỹ Năng Gỉai Quyết Tranh Chấp Lao Động (1).pptx
 
TÀI LIỆU BỒI DƯỠNG HỌC SINH GIỎI KỸ NĂNG VIẾT ĐOẠN VĂN NGHỊ LUẬN XÃ HỘI 200 C...
TÀI LIỆU BỒI DƯỠNG HỌC SINH GIỎI KỸ NĂNG VIẾT ĐOẠN VĂN NGHỊ LUẬN XÃ HỘI 200 C...TÀI LIỆU BỒI DƯỠNG HỌC SINH GIỎI KỸ NĂNG VIẾT ĐOẠN VĂN NGHỊ LUẬN XÃ HỘI 200 C...
TÀI LIỆU BỒI DƯỠNG HỌC SINH GIỎI KỸ NĂNG VIẾT ĐOẠN VĂN NGHỊ LUẬN XÃ HỘI 200 C...
 

đề thi ko đáp.pdf

  • 1. Tuyển tập đề chọn đội tuyển VMO Nguyễn Minh Tuấn, Huỳnh Kim Linh, Nguyễn Nhất Huy, Đoàn Duy Tùng Ngày 12 tháng 10 năm 2020 Mục lục 1 Đề thi tỉnh Phú Thọ. 5 A Phú Thọ ngày thi thứ nhất. 5 B Phú Thọ ngày thi thứ hai. 5 2 Đề thi tỉnh Nam Định. 6 A Nam Định ngày thi thứ nhất. 6 B Nam Định ngày thi thứ hai. 6 3 Đề thi tỉnh Hưng Yên. 7 A Hưng Yên ngày thi thứ nhất. 7 B Hưng Yên ngày thi thứ 2. 8 4 Đề thi tỉnh Hà Tĩnh. 8 A Hà Tĩnh ngày thi thứ nhất. 8 B Hà Tĩnh ngày thi thứ hai. 9 5 Đề thi tỉnh Quảng Ninh. 9 A Ngày thi thứ nhất. 9 B Ngày thi thứ hai. 10 6 Đề thi tỉnh Lạng Sơn. 10 A Ngày thi thứ nhất. 10 1
  • 2. Š Tuyển tập đề chọn đội tuyển các tỉnh trên cả nước VMO 2021 7 Đề thi học sinh giỏi Chuyên Lào Cai 11 8 Trải nghiệm VMO 2021 - Bring Math to Everyone - BM2E 12 A Vòng 1, ngày thi thứ nhất. 12 B Vòng 1, ngày thi thứ hai. 12 9 Đề thi tỉnh Sóc Trăng. 13 A Sóc Trăng ngày thi thứ nhất. 13 B Sóc Trăng ngày thi thứ hai. 13 10 Đề thi tỉnh Nghệ An. 14 A Ngày thi thứ nhất. 14 B Ngày thi thứ hai 14 11 Đề thi Chuyên Sư Phạm Hà Nội 2020 14 A Ngày thi thứ nhất. 14 B Ngày thi thứ hai. 15 12 Đề thi tỉnh Vũng Tàu 15 A Ngày thi thứ nhất. 15 13 Đề thi phổ thông năng khiếu 16 A Ngày thi thứ nhất. 16 B Ngày thi thứ hai. 17 14 Đề thi tỉnh Ninh Bình. 18 15 Đề thi thành phố Cần Thơ 18 16 Đề thi tỉnh Bắc Ninh. 19 A Ngày thi thứ nhất. 19 B Ngày thi thứ hai. 20 ® Chinh phục olympic toán 2 h Tạp chí và tư liệu toán học
  • 3. Š Tuyển tập đề chọn đội tuyển các tỉnh trên cả nước VMO 2021 17 Đề thi tỉnh Lâm Đồng. 20 A Ngày thi thứ nhất. 20 18 Đề thi tỉnh Bắc Giang 21 19 Đề thi tỉnh Đồng Tháp. 21 20 Đề thi thành phố Hà Nội. 22 21 Đề thi tỉnh Hà Nam. 23 22 Đề thi tỉnh Kiên Giang. 23 A Ngày thi thứ nhất. 23 B Ngày thi thứ hai. 24 23 Đề thi tỉnh Quảng Ngãi. 25 24 Đề thi tỉnh Tây Ninh. 25 A Ngày thi thứ nhất. 25 B Ngày thi thứ hai. 26 25 Đề thi tỉnh Quảng Nam. 26 26 Đề thi tỉnh Quảng Trị. 27 27 Đề thi chọn đội tuyển THPT Chuyên Nguyễn Du. 27 A Ngày thi thứ nhất. 27 B Ngày thi thứ hai. 28 28 Đề thi tỉnh Phú Yên. 28 29 Đề chọn đội tuyển chuyên Đại học Vinh. 29 A Ngày thi thứ nhất. 29 B Ngày thi thứ hai. 30 30 Đề thi tỉnh Ninh Thuận. 30 ® Chinh phục olympic toán 3 h Tạp chí và tư liệu toán học
  • 4. Š Tuyển tập đề chọn đội tuyển các tỉnh trên cả nước VMO 2021 31 Đề thi tỉnh Đồng Nai. 31 32 Đề thi tỉnh Quảng Ngãi. 31 A Ngày thi thứ nhất. 31 B Ngày thi thứ hai. 32 33 Đề thi tỉnh Vĩnh Long. 33 A Ngày thi thứ nhất. 33 B Ngày thi thứ hai. 33 34 Đề thi tỉnh Quảng Bình. 34 35 Đề thi tỉnh Bình Dương 34 36 Đề thi tỉnh Yên Bái. 35 ® Chinh phục olympic toán 4 h Tạp chí và tư liệu toán học
  • 5. Š Tuyển tập đề chọn đội tuyển các tỉnh trên cả nước VMO 2021 Tóm tắt nội dung Các đề vẫn đang tiếp tục được cập nhật trong link của bài viết này! 1 Đề thi tỉnh Phú Thọ. A Phú Thọ ngày thi thứ nhất. L Câu 1. Cho a, b ∈ R, a 6= b. Giải hệ phương trình      3x + z = 2y + (a + b) 3x2 + 3xz = y2 + 2 (a + b) y + ab x3 + 3x2 z = y2 (a + b) + 2yab. L Câu 2. Cho dãy số thực dương (an)n>1 thỏa mãn điều kiện a1 + a2 + ... + an + an+1 + an+2 < 4an+1, ∀n ∈ N∗ . Chứng minh rằng a1 + a2 + ... + an 6 an+1, ∀n ∈ N∗ . L Câu 3. Giả sử O, I lần lượt là tâm đường tròn ngoại tiếp, nội tiếp tam giác ABC với bán kính R, r tương ứng. Gọi P là điểm chính giữa cung _ BAC, QP là đường kính của (O), D là giao điểm của PI và BC, F là giao điểm của đường tròn ngoại tiếp tam giác AID với đường thẳng PA. Lấy E trên tia DP sao cho DE = DQ. 1 Chứng minh rằng [ IDF = 90◦ . 2 Giả sử [ AEF = [ APE, chứng minh rằng sin2 [ BAC = 2r R . L Câu 4. Trên mặt phẳng tọa độ Oxy, cho S là tập hợp các điểm (x; y) thỏa mãn đồng thời hai điều kiện x, y ∈ N; 0 6 y 6 x 6 2020. 1 Tính số phần tử của S. 2 Hỏi có bao nhiêu tập con A gồm 2020 phần tử của S sao cho A không chứa hai điểm (x1; y1) ; (x2; y2) thỏa mãn (x1 − x2) (y1 − y2) = 0? B Phú Thọ ngày thi thứ hai. L Câu 5. Cho đường tròn tâm O với hai điểm B, C cố định nằm trên đường tròn đó. Điểm A thay đổi trên đường tròn (O) sao cho tam giác ABC nhọn. Gọi D là điểm đối xứng với A qua O, tiếp tuyến với đường tròn (O) tại D cắt đường thẳng BC tại điểm K. Đường thẳng KO cắt AB, AC lần lượt tại E và F. 1 Chứng minh DF song song với AB. 2 Gọi I là tâm đường tròn ngoại tiếp tam giác AEF. Chứng minh rằng AI luôn đi qua một điểm cố định khi A thay đổi. L Câu 6. Xét a, b, c ∈ Z, 1 6 a < b < c thỏa mãn hệ phương trình đồng dư      (b + 1) (c + 1) ≡ 1(moda) (c + 1) (a + 1) ≡ 1(modb) (a + 1) (b + 1) ≡ 1(modc). ® Chinh phục olympic toán 5 h Tạp chí và tư liệu toán học
  • 6. Š Tuyển tập đề chọn đội tuyển các tỉnh trên cả nước VMO 2021 1 Chứng minh rằng hệ phương trình có vô hạn nghiệm (a; b; c) . 2 Giả sử gcd (a, b) = 1. Tìm tất cả các nghiệm của hệ phương trình đó. L Câu 7. Cho hàm số f : R → [0; +∞) thỏa mãn f2 (x + y) + f2 (x − y) = 2f2 (x) + 2f2 (y), ∀x, y ∈ R. Chứng minh rằng f (x + y) 6 f(x) + f(y), ∀x, y ∈ R, (ở đây f2 (x) = (f(x))2 ). 2 Đề thi tỉnh Nam Định. A Nam Định ngày thi thứ nhất. L Câu 1. Giải hệ phương trình ( x3 + y2 + 3x = x2 + x + 3 y p 3y + 4 + 6x + 21 = p x − 2y + 5 + 3xy. . L Câu 2. Cho dãy số (xn) xác định bởi x1 = 1 2 và xn+1 = x2 n − xn + 1, ∀n ∈ N∗ . 1 Chứng minh rằng xn 6 2n − 1 2n , ∀n ∈ N∗ . 2 Tìm giới hạn lim (xn+1 + x1x2 2x3 3 . . . xn n). L Câu 3. Tìm tất cả các hàm số f : R → R thỏa mãn f(x+y)·f(x−y)+y2 = f (x2 ) , ∀x, y ∈ R. L Câu 4. Trên đường tròn (O) lấy hai điểm A, M sao cho AM không là đường kính. Điểm I nằm trên đoạn OA (I không trùng với các điểm O, A). Đường tròn (I, IA) và đường tròn đường kính IM cắt nhau tại hai điểm B, C. Các đường thẳng MB, MI, MC cắt đường tròn (O) tại các điểm thứ hai lần lượt là D, E, F sao cho tia MA nằm giữa hai tia MD, ME. Đường thẳng DF cắt các đường thẳng ME, MA, AE lần lượt tại T, S, Q. 1 Chứng minh rằng ME là phân giác của DMF và SD.SF = ST.SQ. 2 Chứng minh rằng AD AF = BD CF và ba điểm B, C, Q thẳng hàng. B Nam Định ngày thi thứ hai. L Câu 5. 1 Có bao nhiêu cách chọn ra hai số phân biệt từ 100 số nguyên dương đầu tiên sao cho hai số này hơn kém nhau không quá 10 đơn vị? 2 Cho tập hợp X = n 22k |k ∈ N; 0 6 k 6 2020 o . Xét tất cả các tập hợp con khác rỗng của X. Với mỗi tập hợp con như vậy, ta tính tích của mọi phần tử trong tập hợp đó (nếu tập hợp con chỉ có một phần tử thì quy ước tích chính là phần tử đó). Tính tổng của tất cả các tích thu được. L Câu 6. Xét x, y, z ∈ (0; 1]. Chứng minh rằng 36 2 √ x + √ y √ y + 2 √ z + 27 (x − 1) (y − 1) (z − 1) 4 Tìm tất cả các bộ giá trị (x; y; z) để dấu bằng trong bất đẳng thức trên xảy ra. ® Chinh phục olympic toán 6 h Tạp chí và tư liệu toán học
  • 7. Š Tuyển tập đề chọn đội tuyển các tỉnh trên cả nước VMO 2021 L Câu 7. Với P(x) = x2 − 2020x + 1, ta định nghĩa ( P1(x) = P(x) Pn+1(x) = P (Pn(x)) , ∀n ∈ N∗ . 1 Chứng minh rằng với mỗi số thực k thuộc (0; 1), đa thức P2(x) − α có đúng bốn nghiệm thực phân biệt. 2 Tìm tất cả các số thực dương k sao cho Với mọi số nguyên dương n, đa thức Pn(x) − kx luôn có hai nghiệm thực mà hiệu của chúng lớn hơn √ 2019 × 2023. L Câu 8. Cho tam giác ABC nội tiếp đường tròn (O). P là điểm bất kì nằm trong tam giác ABC nhưng không thuộc đường thẳng AO. Đường thẳng AP cắt đường tròn (O) tại điểm D (khác A). Kẻ các đường kính DE, AF của đường tròn (O). Các đường thẳng EP, FP lần lượt cắt đường tròn (O) tại các điểm G, H (khác E, F). Gọi K là giao điểm của các đường thẳng AH và DG, L là hình chiếu vuông góc của K trên đường thẳng OP. Giả sử rằng hai đường thẳng KL và AD cắt nhau. 1 Chứng minh rằng bốn điểm A, L, K, D cùng thuộc một đường tròn (gọi đường tròn này là (S)). 2 Chứng minh rằng các đường thẳng OP và EF cắt nhau tại điểm T thuộc đường tròn (S). L Câu 9. 1 Cho p là số nguyên tố và n là số nguyên dương lớn hơn 1. Chứng minh rằng mọi ước nguyên tố của số A = 1 + n + n2 + ... + np−1 hoặc là p, hoặc chia cho p dư 1. 2 Tìm tất cả các số nguyên tố p sao cho tồn tại số nguyên dương n mà số B = 2+n+n2 +...+np−1 là lũy thừa bậc 5 của một số nguyên dương. 3 Đề thi tỉnh Hưng Yên. A Hưng Yên ngày thi thứ nhất. L Câu 1. Cho dãy số (an) xác định bởi a0 = 2020 và an+1 = a2 n 1 + an , ∀n 0. 1 Chứng minh rằng dãy số (an) có giới hạn và tìm giới hạn đó. 2 Tính [a1000] ([x] phần nguyên của số thực x). L Câu 2. Cho P(x) là một đa thức bậc ba, xét đa thức Q(x) = x3 + 2x + 1 − P(x) 2x3 − 6x2 + 5 − P(x) 1 Giả sử Q(x) 6 0, ∀x ∈ R và P(0) = 4, tính Q (−1). 2 Hỏi có tồn tại hay không đa thức P(x) để Q(x) 0, ∀x ∈ R. L Câu 3. Cho tam giác ABC có đường tròn nội tiếp (I) tiếp xúc với các cạnh BC, CA, AB lần lượt tại các điểm X, Y, Z. Các đường phân giác trong và ngoài tại góc A của tam giác ABC cắt BC tại E, F. Các tiếp tuyến kẻ từ E, F đến (I) cắt nhau tại điểm D (D khác E, F). Trên BI lấy điểm K sao cho DK ⊥ AI. 1 Giả sử BC cố định và A thay đổi, chứng minh K thuộc một đường tròn cố định. 2 Gọi M, N là các tiếp điểm của (I) với các tiếp tuyến DE, DF. Đường thẳng qua D song song với AB cắt AC, MN tại P, Q. Đường thẳng QZ cắt đường tròn (I) tại T. Chứng minh PT tiếp xúc với đường tròn (I). ® Chinh phục olympic toán 7 h Tạp chí và tư liệu toán học
  • 8. Š Tuyển tập đề chọn đội tuyển các tỉnh trên cả nước VMO 2021 L Câu 4. Cho số nguyên tố p 5. Đặt n = 4p − 1. 1 Chứng minh rằng n có ít nhất 3 ước nguyên tố phân biệt và 2n ≡ 8 (modn). 2 Với a là một số tự nhiên, chứng minh rằng 2 + a + a2 + ... + ap−1 không là số chính phương. B Hưng Yên ngày thi thứ 2. L Câu 1. Tìm tất cả các hàm số f : R → R thỏa mãn điều kiện f(x + 2020.f(x)) = f(x) + 2020xf(y) với mọi x, y ∈ R. L Câu 2. Cho tam giác ABC ngoại tiếp đường tròn (I) và có trực tâm H. Gọi D, E, F lần lượt là các tiếp điểm của BC, CA, AB với đường tròn (I). Gọi S, T lần lượt là trung điểm của AH, BH và gọi M là điểm đối xứng với S qua FE, gọi N là điểm đối xứng với T qua FD. Chứng minh rằng đường thẳng MN đi qua trực tâm của tam giác DEF. L Câu 3. Cho bảng ô vuông ABCD kích thước 2021 × 2021 gồm 20212 ô vuông đơn vị, mỗi ô vuông đơn vị được tô bởi một trong ba màu đen, trắng hoặc xám. Một cách tô màu được gọi là đối xứng nếu mỗi ô vuông đơn vị có tâm trên đường chéo AC được tô màu xám và mỗi cặp ô vuông đơn vị đối xứng qua AC được tổ cùng màu đen hoặc cùng màu trắng. Người ta điền vào mỗi ô xám số 0, mỗi ô trắng một số nguyên dương và mỗi ô đen một số nguyên âm. Một cách điền số như vậy được gọi là k - cân đối (với k là số nguyên dương) nếu thỏa mãn các điều kiện sau 1 Mỗi cặp ô vuông đơn vị đối xứng qua AC được điền cùng một số nguyên thuộc đoạn [−k; k]. 2 Nếu một hàng và một cột giao nhau tại ô đen thì tập các số nguyên dương được điền trên hàng đó và tập số nguyên dương được điền trên cột đó không giao nhau, nếu một hàng và một cột giao nhau tại ô trắng thì tập các số nguyên âm được điền trên hàng đó và tập các số nguyên âm được được điền trên cột đó không giao nhau. Tìm giá trị nhỏ nhất của k để với mọi cách tô màu đối xứng, luôn tồn tại cách điền k - cân đối. 4 Đề thi tỉnh Hà Tĩnh. A Hà Tĩnh ngày thi thứ nhất. L Câu 1. Cho phương trình xn = x + 1. Chứng minh rằng với mỗi n ∈ N, n 2, phương trình có nghiệm dương duy nhất, ký hiệu là xn. 1 Tính giới hạn của dãy số (un) với un = n (xn − 1). 2 Tìm số thực k sao cho dãy số vn = nk (xn+1 − xn) có giới hạn hữu hạn khác 0. L Câu 2. Tìm tất cả các hàm số f : R → R thỏa mãn f(y−f(x)) = f(f(x))−2yf(x)+f(y)∀x, y ∈ R. L Câu 3. Cho tam giác nhọn ABC có AB AC BC và nội tiếp đường tròn (O; R). Đường thẳng d thay đổi nhưng luôn vuông góc với đoạn thẳng OA và cắt cạnh AB, AC lần lượt tại M, N. Gọi K là giao điểm của đường thẳng BN và CM; P là giao điểm của đường thẳng AK và BC; I là trung điểm của BC. 1 Chứng minh tứ giác MNIP nội tiếp được trong một đường tròn. 2 Gọi H là trực tâm tam giác AMN. Chứng minh rằng đường thẳng HK luôn đi qua một điểm cố định khi đường thẳng d thay đổi. L Câu 4. Tìm tất cả các số nguyên tố m, n sao cho 7m + 7n mn là một số nguyên. ® Chinh phục olympic toán 8 h Tạp chí và tư liệu toán học
  • 9. Š Tuyển tập đề chọn đội tuyển các tỉnh trên cả nước VMO 2021 B Hà Tĩnh ngày thi thứ hai. L Câu 5. Với a, b ∈ Z, xét hai dãy đa thức ( P0(x) = x + a, Pn+1(x) = P2 n (x) + (−1)n n Q0(x) = x + b, Qn+1(x) = −Q2 n(x) + (−1)n n , ∀n 0, n ∈ Z. 1 Cho a 6= b, a − b 6= ±1. Hỏi đa thức f(x) = P0 2020(x).Q0 2020(x) có bao nhiêu nghiệm phân biệt? 2 Tìm điều kiện của a, b để tồn tại nsao cho Pn(x) + Qn(x) chia hết cho x + 2. L Câu 6. Cho tam giác ABC không cân, đường tròn (I) nội tiếp tam giác tiếp xúc với các cạnh BC, CA, AB lần lượt tại D, E, F. Đường thẳng AI cắt đường tròn (I) lần lượt tại M, Hvà cắt EF tại N(M nằm giữa A và I). Tiếp tuyến tại I của đường tròn ngoại tiếp tam giác IMD cắt DN tại P. Trên đường thẳng PM lấy Q sao cho DQ vuông góc với EF. 1 Chứng minh rằng PH song song với AD. 2 Chứng minh đường thẳng DA đi qua trung điểm của đoạn thẳng MQ. L Câu 7. Cho bảng vuông n × n ô vuông (n 2) với các ô vuông được tô bằng 2 màu đen hoặc trắng (mỗi ô chỉ tô bởi một màu). Biết rằng cứ mỗi bước, ta chỉ được thay đổi màu của toàn bộ các ô trong một hàng hoặc một cột (ô trắng thành đen và ô đen thành trắng). 1 Giả sử trong bảng có đúng 1 ô được tô đen. Hỏi sau một số bước đổi màu các hàng hoặc cột nào đó thì bảng toàn ô trắng được hay không? 2 Có tất cả bao nhiêu cấu hình ban đầu sao cho sau hữu hạn bước đổi màu hàng hoặc cột thì bảng gồm toàn ô trắng? Ví dụ. Cấu hình H1 là 1 cấu hình thỏa mãn với n = 3. 5 Đề thi tỉnh Quảng Ninh. A Ngày thi thứ nhất. L Câu 1. Cho các số thực a, b, c ∈ [1; 2], chứng minh rằng 9 6 (a + b + c) 1 a + 1 b + 1 c 6 10 L Câu 2. Với mỗi số nguyên dương n, xét đa thức Pn(x) = xn + (x − 1)n − (x + 1)n . 1 Chứng minh rằng với mỗi n nguyên dương, đa thức Pn(x) có duy nhất một nghiệm dương, kí hiệu là rn và rn rn+1, ∀n ∈ N∗ . 2 Tính lim rn n . ® Chinh phục olympic toán 9 h Tạp chí và tư liệu toán học
  • 10. Š Tuyển tập đề chọn đội tuyển các tỉnh trên cả nước VMO 2021 L Câu 3. Cho tam giác ABC nội tiếp đường tròn (C) có AB AC. Các điểm M, N lần lượt nằm trên các cạnh AB, BC sao cho AM = CN. Gọi K là giao điểm của các đường thẳng MN và AC, P và Q lần lượt là tâm đường tròn nội tiếp tam giác AMK và tâm đường tròn bàng tiếp góc K của tam giác CNK. Gọi S là điểm chính giữa cung ABC của (C). Chứng minh rằng SP = SQ. L Câu 4. 1 Cho các số nguyên dương a, n với a 2. Chứng minh rằng k | ϕ(ak − 1), trong đó ϕ là phi hàm Euler. 2 Cho số nguyên m 2 và p là một số nguyên tố không là ước của m nhưng p là ước của ϕ(m). Chứng minh rằng tồn tại số nguyên tố q sao cho q ≡ 1 (mod p). 3 Cho p là một số nguyên tố. Chứng minh rằng có vô hạn số nguyên tố có dạng pk + 1. B Ngày thi thứ hai. L Câu 5. Tìm tất cả các hàm số f : R → R thỏa mãn f(x+y)−f(x)f(y) = f(xy)−2xy−1, ∀x, y ∈ R. L Câu 6. Cho P(x) là đa thức bậc 4 với hệ số thực, có đúng 4 nghiệm dương phân biệt. Chứng minh rằng phương trình sau cũng có 4 nghiệm dương phân biệt 1 − 3x x4 · P(x) + 1 − 1 − 3x x4 · P0 (x) − P00 (x) = 0 L Câu 7. Cho tam giác ABC nội tiếp đường tròn (O), điểm D cố định trên cung BC không chứa A nhưng phải là điểm chính giữa cung đó. Điểm P di động trên đoạn AD và P 6= A, D. Điểm Q đẳng giác với P trong tam giác ABC. 1 Gọi G là giao điểm của DQ và BC. Chứng minh rằng PG k AQ. 2 Kẻ dây cung DE của đường tròn (O) vuông góc với BC. K là trung điểm DE, R là hình chiếu vuông góc của Q lên BC. Chứng minh rằng đường thẳng qua R và vuông góc với PK luôn đi qua một điểm cố định khi P di động trên AD. L Câu 8. Tìm tất cả các số nguyên dương n 60 sao cho tập hợp M = {n; n + 1; n + 2; ...; 60} có thể viết được thành hợp của các tập con đôi một rời nhau thỏa mãn tính chất trong mỗi tập con có một phần tử bằng tổng tất cả các phần tử còn lại trong tập con đó . 6 Đề thi tỉnh Lạng Sơn. A Ngày thi thứ nhất. L Câu 1. 1 Giải phương trình n q (x − 1)2 − 4 n q (x + 1)2 = −3 n √ x2 − 1, với n ∈ N∗ , n 2. 2 Giải hệ phương trình x3 − 2y3 = 4x2 y − 5xy2 √ x + y + √ x − y = 2 . L Câu 2. Cho các số thực dương a, b, c thỏa mãn a + b + c = 1. Chứng minh rằng 1 a b + c + b c + a + c a + b 3 − 9 2 (ab + bc + ca). ® Chinh phục olympic toán 10 h Tạp chí và tư liệu toán học
  • 11. Š Tuyển tập đề chọn đội tuyển các tỉnh trên cả nước VMO 2021 2 27abc 6 a3 + b3 + c3 + 24abc 6 1. L Câu 3. Cho tam giác ABC nhọn và nội tiếp đường tròn (O) với các đường cao BE, CF(E ∈ CA, F ∈ AB). Gọi D = AO ∩ BC. Tiếp tuyến tại B và C của (O) cắt nhau tại S. Gọi P, Q lần lượt là giao điểm của SB, SC với EF. 1 Chứng minh rằng DB DC = SQ SP = sin 2 [ ACB sin 2 [ ABC . 2 Chứng minh rằng SD đi qua trung điểm của PQ. 3 Chứng minh rằng đường tròn ngoại tiếp tam giác SPQ tiếp xúc với (O). L Câu 4. Cho dãy số (un) xác định bởi    u0 = 1 un+1 = un + 1 u2 n , (n ∈ N) . 1 Chứng minh rằng u3 n 1 + 3n với mọi n ∈ N∗ . 2 Tìm lim u3 n n . 7 Đề thi học sinh giỏi Chuyên Lào Cai L Câu 1. Cho dãy số (un) xác định bởi ( u1 = 1 2 un+1 = 1 + un − u2 n, ∀n ∈ N∗ 1 Chứng minh rằng 1 2 6 un 6 5 4 , ∀n ∈ N∗ . 2 Chứng minh dãy (un) có giới hạn hữu hạn. Tìm giới hạn hữu hạn đó. L Câu 2. Cho đa thức hai đa thức Q(x) ∈ R[x] và P(x) = a + bx + cx2 + x3 Q(x), trong đó a, b, c là các số thực thỏa mãn ac 6= 0. Chứng minh rằng nếu tất cả các nghiệm của P(x) đều là số thực, thì b2 2ac. L Câu 3. Cho tam giác ABC nhọn không cân nội tiếp đường tròn (O) có các đường cao BE, CF cắt nhau tại H. Gọi I là trung điểm của BC. Tia IH cắt (O) tại T. Trên đường thằng EF lấy điểm D sao cho HD k BC. 1 Chứng minh rằng DT tiếp xúc với đường tròn ngoại tiếp tam giác HEF. 2 Gọi M, N lần lượt là giao điểm của EF với các đường tròn (IBT), (ICT) thỏa mãn M khác phía E đối với F và N khác phía F đối với E. Gọi P là giao điểm thứ hai của AH với (O). Chứng minh rằng BM, CN, TP đồng quy. L Câu 4. Cho p là số nguyên tố lớn hơn 3. Đặt 1 − 1 2 + 1 3 − . . . + 1 p − 2 − 1 p − 1 = a (p − 1)! Chứng minh rằng a ≡ 2 − 2p p (mod p). L Câu 5. Cho tập hợp T = {1; 2; ...; 2020}. Có bao nhiêu cách chọn ra 10 số từ tập T sao cho hai số bất kì trong các số được chọn luôn hơn kém nhau ít nhất 7 đơn vị. ® Chinh phục olympic toán 11 h Tạp chí và tư liệu toán học
  • 12. Š Tuyển tập đề chọn đội tuyển các tỉnh trên cả nước VMO 2021 8 Trải nghiệm VMO 2021 - Bring Math to Everyone - BM2E A Vòng 1, ngày thi thứ nhất. L Câu 1. Cho α = 3 + √ 10. Các dãy số (xn)∞ n=1 , (yn)∞ n=1 được xác định như sau xn = 5α + 1 αn + α và yn = (1 + x1) (1 + x2) . . . (1 + xn) với mọi n = 1, 2, 3, .... Chứng minh rằng dãy (yn) có giới hạn hữu hạn và tìm giới hạn đó. L Câu 2. Tìm tất cả các hàm số f : R → R sao cho với mọi số thực x, y đẳng thức sau được thỏa mãn f(f(x) + y) + 1 = f (x2 + y) + 2f(x) + 2y L Câu 3. Cho tam giác nhon ABC nội tiếp đường tròn (C), AB AC. Gọi P, Q là trung điểm các cung nhỏ và cung lớn BC. M là chân đường vuông góc hạ từ Q xuống AB. Chứng minh rằng đường tròn ngoại tiếp tam giác AMC đi qua trung điểm của đoạn thẳng AP. L Câu 4. Trên bảng có các số 1, 2, 3, ..., 14, 15. An và Bình thực hiện quá trình sau: An chọn hai số bất kỳ trên bảng (ví dụ là x, y) xóa chúng đi và viết lại tổng của hai số này lên bảng. Trong lúc đó Bình viết giá trị của xy(x + y) và vở của bạn ấy. Hai bạn cứ làm như vậy cho đến khi trên bảng chỉ còn lại một số thì dừng lại. Lúc đó Bình cộng tất cả các số mà bạn ấy đã viết vào vở, gọi tổng thu được là S. 1 Chứng minh rằng S là một hằng số, tức là giá trị của S không phụ thuộc và thứ tự mà An và Bình thực hiện. 2 Hãy tìm giá trị của S. B Vòng 1, ngày thi thứ hai. L Câu 5. Với mỗi số thực x, ký hiệu [x], là số nguyên lớn nhất không vượt quá x, gọi là phần nguyên của x. Ta cũng ký hiệu {x} = x − [x] và gọi là phần lẻ của x. Xét tập hợp X các số nguyên dương không lớn hơn 2020 và không phải là lập phương của một số nguyên. 1 Chứng minh rằng với mọi n thuộc X ta có { 3 √ n} 1 500 . 2 Chứng minh rằng tồn tại n thuộc X sao cho { 3 √ n} 1 432 . L Câu 6. Một số nguyên dương được gọi là siêu hợp số nếu nó có thể phân tích thành tích của 3 thừa số nguyên dương 1. Ví dụ 12 là siêu hợp số vì 12 = 2.2.3. Một số nguyên dương được gọi là mịn nếu mọi ước nguyên tố của nó đều nhỏ hơn hay bằng 19. Chứng minh rằng từ một tập hợp gồm 46 số mịn, luôn tìm được 2 số có tổng là một siêu hợp số. L Câu 7. Cho tam giác ABC nhọn, không cân có các đường cao AD, BE, CF và (M) là đường tròn đường kính BC. Đường thẳng qua A song song với EF cắt DE, DF theo thứ tự tại K, L. 1 Đường tròn ngoại tiếp tam giác AEF cắt KL, AM tại R, S. Chứng minh rằng ba đường thẳng BK, CL và RS đồng quy. 2 Gọi (I), (J) theo thứ tự là đường tròn nội tiếp của các tam giác ADK, ADL. Chứng minh rằng ba đường tròn (I), (J) và (M) có một tiếp tuyến chung. ® Chinh phục olympic toán 12 h Tạp chí và tư liệu toán học
  • 13. Š Tuyển tập đề chọn đội tuyển các tỉnh trên cả nước VMO 2021 9 Đề thi tỉnh Sóc Trăng. A Sóc Trăng ngày thi thứ nhất. L Câu 1. Giải hệ phương trình sau trên tập số thực ( x2021 − y2022 = y4042 − xy2020 x2 − 2y2 = 2 √ 2x − 1 . L Câu 2. 1 Tìm nghiệm nguyên của phương trình x4 + y4 − z4 + 2z3 + 2x2 y2 + 3x2 + 4y2 + 21z2 − 22z − 120 = 0 2 Có bao nhiêu số tự nhiên có 3 chữ số khác nhau và chia hết cho 3 ? L Câu 3. Cho hình vuông ABCD và một đường thẳng ∆ đi qua A, Gọi E, F lần lượt là hình chiếu của B, D trên ∆. 1 Chứng minh DE⊥CF. 2 Gọi G, H lần lượt là giao điểm của BF với EC và ED, I là giao điểm của CH với ∆. Chứng minh bốn điểm C, F, G, I cùng nằm trên một đường tròn. L Câu 4. Cho các số thực dương a, b, c thỏa mãn 1 a + 1 b + 1 c = 2034. Tìm giá trị lớn nhất của biểu thức P = 1 4a + 10b + 2020c + 1 4c + 10a + 2020b + 1 4b + 10c + 2020a B Sóc Trăng ngày thi thứ hai. L Câu 1. Cho dãy số (xn) thỏa x1 = 2020, xn+1 = xn 2021 + xn + 25 xn 2020 − xn + 11 ∀n ∈ N∗ Đặt un = 1 x1 2020 + 6 + 1 x2 2020 + 6 + 1 x3 2020 + 6 ....... + 1 xn 2020 + 6 = n X i=1 1 xi 2020 + 6 . Tính lim un. L Câu 2. Cho đa giác đều có 1000 đỉnh A1A2A3A4....An nội tiếp đường tròn (O). 1 Có bao nhiêu hình chữ nhật có các đỉnh là đỉnh của đa giác và hình chữ nhật đó có kích thước khác nhau? 2 Cho quy tắc sau: “Trên đường tròn (O) đánh dấu đỉnh A, đi theo chiều tăng của thứ tự các đỉnh lần lượt đánh dấu các đỉnh cách đỉnh vừa được đánh dấu 15 đỉnh, đến gặp vị trí đỉnh đã được đánh dấu thì kết thúc”. Hỏi khi quy tắc này kết thúc, trên đường tròn (O) còn lại bao nhiêu đỉnh chưa đámh dấu, các đỉnh này có lập thành đa giác đều hay không? L Câu 3. Cho tam giác ABC có ba góc nhọn và các đường cao AE, BF, CI. 1 Cho ∠BAC = 60◦ và AD là phân giác trong của góc A(D ∈ BC). Chứng minh AD AB + AD AC = √ 3. 2 Gọi M, Q lần lượt là hình chiếu vuông góc của E trên AB, AC; N, R lần lượt là hình chiếu vuông góc của F trên AB, BC; P, S lần lượt là hình chiếu vuông góc của I trên AC, BC. Chứng minh các điểm M, N, P, Q, R, S cùng nằm trên một đường tròn. ® Chinh phục olympic toán 13 h Tạp chí và tư liệu toán học
  • 14. Š Tuyển tập đề chọn đội tuyển các tỉnh trên cả nước VMO 2021 10 Đề thi tỉnh Nghệ An. A Ngày thi thứ nhất. L Câu 1. Cho α là số thực thuộc khoảng (1; 2). Xét dãy số dương (un) thỏa mãn uα n u1 + u2 + ... + un−1, ∀n 2 Chứng minh rằng tồn tại hằng số C sao cho un Cn, ∀n ∈ N∗ . L Câu 2. Tìm tất cả các đa thức hệ số nguyên P(x) thỏa mãn a2 +b2 −c2 là ước của P(a)+P(b)− P(c) với mọi bộ số tự nhiên a, b, c mà a2 + b2 − c2 6= 0. L Câu 3. Cho tứ giác lồi ABCD có AB = BD = DA, CB CD, các đường chéo AC và BD cắt nhau tại O, Gọi K là trung điểm của AO. Trên tia đối của tia BA lấy điểm E sao cho BE = DO, trên tia đối của tia DA lấy điểm F sao cho DF = BO. Các đường thẳng KE, KF lần lượt cắt các đường thẳng CB, CD tại M, N. Các đường thẳng BD, MN cắt nhau tại I. Chứng minh AK vuông góc với KI. L Câu 4. Cho bảng ô vuông với m hàng, n cột m, n ∈ N∗ . Ta tô màu k ô vuông của bảng thỏa mãn: với 4 ô vuông bất kỳ mà tâm của chúng là 4 đỉnh của một hình chữ nhật (có cạnh song song với cạnh của bảng ô vuông) thì chỉ cho phép tối đa 2 ô trong đó được tô màu. Tìm giá trị lớn nhất có thể của k. B Ngày thi thứ hai L Câu 5. Cho tập hợp A gồm hữu hạn các số thực dương. B là tập hợp gồm các số có dạng a + b c + d với a, b, c, d là các số thuộc tập hợp A. Chứng minh rằng bất đẳng thức sau luôn đúng : |B| 2|A|2 −1. L Câu 6. Cho tam giác ABC cân tại A nội tiếp (O). Gọi I là tâm nội tiếp tam giác ABC. BI, CI cắt lại (O) tại M, N. Điểm D bất kỳ nằm trên cung BC không chứa A, DB DC,D khác B, C. AD cắt BI, CI tại E, F. DM, DN cắt CI, BI tại P, Q. 1 Chứng minh rằng D, I, P, Q cùng nằm trên một đường tròn. Giả sử đường tròn đó là (ω). 2 Chứng minh rằng CE, BF cắt nhau tại một điểm thuộc đường tròn (ω). L Câu 7. Tồn tại hay không một tập hợp M thỏa mãn 1 M là tập hợp gồm 2020 số tự nhiên phân biệt. 2 Tổng tất cả các phần tử của các tập con bất kỳ khác rỗng của M đều là một số tự nhiên có dạng mk với m, k là số tự nhiên, k 2. 11 Đề thi Chuyên Sư Phạm Hà Nội 2020 A Ngày thi thứ nhất. L Câu 1. Cho dãy số (an) được xác định bởi a1 = 1 và an+1 = an √ an + 1 với n = 1, 2, 3, ... Chứng minh rằng lim n→+∞ (nan) = 0. L Câu 2. Cho số thực q. Biết rằng tồn tại một cấp số cộng (an) tăng ngặt sao cho với mỗi số nguyên dương k thì qk là một số hạng của (an). Chứng minh rằng q là số nguyên. ® Chinh phục olympic toán 14 h Tạp chí và tư liệu toán học
  • 15. Š Tuyển tập đề chọn đội tuyển các tỉnh trên cả nước VMO 2021 L Câu 3. Cho tam giác ABC không cân, có trọng tâm G và đường tròn nội tiếp (I) tiếp xúc với cạnh BC tại D. Gọi M, N, P lần lượt là trung điểm của các cạnh BC, CA, AB. Qua D, kẻ đường thẳng song song với AI cắt IM tại S. 1 Chứng minh rằng AS = ID. 2 Gọi X, Y là các điểm theo thứ tự thuộc MN, MP sao cho PX, NY là các tiếp tuyến của đường tròn (I). Chứng minh rằng IG vuông góc với XY . L Câu 4. Có 2020 tấm thẻ đỏ và 2020 tấm thẻ xanh. Trên mỗi tấm thẻ, người ta viết một số nguyên dương không vượt quá 2020 (hai tấm thẻ khác nhau có thể viết cùng một số). Chứng minh rằng có thể chọn ra một bộ thẻ đỏ và một bộ thẻ xanh sao cho tổng các số viết trên mỗi tấm thẻ ở hai bộ là bằng nhau (chú ý rằng một bộ có thể chỉ gồm 1 tấm thẻ). B Ngày thi thứ hai. L Câu 5. Tìm tất cả các hàm số f : R → R thỏa mãn f(x) = x.f(x2 ) với mọi x ∈ [0; 1]. L Câu 6. Cho dãy số (xn) được xác định bới x0 = 1, x1 = 3 và xn+2 = 6xn+1 − xn với mọi n ∈ N. 1 Chứng minh rằng với mỗi số nguyên dương n thì (x2 n − 1)2 + 1 là hợp số. 2 Cho k là số nguyên dương và p là một ước nguyên tố của x2k . Chứng minh rằng nếu p − 1 chia hết cho 4 thì p − 1 chia hết cho 2k+2 . L Câu 7. Cho tam giác ABC có AB AC, nội tiếp trong đường tròn (O). Gọi M, N theo thứ tự là trung điểm BC, OA. Xét D là một điểm thuộc đoạn thẳng MC, không trùng với hai đầu mút và E, F lần lượt là hình chiếu của B, C lên AD. Gọi G là giao điểm của đường thẳng qua E, song song AB và đường thẳng qua F, song song AC. Gọi I là tâm đường tròn ngoại tiếp tam giác GEF. Chứng minh rằng NI = NM. L Câu 8. Một hội nghị có n người tham dự (n 5). Biết rằng tồn tại số nguyên dương m sao cho trong n − 2 người bất kỳ tham dự hội nghị, luôn có đúng 3m cặp quen nhau. Tìm tất cả các giá trị có thể có của n. 12 Đề thi tỉnh Vũng Tàu A Ngày thi thứ nhất. L Câu 1. 1 Giải hệ phương trình      1 √ x − 3 7x − y = 1 2 1 √ y + 6 7x − y = 2 . 2 Cho các số thực dương x, y, z. Tìm giá trị lớn nhất của biểu thức P = x p (2x + y)(2x + z) + y p (2y + x)(2y + z) + z p (2z + x)(2z + y) L Câu 2. Cho a là một số thực và dãy số (xn) xác định bởi x1 = a; xn+1 =      √ xn + 3 − (2n + 1)xn − n n + 1 nếu xn 6 1 xn + n xn nếu xn 1 , n ∈ N∗ ® Chinh phục olympic toán 15 h Tạp chí và tư liệu toán học
  • 16. Š Tuyển tập đề chọn đội tuyển các tỉnh trên cả nước VMO 2021 1 Khi a = 2. Chứng minh rằng lim xn n = 1 và xác định tất cả các số thực β sao cho dãy số (yn) xác định bởi yn = nβ √ xn+1 − √ xn , ∀n ∈ N∗ có giới hạn hữu hạn khác 0. 2 Tìm tất cả các giá trị a 0 sao cho dãy số xn n có giới hạn. L Câu 3. Cho tam giác ABC có 3 góc nhọn ngoại tiếp đường tròn tâm (I) với AB AC. Gọi D, E, F lần lượt là tiếp điểm của (I) với BC, CA, AB. Các đường thằng ID và EF cắt nhau tại J. Đường thằng AJ cắt đường tròn tâm (I) tại các điểm K, L với K nằm giữa A và L. Đường thẳng qua A song song với BC cắt ID, EF lần lượt tại N và S. Đường thẳng qua K và song song với BC cắt (I) tại điểm X(X 6= K). Đường thằng qua L song song với BC cắt (I) tại điểm Y (Y 6= L). Các đường thẳng AX, AY cắt BC lần lượt tại Q và P. 1 Chứng minh rằng ND là phân giác của ENF và AJ đi qua trung điểm M của BC. 2 Chứng minh rằng M là trung điểm của PQ. L Câu 4. Tìm tất cả các hàm số f : (0; +∞) → R thỏa mãn f(x) + f(y) = q x y + py x f( √ xy); ∀x, y 0 L Câu 5. 1 Cho số nguyên tố p 7. Đặt 1+ 1 2 +. . .+ 1 p2 − 1 = A B với A, B là các số nguyên dương nguyên tố cùng nhau. Chứng minh rằng B không chia hết cho p và A chia hết cho p. 2 Trong không gian cho N điểm, trong đó không có 3 điểm nào thẳng hàng (N là số nguyên dương lớn hơn 3). Tất cả các cặp điểm trên được nối với nhau bởi N 2 đoạn thẳng. Mỗi đoạn thẳng được tô một trong hai màu xanh hoặc đỏ và thỏa mãn 2 điều kiện sau (a) Không có tam giác nào có đúng 1 cạnh xanh. (b) Không có 13 điểm nào mà tất cả các đoạn nối được tô cùng màu. Chứng minh rằng N 6 144. 13 Đề thi phổ thông năng khiếu A Ngày thi thứ nhất. L Câu 1. Với mỗi số nguyên dương n, tìm số thực Mn 0 lớn nhất sao cho với mọi số dương x1, x2, ..., xn đều có n X k=1 1 x2 k + 1 n X k=1 xk !2 Mn       n X k=1 1 xk + 1 n X k=1 xk       2 L Câu 2. Cho 2021 số nguyên khác 0. Biết rằng tổng của một số bất kỳ trong chúng với tích của tất cả 2020 số còn lại luôn âm ® Chinh phục olympic toán 16 h Tạp chí và tư liệu toán học
  • 17. Š Tuyển tập đề chọn đội tuyển các tỉnh trên cả nước VMO 2021 1 Chứng minh rằng với mọi cách chia 2021 số này thành hai nhóm và nhân các số cùng nhóm với nhau thì tổng của hai tích thu được cũng luôn âm. 2 Một bộ số thỏa mãn điều kiện đề bài có thể có nhiều nhất bao nhiêu số âm? L Câu 3. Cho 2 hàm số f : R → R và g : R → R, thỏa g(2020) 0 và ( f (x − g (y)) = f (−x + 2g (y)) + xg (y) − 6 g (y) = g (2f (x) − y) , ∀x, y ∈ R 1 Chứng minh rằng g là hàm hằng. 2 Chứng minh đồ thị hàm số h(x) = f(x) − x nhận đường thẳng x = 1 làm trục đối xứng. L Câu 4. Tam giác ABC nhọn nội tiếp đường tròn (O) có H là trực tâm. AH, BH, CH lần lượt cắt BC, CA, AB tại D, E, F và I, M, N là trung điểm của BC, HB, HC. BH, CH cắt (O) tại L, K(L 6= B, K 6= C); KL cắt MN tại G. 1 Trên EF lấy điểm T sao cho AT vuông góc với IH. Chứng minh GT vuông góc với OH. 2 DE, DF lần lượt cắt MN tại P, Q. Gọi S là giao điểm của BQ với CP. Chứng minh rằng HS qua trung điểm của EF. B Ngày thi thứ hai. L Câu 5. Cho số nguyên dương n 1. Chứng minh rằng với mọi số thực a ∈ 0; 1 n và mọi đa thức P(x) có bậc 2n + 1 thỏa mãn điều kiện P(0) = P(1) = 0 luôn tồn tại các số thực x1, x2 thuộc [0; 1] sao cho P (x1) = P (x2) và x2 − x1 = a. L Câu 6. Giải phương trình sau trên tập số nguyên dương (x2 + 3) 3x+1 h (x2 + 3) 3x+1 + 1 i + x2 + y = x2 y L Câu 7. Cho các số nguyên n k t 0 và X = {1, 2, . . . , n}. Gọi F là họ các tập con có k phần tử của tập hợp X sao cho với mọi F, F0 ∈ F thì |F ∩ F0 | t. Giả sử không có tập con có t phần tử nào được chứa trong tất cả các tập F ∈ F. 1 Chứng minh rằng tồn tại một tập hợp B ⊂ X sao cho |B| 3k và |B ∩ F| t + 1 với mọi F ∈ F. 2 Chứng minh rằng |F| 3k t + 1 n k − t − 1 . L Câu 8. Cho tam giác ABC nội tiếp trong đường tròn (O) với B, C cố định và A thay đổi trên cung lớn BC. Dựng hình bình hành ABDC và AD cắt (BCD) ở K. 1 Gọi R1, R2 lần lượt là bán kính đường tròn ngoại tiếp (KBC), (KAC). Chứng minh rằng tích R1R2 không đổi. 2 Ký hiệu (T), (T0 ) lần lượt là các đường tròn cùng đi qua K, tiếp xúc với BD ở B và tiếp xúc với CD ở C. Giả sử (T), (T0 ) cắt nhau ở L 6= K. Chứng minh rằng AL luôn đi qua một điểm cố định. ® Chinh phục olympic toán 17 h Tạp chí và tư liệu toán học
  • 18. Š Tuyển tập đề chọn đội tuyển các tỉnh trên cả nước VMO 2021 14 Đề thi tỉnh Ninh Bình. L Câu 1. 1 Giải phương trình (2x − 4) √ 3x − 2 + √ x + 3 = 5x − 7 + √ 3x2 + 7x − 6. 2 Cho các số thực dương x, y. Chứng minh rằng ( √ x + √ y) 1 √ x + 3y + 1 √ y + 3x 6 2 L Câu 2. Cho dãy số (un) được xác định bởi công thức un = 22n+1 + n2 + n + 2 2n+1 + 2 , n ∈ N, trong đó [x] là số nguyên lớn nhất không vượt quá x và {x} = x − [x]. 1 Tính sáu số hạng đầu của dãy (un). 2 Tính giới hạn của dãy số (un). 3 Có bao nhiêu số hạng của dãy số (un) với n 6 86 thỏa mãn 2526 · 2n−99 2n + 1 6 un 6 23 65 . L Câu 3. Cho tam giác ABC nội tiếp đường tròn tâm (O). Đường tròn ngoại tiếp tam giác OBC có tâm S, cắt đường thẳng AB tại điểm X khác B và cắt đường tròn Euler của tam giác ABC tại hai điểm D, E. Gọi K, L theo thứ tự là các điểm đối xứng của S qua AB, AC. Chứng minh rằng 1 XO⊥AC. 2 Đường thằng KL đi qua tâm đường tròn Euler của tam giác ABC và hai đường thẳng AD, AE đối xứng nhau quá đường phân giác trong của [ BAC. L Câu 4. 1 Cho số nguyên tố p, số nguyên dương a thỏa mãn 1 a p + 1 và q là ước nguyên tố của A = 1 + a + · · · + ap−1 . Chứng minh rằng q − 1 chia hết cho p. 2 Cho số nguyên dương n. Có bao nhiêu số tự nhiên chia hết cho 3, có n chữ số và các chữ số đều thuộc tập A = {3; 4; 5; 6; 9}. 15 Đề thi thành phố Cần Thơ L Câu 1. Cho dãy số xác định bởi    u1 = 1 un+1 = u2 n + 2n 3un , n ∈ N∗ . 1 Chứng minh rằng √ n − 1 6 un 6 √ n, ∀n ∈ N∗ . 2 Tìm lim n→+∞ un √ n . L Câu 2. Cho x1, x2, ..., x2020 là các số thực dương. Chứng minh rằng 1 + x2 1 1 + x1x2 + 1 + x2 2 1 + x2x3 + ... + 1 + x2 2020 1 + x2020x1 2020. ® Chinh phục olympic toán 18 h Tạp chí và tư liệu toán học
  • 19. Š Tuyển tập đề chọn đội tuyển các tỉnh trên cả nước VMO 2021 L Câu 3. Cho hàm số f xác định trên tập số thực R thỏa mãn ( f(xy) = xf(y) + yf(x) f(x + y) = f x2021 + f y2021 , ∀x, y ∈ R. 1 Chứng minh rằng f là hàm cộng tính trên R. 2 Tính f √ 2020 . L Câu 4. Cho tam giác ABC có đường tròn nội tiếp (I) tiếp xúc với ba cạnh BC, CA, AB lần lượt tại D, E, F. Gọi K, M, Nlần lượt là giao điểm của EF, FD, DE với BC, CA, AB. Đường tròn đường kính KD, EM, FN lần lượt cắt (I) tại A1, B1, C1. 1 Chứng minh rằng các đường tròn đường kính KD, EM, FN đồng trục. 2 Chứng minh rằng DA1, EB1, FC1 đồng quy tại một điểm J. 3 Gọi T là trung điểm của EF. Chứng minh tiếp tuyến tại B và tại C của đường tròn ngoại tiếp tam giác IBC cắt nhau tại một điểm thuộc JT. L Câu 5. Cho n là số nguyên dương, n không có ước là số chính phương khác 1. Tồn tại hay không hai số nguyên dương nguyên tố cùng nhau x, y để xn + yn chia hết cho (x + y)3 ? L Câu 6. Một tàu du lịch có 100 khoang với sức chứa trong các khoang lần lượt là 101, 102, ..., 200 người. Hiện tại trên tàu đang có tổng cộng n người. Bây giờ, có một khách VIP đến và người thuyền trưởng muốn cấp cho anh ta một khoang cá nhân. Với mục đích đó, người thuyền trưởng muốn chọn hai khoang A và B,sau đó chuyển tất cả khách từ khoang A sang khoang B mà không vượt quá khả năng chứa của khoang B. Xác định n lớn nhất mà người thuyền trưởng có thể chắc chắn đạt được mục tiêu của mình cho dù lúc đầu n người này phân bố ở các khoang tàu như thế nào chăng nữa. 16 Đề thi tỉnh Bắc Ninh. A Ngày thi thứ nhất. L Câu 1. Cho số thực a và xét dãy số (xn) thỏa mãn x1 = x2 = 1, x3 = 0, xn+3 = x2 n+2 + x2 n+1 + x2 n 6 + a, ∀x ∈ N∗ 1 Chứng minh rằng với a = 0 thì dãy (xn) hội tụ. 2 Tìm số thực a lớn nhất sao cho dãy (un) hội tụ. L Câu 2. Cho tập S = {p1, p2, ..., p2021} gồm 2021 số nguyên tố phân biệt và P(x) là đa thức với các hệ số nguyên sao cho với mỗi số nguyên dương n đều tồn tại pi trong tập S là ước của P(n). Chứng minh rằng tồn tại pk trong tập S sao cho pk là ước của P(2021). L Câu 3. Cho tam giác ABC nhọn, không cân, nội tiếp đường tròn (O), có đường cao AD. Trên các cạnh AC, AB lần lượt lấy các điểm E, F sao cho AD, BE, CF đồng quy tại H. Giả sử tứ giác EFBC nội tiếp. 1 Chứng minh rằng H là trực tâm của tam giác ABC. 2 Đường thẳng vuông góc với OD tại D cắt AB tại K. Chứng minh rằng DHK + AHC = 180◦ L Câu 4. Tìm tất cả các hàm số f : R → R thỏa mãn f (x2020 + f (y)) = y+(f (x))2020 , ∀x, y ∈ R. ® Chinh phục olympic toán 19 h Tạp chí và tư liệu toán học
  • 20. Š Tuyển tập đề chọn đội tuyển các tỉnh trên cả nước VMO 2021 B Ngày thi thứ hai. L Câu 5. Cho dãy số (an)+∞ n=0 xác định như sau a0 = 0, a1 = 1, an+2 = 2an+1 − pan, ∀n ∈ N Biết rằng p là số nguyên tố và trong dãy có số hàng bằng −1. Tìm tất cả các giá trị có thể của p. L Câu 6. Cho đường tròn (O) bán kính R tiếp xúc với đường thẳng d tại điểm T cho trước. Một điểm M di động trên (O), tiếp tuyến của (O) tại M cắt d tại P. Gọi (C) là đường tròn tâm J qua M và tiếp xúc với d tại P và I là điểm đối xứng với P qua J. 1 Chứng minh OI = IP và (C) tiếp xúc với đường tròn cố định. 2 Tìm quỹ tích tâm J của đường tròn (C) khi M di động trên (O). L Câu 7. Một thành phố phát động phong trào đi bộ cho người dân. Thống kê điều tra cho thấy, trong tháng 9 dương lịch, mỗi ngày có ít nhất 84% tổng số người dân đi bộ. Trong một ngày, ta gọi hai người dân là một cặp chăm chỉ nếu trong ngày đó có ít nhất một trong hai người đi bộ. Chứng tỏ rằng, luôn tìm được hai người là cặp chăm chỉ trong tất cả các ngày của tháng 9. (Giả sử rằng, số người dân là số nguyên lớn hơn 2). 17 Đề thi tỉnh Lâm Đồng. A Ngày thi thứ nhất. L Câu 1. Chứng minh rằng (x2 + 2) (y2 + 2) (z2 + 2) 9 (xy + yz + zx) , ∀x, y, z 0. L Câu 2. Cho hàm số f(x) = ex (x + 1)2 . 1 Chứng minh rằng phương trình f(x) = x có duy nhất một nghiệm trong 1 2 , 1 . 2 Chứng minh dãy số (un) xác định bởi u1 = 1, un+1 = f (un) , ∀n ∈ N∗ có giới hạn. L Câu 3. Cho tam giác ABC nhọn, không cân, nội tiếp trong đường tròn (O). Gọi H là hình chiếu của A lên BC và D, E, M lần lượt là trung điểm HB, HC, BC. Đường tròn (ABE) tâm I cắt AC tại S và đường tròn (ACD) tâm J cắt AB tại R. 1 Chứng minh rằng BC = 4IJ. 2 Trung tuyến đỉnh H của tam giác AHM cắt RS tại T, chứng minh rằng các đường thẳng AT, BS, CR đồng quy. L Câu 4. Cho số a = 2019·2020·2021 và số nguyên dương n 3. Người ta xếp n số nguyên dương nào đó lên một đường tròn thỏa mãn đồng thời hai điều kiện sau (i) Hai số nằm cạnh nhau có tích không chia hết cho a. (ii) Hai số không nằm cạnh nhau có tích chia hết cho a. 1 Tìm một bộ các số nguyên dương thỏa mãn cách xếp trên. 2 Tìm giá trị lớn nhất của n. ® Chinh phục olympic toán 20 h Tạp chí và tư liệu toán học
  • 21. Š Tuyển tập đề chọn đội tuyển các tỉnh trên cả nước VMO 2021 L Câu 5. Cho tập S = {1, 2, . . . , n} với n là số nguyên dương. Gọi An là tập hợp các hoán vị (a1, a2, . . . , an) của tập S thỏa mãn điều kiện 2 (a1 + a2 + · · · + ak) chia hết cho k với mọi k = 1, 2, . . . , n. 1 Chứng minh rằng an − 1 chia hết cho n − 1 khi n chẵn và n 3. 2 Tìm số phần tử của A2020. 18 Đề thi tỉnh Bắc Giang L Câu 1. Cho các số thực a, b, c thỏa mãn 0 6 a 6 b 6 c và a + b + c = ab + bc + ca. Chứng minh rằng √ bc(a + 1) 2. L Câu 2. Cho tam giác ABC nhọn nội tiếp đường tròn (O), đường cao AD, trực tâm H. Đường tròn đường kính AH cắt (O) tại điểm Q khác A. Đường tròn đường kính HQ cắt (O) tại điểm K khác Q. Gọi M là trung điểm BC. 1 Đường thẳng qua H vuông góc với MH cắt BC tại X. Chứng minh rằng XK tiếp xúc với đường tròn ngoại tiếp tam giác KDM. 2 Đường thẳng KQ cắt đường tròn ngoại tiếp tam giác KDM tại N khác K. Chứng minh rằng MN chia đôi AQ. L Câu 3. Cho số thực a và dãy số (un)n1 xác định bởi u1 = a, un+1 = u2 n + un + a3 (n 1) . 1 Chứng minh rằng, với dãy a ∈ − 1 2 ; 0 , dãy số hội tụ và tìm giới hạn đó. 2 Cho a = 2020. Chứng minh rằng u2 n + 20203 luôn có ít nhất n+4 ước số nguyên tố khác nhau. L Câu 4. 1 Tìm tất cả các số tự nhiên k sao cho 2k + 1 và 4k + 1 đều là các số chính phương. 2 Với mỗi số tự nhiên k thỏa mãn đề bài, chứng minh rằng 35 | (k2 − 12k). L Câu 5. Sắp đến ngày Tết Trung thu, tổ chức Smile Foundation của trường THPT Chuyên Bắc Giang làm bánh gây quỹ từ thiện thường niên. Sản phẩm năm nay là một cặp bánh dẻo, bánh nướng có tổng giá cặp bánh đó là 50000 đồng. Do số lượng có hạn nên mỗi bạn chỉ được mua đúng một cặp. Để mua bánh các bạn học sinh trường chuyên phải xếp hàng. Biết rằng trong hàng có m + n bạn, trong đó m bạn cầm tờ 50000 đồng và n bạn cầm tờ 100000 đồng (m, n ∈ N∗ , m n). Hỏi có bao nhiêu cách xếp hàng để không bạn nào phải chờ tiền trả lại, giả thiết rằng ban đầu ban tổ chức không cầm theo đồng tiền nào. 19 Đề thi tỉnh Đồng Tháp. L Câu 1. Với mỗi số nguyên dương n 2, xét số thực un 1 sao cho phương trình [unx] = x có đúng n nghiệm nguyên (theo ẩn x và [unx] là phần nguyên của unx). 1 Chứng minh rằng [un] = 1, ∀n ∈, n 2. 2 Với mỗi cách xác định của dãy (un) thỏa điều kiện trên. Chứng minh rằng dãy (un) luôn có giới hạn và tìm giới hạn ấy. ® Chinh phục olympic toán 21 h Tạp chí và tư liệu toán học
  • 22. Š Tuyển tập đề chọn đội tuyển các tỉnh trên cả nước VMO 2021 L Câu 2. 1 Giải hệ phương trình (x + 1) (y + 1) (z + 1) = 5 √ x + √ y + √ z 2 = x + 6 2 Xét số T = 3n − 2n , trong đó n là số nguyên dương, n 2. (a) Chứng minh rằng không tồn tại n để T là bình phương của một số nguyên tố. (b) Nếu T là lập phương của một số nguyên tố thì n là một số nguyên tố. L Câu 3. Với mỗi m ∈ N∗ ta kí hiệu α (2m) = (m!)2 , α (2m + 1) = (m!) . ((m + 1)!). Cho đa thức p(x) hệ số nguyên, có bậc lớn hơn hoặc bằng k (k ∈ N∗ ) và có ít nhất k nghiệm nguyên phân biệt. Xét số nguyên n (n 6= 0) sao cho đa thức q(x) = p(x) − n có ít nhất một nghiệm nguyên. Chứng minh rằng |n| α(k). L Câu 4. Cho tam giác ABC, đường tròn nội tiếp (I) tiếp xúc với các cạnh BC, CA, AB tại D, E, F. 1 Gọi S là giao điểm của EF với BC. Chứng minh SI vuông góc với AD. 2 Đường thẳng d thay đổi, đi qua S và cắt đường tròn (I) tại hai điểm phân biệt M, N. Các tiếp tuyến tại M, N của (I) cắt nhau tại T. Chứng minh T thuộc một đường thẳng cố định. 3 Gọi K là giao điểm của ME và NF, G là giao điểm của MC và NB. Chứng minh K và G cùng thuộc đường thẳng AD. L Câu 5. Viết n số thực có tổng bằng n − 1 (n 1) quanh một đường tròn. Chứng minh rằng ta có thể gắn nhãn cho các số đó theo chiều kim đồng hồ là x1, x2, . . . , xn sao cho x1 + x2 + . . . + xk k − 1, ∀1 6 k 6 n. 20 Đề thi thành phố Hà Nội. L Câu 1. Cho hàm số y = x3 − 3 2 mx2 + m3 có đồ thị (Cm). Tìm tất cả các giá trị của tham số m để hàm số có hai điểm cực trị A, B sao cho tam giác ABO có diện tích bằng 32 (với O là gốc tọa độ). L Câu 2. 1 Giải phương trình x3 + 1 = √ 4x − 3 + √ 2x − 1. 2 Giải hệ phương trình y3 + y = x2 + 2 (1) 8y3 − 3y = 2x2 − 3 p 2x2 + y + 7 + 7 (2) L Câu 3. Cho đa giác đều 30 đỉnh A1, A2, ..., A30. Hỏi có bao nhiêu tam giác có 3 đỉnh là 3 điểm trong 30 điểm A1, A2, ..., A30 đồng thời không có cạnh nào là cạnh của đa giác. L Câu 4. Cho hình lập phương ABCD.A0 B0 C0 D0 có cạnh bằng 1. Gọi M, N là hai điểm thay đổi lần lượt trên các cạnh AB, A0 D0 sao cho đường thẳng MN tạo với mặt phẳng (ABCD) một góc bằng 60◦ . 1 Tính độ dài đoạn thẳng MN. 2 Tìm giá trị lớn nhất của khoảng cách giữa hai đường thẳng MN và CC0 . ® Chinh phục olympic toán 22 h Tạp chí và tư liệu toán học
  • 23. Š Tuyển tập đề chọn đội tuyển các tỉnh trên cả nước VMO 2021 L Câu 5. Cho dãy số (un) xác định bởi u1 = 6, un+1 = 1 2 (u2 n − 4un + 9) ; n = 1, 2, ..... 1 Chứng minh dãy số là dãy số tăng. 2 Chứng minh 1 u1 − 1 + 1 u2 − 1 + ... + 1 u2020 − 1 1 3 . L Câu 6. Cho các số thực a, b, c thỏa mãn a2 + b2 + c2 = ab + bc + ca + 6. Tìm giá trị nhỏ nhất của biểu thức P = (a − b) (b − c) (c − a) . 21 Đề thi tỉnh Hà Nam. L Câu 1. 1 Cho hàm số f(x) = −x + p (a + x)(b + x) trong đó a, b là hai số thực dương khác nhau cho trước. Chứng minh rằng với mỗi số thực s ∈ (0; 1) đều tồn tại duy nhất số thực dương x0 sao cho f (x0) = as + bs 2 1 s . 2 Xếp 35 học sinh, trong đó có bốn bạn Dũng, Minh, Công, Đoàn thành một hàng ngang. Hỏi có tất cả bao nhiêu cách xếp hàng, mà trong mỗi cách xếp hàng không có ba bạn nào trong bốn bạn Dũng, Minh, Công, Đoàn đứng ở ba vị trí liên tiếp. L Câu 2. Cho hàm số f(x) = x3 − 3x2 + 3x + 5 x + 1 . 1 Chứng minh đồ thị hàm số có ba điểm cực trị không thẳng hàng. 2 Gọi A, B, C là ba điểm cực trị của đồ thị hàm số. Tính diện tích tam giác ABC. L Câu 3. Cho p là một số nguyên tố lẻ và a là một số nguyên sao cho (a, p) = 1. Chứng minh phương trình x2 ≡ a (mod p) có nghiệm khi và chỉ khi a p−1 2 ≡ 1 (mod p). L Câu 4. Tìm tất cả các hàm f : R → R thỏa mãn điều kiện f (x2 ) + 4y2 f(y) = (f(x − y) + y2 ) .(f(x + y) + f(y)) với mọi x, y ∈ R. L Câu 5. Cho tứ giác ABCD cố định có hai đường chéo AC, BD cắt nhau tại P. Đường trung trực của các đoạn thẳng AC và BD cắt nhau tại K. Một đường thẳng d thay đổi đi qua K, cắt đường tròn ngoại tiếp tam giác PAB tại Q, R. Chứng minh trực tâm tam giác PQR luôn nằm trên một đường tròn cố định, khi đường thẳng d thay đổi. 22 Đề thi tỉnh Kiên Giang. A Ngày thi thứ nhất. L Câu 1. Cho dãy số (xn) được xác định như sau x1 = 7 3 , xn+1 = x2 n + 2xn − 2, ∀n ∈ N∗ 1 Tìm số hạng tổng quát của dãy số (xn). ® Chinh phục olympic toán 23 h Tạp chí và tư liệu toán học
  • 24. Š Tuyển tập đề chọn đội tuyển các tỉnh trên cả nước VMO 2021 2 Tìm lim n→+∞ 1 1 + x1 + 1 (1 + x1)(1 + x2) + ... + 1 (1 + x1)(1 + x2)...(1 + xn) . L Câu 2. Tìm tất cả các hàm số liên tục f : R → R sao cho 8f(4x) − 10f(2x) + 3f(x) = 30x, ∀x ∈ R L Câu 3. Trên tập hợp các số nguyên không âm, xét phương trình x2 + 2.3y = x 2y+1 − 1 (1) 1 Tìm tất cả các cặp số nguyên không âm (x; y) thỏa mãn (1) mà y 6 5. 2 Chứng minh rằng không tồn tại cặp số nguyên không âm (x; y) với y 6 thỏa mãn phương trình (1). L Câu 4. Cho đường tròn (C1) và điểm B thuộc (C1). Điểm A khác B sao cho đường thẳng AB là tiếp tuyến của (C1). Điểm C không thuộc (C1) sao cho đoạn thẳng AC cắt (C1) tại hai điểm phân biệt. Gọi (C2) là đường tròn tiếp xúc với AC tại C và tiếp xúc với (C1) tại D(điểm B và D ở khác phía so với bờ AC). Gọi I là tâm đường tròn ngoại tiếp tam giác BCD và ∆ là tiếp tuyến chung của (C1), (C2) tại D. 1 Chứng minh rằng điểm I cách đều hai đường thẳng AB và ∆. 2 Chứng minh rằng tâm đường tròn ngoại tiếp tam giác BCD nằm trên đường tròn ngoại tiếp tam giác ABC. B Ngày thi thứ hai. L Câu 5. Xét hệ phương trình          x2 = yz + at y2 = zx + bt z2 = xy + ct x + y + z = a + b + c với a, b, c, x, y, z, t ∈ R. 1 Giải hệ phương trình đã cho khi a = b = c = 1. 2 Giải hệ phương trình đã cho khi a, b, c là ba số thực được cho trước, khác không và đôi một khác nhau. L Câu 6. Trong mặt phẳng, cho đường tròn (O) cố định và hai điểm B, C phân biệt, cố định thuộc đường tròn (O) sao cho BC không là đường kính của (O). Xét điểm A di động trên đường tròn (O) sao cho tam giác ABC không là tam giác cân. Gọi H là trực tâm của tam giác ABC. 1 Chứng minh rằng vector − − → AH có hướng và độ dài không đổi. 2 Gọi d là tiếp tuyến của đường tròn (O) tại điểm A. Gọi ∆ là đường thẳng đi qua H và vuông góc với d. Chứng minh rằng ∆ luôn luôn đi qua một điểm cố định. L Câu 7. Cho n là số nguyên lớn hơn 1. Gọi Tn = {a1a2...an | ak ∈ {1; 2; 3}, 1 6 k 6 n}. 1 Tính trung bình cộng của tất cả các số thuộc Tn. 2 Hỏi có tất cả bao nhiêu số a = a1a2...an thuộc Tn mà |ak − ak+1| = 1 với mọi k thuộc tập hợp {1; 2; 3; ...; (n − 1)} ? ® Chinh phục olympic toán 24 h Tạp chí và tư liệu toán học
  • 25. Š Tuyển tập đề chọn đội tuyển các tỉnh trên cả nước VMO 2021 23 Đề thi tỉnh Quảng Ngãi. L Câu 1. Tìm số hạng tổng quát của dãy số (un) biết    u1 = 1 2 ; u2 = 2023 3 ; (n + 3) un+2 − 2(n + 2)2 un+1 + (n + 2) (n + 1)2 un = 0, n ∈ N, n 1 L Câu 2. 1 Cho n là một số nguyên dương có ít nhất 6 ước nguyên dương. Giả sử các ước nguyên dương của n được sắp theo thứ tự sau 1 = d1 d2 ... dk = n với k 6. Tìm tất cả số nguyên dương n sao cho n = d2 5 + d2 6. 2 Cho p là số nguyên tố. Chứng minh rằng tồn tại các số nguyên x, y, z, n với 0 n p thỏa mãn x2 + y2 + z2 − np = 0. L Câu 3. Cho tam giác ABC nội tiếp đường tròn (O). Một đường tròn (I) bất kì đi qua B, C và lần lượt các cạnh CA, AB tại D, E. BD cắt CE tại F và G là hình chiếu vuông góc của I lên đường thẳng AF. 1 Chứng minh rằng bốn điểm D, E, G, I cùng nằm trên một đường tròn và GA là tia phân giác của góc DGE. 2 BD cắt GE tại H, CE cắt GD tại K. Đường thẳng DE cắt đường tròn (O) tại hai điểm M, N. Chứng minh rằng đường tròn ngoại tiếp tam giác GHK tiếp xúc với đường tròn ngoại tiếp tam giác GMN. 24 Đề thi tỉnh Tây Ninh. A Ngày thi thứ nhất. L Câu 1. Cho x, y, z là ba số thực dương x + y + z = 3. Tìm giá trị nhỏ nhất của từng biểu thức sau 1 A = x2 x + y + y2 y + z + z2 z + x 2 B = 1 x2 + 1 + 1 y2 + 1 + 1 z2 + 1 . L Câu 2. Tìm tất cả các hàm số f : R → R thỏa mãn f (yf (x + y) + f (x)) = 4x + 2y.f (x + y) , ∀x, y ∈ R L Câu 3. Cho tam giác nhọn ABC không cân. Gọi H, O lần lượt là trực tâm, tâm đường tròn ngoại tiếp tam giác ABC; D, E lần lượt là chân đường cao hạ từ các đỉnh A, B của tam giác ABC. Các đường thằng OD và BE cắt nhau tại K. Các đường thẳng OE và AD cắt nhau tại L. Gọi M là trung điểm cạnh AB. Chứng minh rằng ba điểm K, L, M thẳng hàng khi và chỉ khi bốn điểm C, D, O, H cùng nằm trên một đường tròn. L Câu 4. Bạn Dung và bạn Lan tham gia giải cờ vua nữ cấp trường (cùng một số bạn khác). Biết rằng, trong số những bạn tham gia, 2 bạn bất kỳ sẽ đấu với nhau đúng 1 lần, người thắng sẽ được 1 điểm, hòa được 0,5 điểm và thua được 0 điểm. Sau khi giải đấu kết thúc, người ta thấy rằng: tổng điểm của Dung và Lan là 4 điểm, trong khi tất cả các bạn còn lại đều có số điểm bằng nhau. Hơn nữa, điểm của mỗi bạn Dung, Lan luôn ít hơn điểm của mỗi bạn khác. ® Chinh phục olympic toán 25 h Tạp chí và tư liệu toán học
  • 26. Š Tuyển tập đề chọn đội tuyển các tỉnh trên cả nước VMO 2021 1 Hãy tính xêm có bao nhiêu bạn tham gia giải cờ vua nữ cấp trường này. 2 Nêu ra mô hình thi đấu cụ thể thỏa mãn đề bài (với đáp số tìm được trong câu 1/). B Ngày thi thứ hai. L Câu 5. Giải hệ phương trình trên tập số thực      2x = y 1 − x2 2y = z 1 − y2 2z = x 1 − z2 . L Câu 6. Cho dãy số (un) thỏa mãn    u1 = 1 un+1 = 2u2 n + 5un + 2 u2 n + un + 1 ; n 1, n ∈ N Chứng minh dãy số (un) có giới hạn hữu hạn và tìm giới hạn đó. L Câu 7. Tìm tất cả các số nguyên tố p, q sao cho pq | (2p + 2q ). L Câu 8. Trong tam giác ABC, đường tròn nội tiếp tiếp xúc với các cạnh BC, CA, AB lần lượt tại D, E, F. Một đường tròn (w) đi qua A và tiếp xúc với BC tại D cắt các đường thẳng BF, CE tương ứng tại K và L. Đường thẳng đi qua E và song song với DL cắt đường thẳng đi qua F và song song với DK tại P. Đặt R1, R2, R3, R4 lần lượt là độ dài bán kính đường tròn ngoại tiếp các tam giác AFD, AED, FPD, EPD. Chứng minh rằng R1.R4 = R2.R3. 25 Đề thi tỉnh Quảng Nam. L Câu 1. Giải hệ phương trình ( 2x2 − x − 3 + x √ x2 + 1 = (y + 1) p y2 + 2y + 2 x2 + y2 = x − 2y + 2 L Câu 2. Cho dãy (un) được xác định bởi u1 = 1; un+1 = p u2 n + 2un + 3 − p u2 n − 2un + 3, ∀n ∈ N Chứng minh rằng dãy số (un) có giới hạn hữu hạn và tìm giới hạn đó. L Câu 3. Cho tam giác ABC nhọn, không cân AB AC, nội tiếp đường tròn tâm O và ngoại tiếp đường tròn tâm I. Đường thẳng qua I vuông góc với AI cắt đường thẳng BC tại S. Gọi M là trung điểm BC, D là điểm chính giữa cung nhỏ BC của đường tròn (O) và J là điểm đối xứng với I qua O. 1 Chứng minh tam giác DBI cân và tam giác DSJ vuông. 2 Gọi X là hình chiếu của điểm S trên đường thẳng OI và Y là giáo điểm thứ hai của MI và đường tròn ngoại tiếp tam giác OMS. Chứng minh ba điểm A, X, Y thẳng hàng. L Câu 4. Gọi α, β là các nghiệm của phương trình x2 −ax+a = 0, trong đó a là một số nguyên lớn hơn 4. Chứng minh rằng với mọi số nguyên dương m, n thì [ma] 6= [nb] với [x] là ký hiệu số nguyên lớn nhất không vượt quá x. L Câu 5. Cho dãy đa thức hệ số thực {fn(x)} xác định bởi f0 (x) = 2; f1 (x) = 2x; fn+2 (x) = 2x.fn+1 (x) + 1 − x2 fn (x) , ∀n ∈ N Tìm tất cả các số tự nhiên n để fn(x) chia hết cho x2 + 3. ® Chinh phục olympic toán 26 h Tạp chí và tư liệu toán học
  • 27. Š Tuyển tập đề chọn đội tuyển các tỉnh trên cả nước VMO 2021 L Câu 6. Một số tự nhiên được gọi là số thân thiện nếu nó có thể phân tích được thành tích của một số các số nguyên dương mà tổng của chúng bằng 2020. Hãy tìm số thân thiện lớn nhất. L Câu 7. Cho các số thực a, b, c thỏa mãn điều kiện a2 + b2 + c2 = 3. Tìm giá trị lớn nhất và giá trị nhỏ nhất của biểu thức P = abc + 2 (ab + bc + ca) + 4 (a + b + c) 26 Đề thi tỉnh Quảng Trị. L Câu 1. 1 Tìm tất cả các điểm cực đại và điểm cực tiểu của hàm số y = cos x − sin x. 2 Tìm m để phương trình |2x4 − 4x2 + 1| − 2m = 0 có đúng 5 nghiệm phân biệt. L Câu 2. 1 Chứng minh rằng 2020 1 + 2 2020 2 + ... + 1010 2020 1010 = 1010.22019 . 2 Tìm tất cả các cặp số thực (x; y) thỏa mãn |xy| 6 4 và (x − y)2 + 20 = (x + y)(xy − 8). L Câu 3. 1 Cho hình chóp S.ABC có đáy ABC là tam giác đều cạnh bằng a, tam giác SAB vuông cân tại S và nằm trong mặt phẳng vuông góc với mặt phẳng đáy. Tính thể tích của khối chóp S.ABC và khoảng cách giữa hai đường thẳng SB và AC theo a. 2 Cho tam giác ABC ngoại tiếp đường tròn (I). Gọi M, D, E lần lượt là trung điểm của BC, IB, IC; F, G lần lượt là tâm đường tròn ngoại tiếp các tam giác ABD và ACE. Chứng minh AM vuông góc FG. L Câu 4. Cho dãy số (xn) được xác định bởi x1 = √ 2 và xn+1 = √ 2 − xn, ∀n 1. Chứng minh dãy số (xn) có giới hạn hữu hạn và tìm giới hạn đó. L Câu 5. Xét các số thực dương a, b, c có tổng bằng 3. Tìm giá trị nhỏ nhất của biểu thức P = 2b + c a + 2c + a b + 2a + b c + 18abc ab + bc + ca 27 Đề thi chọn đội tuyển THPT Chuyên Nguyễn Du. A Ngày thi thứ nhất. L Câu 1. 1 Giải phương trình x4 + 2x3 + 2x2 − 2x + 1 = (x3 + x) r 1 − x2 x . 2 Giải hệ phương trình ( xy + 2 = y √ x2 + 2 y2 + 2(x + 1) √ x2 + 2x + 3 = 2x2 − 4x . L Câu 2. Cho các số thực dương x, y, z thỏa mãn x + y + z = 1. Chứng minh rằng ® Chinh phục olympic toán 27 h Tạp chí và tư liệu toán học
  • 28. Š Tuyển tập đề chọn đội tuyển các tỉnh trên cả nước VMO 2021 1 + x y + z + 1 + y z + x + 1 + z x + y 6 2 x z + z y + y x L Câu 3. Chứng minh rằng với mọi n ∈ Z+ , luôn tồn tại m ∈ N sao cho √ 2 − 1 n = √ m + 1 − √ m L Câu 4. Cho tứ giác lồi ABCD nội tiếp đường tròn (C). Gọi M, N, P lần lượt là giao điểm của các cặp đường thẳng AB và CD, AD và BC, AC và BD. Gọi I1, I2, I3, I4 lần lượt là tâm đường tròn bàng tiếp các tam giác ABN, BCM, CDN và ADM tương ứng với các đỉnh A, C, D và D. 1 Chứng minh các điểm I1, I2, I3, I4 đồng viên. 2 Gọi I là tâm đường tròn qua I1, I2, I3, I4. Chứng minh PI vuông góc với MN. L Câu 5. Tìm tất cả các hàm số f : R → R thỏa mãn f(x + f(y)) − f(f(x) − x) = f(y) − f(x) + 2x + 2y, ∀x, y ∈ R B Ngày thi thứ hai. L Câu 6. Cho (an) ; (bn) thỏa mãn          a1 = 2020; b1 = 1 2020 an+1 = p an + bn + 2 bn+1 = p 2an + bn + 6 . Tính giới hạn (an) ; (bn) nếu có. L Câu 7. Tìm các đa thức P(x), Q(x) ∈ R[x] khác đa thức không và có bậc bé nhất thỏa mãn P (x2 ) + Q(x) = P(x) + x5 Q(x) ∀x ∈ R L Câu 8. Tìm tất cả n tự nhiên để A = 22 ...2 |{z} n số 2 −2 viết được thành a3 + b3 + c3 với a, b, c nguyên. L Câu 9. Cho tam giác ABC(AC AB). Lấy hai điểm M, N lần lượt trên AB và AC sao cho MN song song với BC. Gọi P là giao điểm của hai đoạn thẳng BN và CM. Gọi A0 là điểm đối xứng của A qua đường thẳng BC; (ω) là đường tròn ngoại tiếp tam giác AMN. 1 Gọi E là điểm thuộc đường tròn (ω) sao cho AE k MN. Chứng minh rằng E, P, A0 thẳng hàng. 2 Gọi F là giao điểm thứ hai của A0 P với đường tròn (ω) và I là tâm đường tròn ngoại tiếp tam giác AA0 F. Chứng minh rằng IF tiếp xúc với đường tròn ngoại tiếp tam giác BFC. L Câu 10. Cho tập hợp A = {1, 2, ..., 101}, tô màu ít nhất 50 phần tử của A sao cho nếu a, b ∈ A (a, b không nhất thiết phân biệt) được tô màu và a + b ∈ A thì a + b cũng được tô màu. Gọi S là tổng tất cả các số không được tô màu của A. Tìm giá trị lớn nhất của S. 28 Đề thi tỉnh Phú Yên. L Câu 1. Giải phương trình x + 4 √ x + 3 + 2 √ 3 − 2x = 11. L Câu 2. Cho hệ phương trình      xyz + z = a xyz2 + z = b x2 + y2 + z2 = 4 , (a, b ∈ R) Tìm tất cả các giá trị của a, b để hệ phương trình có nghiệm duy nhất. ® Chinh phục olympic toán 28 h Tạp chí và tư liệu toán học
  • 29. Š Tuyển tập đề chọn đội tuyển các tỉnh trên cả nước VMO 2021 L Câu 3. 1 Cho tam thức bậc hai f(x) = ax2 + bx + c (a, b, c ∈ R; a 6= 0) có hai nghiệm x1, x2 thuộc đoạn [0; 1]. Tìm giá trị lớn nhất của biểu thức A = (a − b)(2a − b) a(a − b + c) . 2 Cho a, b, c là các số thực dương. Chứng minh rằng a b + b c + c a + 9 3 √ abc a + b + c 6. L Câu 4. 1 Cho điểm M tùy ý nằm bên trong tam giác ABC. Gọi S1, S2, S3 lần lượt là diện tích của các tam giác MBC, MAC và MAB. Chứng minh rằng S1. − − → MA + S2 − − → MB + S3 − − → MC = − → 0 . 2 Trong mặt phẳng Oxy, cho parabol (P) : y = x2 + px + q (q 6= 0). Biết rằng (P) cắt trục Ox tại hai điểm phân biệt A, B và cắt trục Oy tại C. Chứng minh rằng khi p và q thay đổi, đường tròn ngoại tiếp tam giác ABC luôn đi qua một điểm cố định. L Câu 5. Cho dãy số (un) xác định bởi: u1 = 2; un+1 = u2 n 2un − 1 ,với n = 1, 2, 3, ... 1 Chứng minh rằng dãy số (un) giảm và bị chặn. 2 Hãy xác định số hạng tổng quát của dãy số (un). L Câu 6. Tìm tất cả các hàm số f : R+ → R thỏa mãn điều kiện f x + y 2020 = f(x) + f(y) 2019 , ∀x, y ∈ R+ 29 Đề chọn đội tuyển chuyên Đại học Vinh. A Ngày thi thứ nhất. L Câu 1. Cho dãy số (un) xác định bởi u1 = 2, un+1 = un + n2 u2 n , n = 1, 2, 3, ... 1 Chứng minh rằng lim un n = 1. 2 Tính giới hạn lim (un − n). L Câu 2. Cho các số thực dương x, y, z thỏa mãn x + y + z = 1 x + 1 y + 1 z . Tìm giá trị nhỏ nhất của P = 4 r xy z + 4 r yz x + 4 r zx y L Câu 3. Cho đường tròn (O) cố định và AB là một dây cung cố định của đường tròn đó (AB không đi qua O). Gọi I là trung điểm AB và M là một điểm di động trên cung lớn _ AB (M khác A và B). Lấy các điểm A0 , B0 lần lượt nằm trên các tia MA, MB sao cho MA0I = MB0I = AMB. 1 Gọi E là tâm của đường tròn (OAB). Chứng minh rằng ME vuông góc với A0 B0 . 2 Chứng minh rằng đường thẳng Euler của tam giác MA0 B0 luôn đi qua một điểm cố định khi điểm M di động trên cung lớn _ AB. ® Chinh phục olympic toán 29 h Tạp chí và tư liệu toán học
  • 30. Š Tuyển tập đề chọn đội tuyển các tỉnh trên cả nước VMO 2021 L Câu 4. Cho bảng ô vuông kích thước 20 × 20, trong đó có n ô vuông được tô màu đen, các ô vuông còn lại được tô màu trắng. Ta đặt lên bảng một tấm bìa hình O là hình vuông kích thước 3 × 3 khuyết ô vuông ở tâm sao cho các ô vuông của tấm bìa trùng khít lên các ô vuông của bảng. 1 Với n = 46, chứng minh với mọi cách tô n ô vuông màu đen ta luôn có thể đặt tấm bìa hình O sao cho không có ô vuông đen nào của bảng bị che khuất. 2 Với n = 56, hỏi khẳng định ở ý 1) có còn đúng hay không? B Ngày thi thứ hai. L Câu 5. Cho hàm số f : (0; +∞) → (0; +∞) thỏa mãn f(x) + f(x + y) = f(f(x) + y) với mọi số thực dương x, y. 1 Chứng minh rằng hàm số g(x) = f(x) − x là đơn ánh, nhận giá trị dương trên (0; +∞) và f(x) + f(y) = 2f x + y 2 với mọi số thực dương x, y. 2 Chứng minh rằng f là đơn ánh và tìm tất cả các hàm f thỏa mãn bài toán. L Câu 6. Cho tam giác ABC không cân. Đường tròn (I; r) nội tiếp tam giác ABC tiếp xúc BC, CA, AB lần lượt tại D, E, F. Gọi J là giao điểm của AD và EF. Lấy M, N trên (I; r) sao cho M, J, N thẳng hàng và M nằm về phía nửa mặt phẳng chứa C bờ AD, N nằm về phía nửa mặt phẳng chứa B bờ AD. Giả sử DM cắt AC tại P, DN cắt AB tại Q. Tiếp tuyến tại M, N của (I; r) cắt nhau ở S. Gọi G là giao điểm của EF và BC và T là giao điểm của MN và AG. 1 Chứng minh P, Q, T thẳng hàng và S, A, G thẳng hàng. 2 Gọi K là giao điểm của SJ và BC, H là giao điểm của IK và TD. Chứng minh rằng IH.IK = r2 . L Câu 7. Chứng minh rằng n = 21 là số nguyên dương nhỏ nhất sao cho với mọi số nguyên dương a, b, c thỏa mãn a|b4 , b|c4 , c|a4 ta đều có abc|(a + b + c)n 30 Đề thi tỉnh Ninh Thuận. L Câu 1. 1 Cho số thực x ∈ 0; 7 2 . Chứng minh rằng 15x + 28 x 2x2 + 36. 2 Cho ba số thực dương a, b, c thỏa mãn a2 + b2 + c2 = 12. Chứng minh rằng 15abc(a + b + c) + 28(ab + bc + ca) 132abc. L Câu 2. Cho dãy số (un) được xác định bởi u0 = 0 và un+1 = − u2 n 10 + un + 1 2 , với mọi số tự nhiên n. Chứng minh rằng dãy số (un) có giới hạn hữu hạn và tính giới hạn của nó? L Câu 3. Một đa thức với hệ số nguyên được gọi là bất khả quy trên tập số nguyên nếu đa thức đó không phân tích được thành tích hai đa thức với hệ số nguyên có bậc lớn hơn hoặc bằng 1. Chứng minh rằng đa thức f(x) = x2021 + 7x2020 + 5 bất khả quy trên tập số nguyên. ® Chinh phục olympic toán 30 h Tạp chí và tư liệu toán học
  • 31. Š Tuyển tập đề chọn đội tuyển các tỉnh trên cả nước VMO 2021 L Câu 4. Cho điểm A nằm ngoài đường tròn (O) tâm O. Dựng các tiếp tuyến AB và AC của (O), trong đó B và C là các tiếp điểm. Gọi O0 là điểm cố định trên tia OA, vẽ đường tròn (O0 ) có tâm là O0 và đi qua hai điểm B, C. Một điểm M di động trên đường tròn (O) sao cho M không trùng với B và C. Gọi D là giao điểm thứ hai của đường thẳng MA và đường tròn (O). Gọi E là giao điểm thứ hai của đường thẳng MB và đường tròn (O0 ). Gọi F là giao điểm thứ hai của đường thẳng BD và đường tròn (O0 ). Gọi N là giao điểm của MA và EF. 1 Chứng minh rằng N là trung điểm của đoạn EF. 2 Khi điểm M di động trên đường thẳng (O), chứng minh đường thẳng EF luôn đi qua một điểm cố định. L Câu 5. Bạn Hảo rất thích chơi với ánh sáng, cậu có một dãy gồm 10 bóng đèn xếp thứ tự từ 1 đến 10. Mỗi bóng đèn có một công tắc mà khi chạm vào nó, bóng đèn đang tắt sẽ chuyển sang trạng thái sáng và ngược lại. Ban đầu tất cả các bóng đèn đều đang tắt. Trò chơi của bạn Hảo bao gồm 15 lần chạm vào công tắc, mỗi lần chỉ chạm một công tắc. Gọi N là số cách thực hiện 15 lần chạm để được kết quả các bóng đèn 1,2,3,4,5 sáng và các bóng đèn 6,7,8,9,10 tắt. Gọi M là số cách thực hiện 15 lần chạm sao cho trong các lần chạm này chỉ được chạm vào các công tắc của các bóng đèn 1,2,3,4,5 để được kết quả các bóng đèn 1,2,3,4,5 sáng và các bóng đèn 6,7,8,9,10 tắt. Hãy tính giá trị của tỉ số N M . 31 Đề thi tỉnh Đồng Nai. L Câu 1. Cho dãy số (un) xác định bởi u1 = −2020 và un+1 = un + 1 2021n với mọi n ∈ N∗ . Chứng minh rằng tồn tại số nguyên dương n sao cho un 0. L Câu 2. Tìm các số nguyên dương x và y thỏa mãn 7x + x4 + 47 = y2 . L Câu 3. Tìm các hàm số f : Z → Z thỏa mãn f(a2 f(a) + f(b)) = f(a)3 + b, ∀a, b ∈ Z. L Câu 4. Cho tam giác ABC cân tại A, lấy điểm D thuộc cạnh AB khác A và B, gọi (O) là đường tròn ngoại tiếp tam giác BCD, tiếp tuyến của đường tròn (O) tại D cắt đường thẳng AC tại điểm E, vẽ tiếp tuyến EFcủa đường tròn (O) tại tiếp điểm F khác D. Gọi I là giao điểm của hai đường thẳng BF và CD, gọi K là giao điểm của hai đường thẳng AI và BC. Chứng minh BK = 2CK. L Câu 5. Một tổ gồm có 5 học sinh được phân công trực nhật 6 ngày trong tuần từ thứ Hai đến thử Bảy thỏa mãn các điều kiện sau: Mỗi ngày đều có từ 1 đến nhiều nhất là 2 học sinh trực và trong cả tuần mỗi học sinh trực đúng 2 lần, mỗi lần trực 1 ngày. Tính số các cách phân công trực nhật của tổ thỏa mãn các điều kiện đã cho. 32 Đề thi tỉnh Quảng Ngãi. A Ngày thi thứ nhất. L Câu 1. Giải hệ phương trình ( 2x + √ 2x + 1 + p 2x + y + 2xy + 1 = 2y + p y + 1 + 1 3 3 p 3y + 1 + 1 = 4x2 y L Câu 2. Cho dãy số (xn) xác định bởi    x1 = a(a 2) xn+1 = x2 n + 12xn 3x2 n + 4 , ∀n ∈ N∗ ® Chinh phục olympic toán 31 h Tạp chí và tư liệu toán học
  • 32. Š Tuyển tập đề chọn đội tuyển các tỉnh trên cả nước VMO 2021 1 Chứng minh rằng dãy số (xn) có giới hạn hữu hạn và tính giới hạn đó. 2 Đặt yn = xn 2 + xn−1 21 + ... + x1 2n , ∀n ∈ N∗ . Tính lim yn? L Câu 3. Cho tam giác ABC nhọn nội tiếp đường tròn ω tâm O. Các tiếp tuyến với ω tại B và C cắt nhau tại T. 1 Gọi M là trung điểm của BC. Chứng minh [ BAT = CAM. 2 Đoạn thẳng OT cắt đường tròn ω tại X. Đoạn thẳng AX cắt đường tròn tâm bán kính TB tại I. Gọi E, F lần lượt là hình chiếu vuông góc của I lên các cạnh AC, AB. Các đoạn thẳng BE, CF cắt nhau tại Z. Chứng minh AZ, BC, OI đồng quy. L Câu 4. Trong lễ khai mạc kỳ thi chọn đội tuyển thi HSGQG, để chuẩn bị cho việc thả bóng bay lên trời thể hiện quyết tâm và khát vọng của học sinh, Ban tổ chức chọn 9 bạn đại diện cho 9 môn: Toán, Tin, Lý, Hóa, Sinh, Văn, Anh, Sử, Địa xếp thành một hàng dọc. Sau đó, Ban tổ chức phát các quả bóng bay gồm hai màu xanh và hồng cho 9 học sinh này (mỗi bạn một quả bóng) sao cho thỏa mãn đồng thời hai điều kiện sau 1 Học sinh đứng cuối hàng nhận bóng bay màu xanh. 2 Nếu học sinh thứ i và thứ j nhận bóng bay khác màu thì học sinh thứ i + j nhận bóng bay màu hồng (1 6 i j 6 9, i + j 6 9). Tìm tất cả các cách phát bóng bay có thể có của ban tổ chức. B Ngày thi thứ hai. L Câu 5. Một số nguyên dương n được gọi là “số tạo cấp số” nếu nó thỏa mãn đồng thời hai điều kiện sau 1 n có ít nhất 4 ước nguyên dương d1, d2, .., dk(k 4) với d1 d2 .. dk. 2 Các số d2 − d1, d3 − d2, .., dk − dk−1 theo thứ tự đó lập thành một cấp số nhân. a) Tìm tất cả các số nguyên dương chẵn là số tạo cấp số. b) Tìm tất cả các số nguyên dương là số tạo cấp số. L Câu 6. Cho đa thức hệ số nguyên P(x) = anxn + an−1xn−1 + ... + a1x + a0(an 6= 0). 1 P(x) được gọi là đa thức đặc biệt nếu các hệ số của nó thỏa mãn đồng thời hai điều kiện sau (a) Nếu 0 6 i 6 n, 3 - i thì 3 | ai. (b) 6 | X ai, trong đó X ai là tổng tất cả các hệ số ai mà 3 - i. Chứng minh rằng với Q(x) là một đa thức hệ số nguyên bất kỳ thì [Q(x)]3 là một đa thức đặc biệt. 2 P(x) được gọi là đa thức tổng lập phương nếu tồn tại m số nguyên b1, b2, ..., bm và m đa thức hệ số nguyên P1(x), P2(x), ..., Pm(x) sao cho P(x) = b1[P1(x)]3 + ... + bm[Pm(x)]3 với m ∈ N∗ . Hỏi T(x) = 15x2 + 9x + 2021 có phải là đa thức tổng lập phương không? Vì sao? 3 Chứng minh rằng mọi đa thức đặc biệt đều là đa thức tổng lập phương. ® Chinh phục olympic toán 32 h Tạp chí và tư liệu toán học
  • 33. Š Tuyển tập đề chọn đội tuyển các tỉnh trên cả nước VMO 2021 L Câu 7. Cho đường tròn (O) và hai điểm B, C cố định trên (O), BC không là đường kính. Một điểm A thay đổi trên (O) sao cho tam giác ABC nhọn, không cân. Đường tròn nội tiếp của tam giác ABC tiếp xúc với các cạnh BC, CA, AB lần lượt tại D, E, F. Gọi I là tâm đường tròn nội tiếp của tam giác ABC. 1 Chứng minh rằng các đường tròn ngoại tiếp tam giác AID, BIE, CIF có hai điểm chung. 2 Gọi G là giao điểm của CI và DE, H là điểm nằm trên đường thẳng EF sao cho IH vuông góc với IC. Các đường thẳng GH và ID cắt nhau tại K, BK và CK lần lượt cắt đường thẳng EF tại M và L. Gọi N là trung điểm của LM. Chứng minh rằng đường thẳng IN luôn đi qua một điểm cố định. 33 Đề thi tỉnh Vĩnh Long. A Ngày thi thứ nhất. L Câu 1. 1 Giải phương trình 4 √ 15 + x − 4 √ 2 − x = 1, x ∈ R. 2 Giải hệ phương trình ( x3 (−2 + 3y) = −8 x(y3 + 2) = −6 (x, y ∈ R). L Câu 2. Cho dãy số (un)(n = 1, 2, 3, ...) được xác định bởi    u1 = 0 un+1 = un + 2018 −un + 2020 , (n = 1, 2, 3, ...) Chứng minh dãy số (un)(n = 1, 2, 3, ...) có giới hạn hữu hạn và tính lim un. L Câu 3. 1 Có bao nhiêu số tự nhiên gồm 4 chữ số thỏa mãn không có chữ số nào lặp lại đúng 3 lần ? 2 Cho tập hợp S = {1; 2; 3; 4; 5; 6; 7; 8; 9; 10}. Hỏi có bao nhiêu cách chia tập S thành ba tập khác rỗng sao cho trong mỗi tập con đó không có hai số nguyên liên tiếp nào ? L Câu 4. Cho tam giác ABC nhọn, các đường cao BK và CL cắt nhau tại H. Một đường thẳng đi qua H cắt AB, AC lần lượt tại P, Q. Chứng minh rằng HP = HQ khi và chỉ khi MP = MQ, với M là trung điểm của BC. B Ngày thi thứ hai. L Câu 5. Cho x, y, z là ba số thực dương thỏa mãn điều kiện x + y + z = 1. Chứng minh rằng √ x + yz + √ y + zx + √ z + xy 1 + √ xy + √ yz + √ zx. L Câu 6. 1 Tìm tất cả các cặp số nguyên dương (x; y) với x, y nguyên tố cùng nhau và thỏa mãn phương trình 2 (x3 − x) = y3 − y. 2 Cho 2020 tập hợp mà mỗi tập chứa đúng 45 phần tử. Biết rằng hai tập tùy ý trong các tập này đều có đúng một phần tử chung. Chứng minh rằng tồn tại phần tử thuộc tất cả 2020 tập họp đã cho. ® Chinh phục olympic toán 33 h Tạp chí và tư liệu toán học
  • 34. Š Tuyển tập đề chọn đội tuyển các tỉnh trên cả nước VMO 2021 L Câu 7. Cho tam giác ABC nội tiếp trong một đường tròn và M là điểm chính giữa cung BC. Gọi I, J, K theo thứ tự là chân các đường vuông góc kẻ từ M đến các đường thẳng AB, BC, CA; X là giao điểm của BK và AJ; L là giao điểm của CX và IJ. Vẽ tia Jy vuông góc với MK cắt AL tại T. Chứng minh CT vuông góc với IM. L Câu 8. Tìm tất cả các hàm số f : R+ → R+ thỏa mãn điều kiện f(x + y) + f(xy) = x + y + xy với mọi x, y ∈ R+ . 34 Đề thi tỉnh Quảng Bình. L Câu 1. 1 Cho dãy số thực (xn) xác định bởi    x1 = 1 xn+1 = q 6 + √ 2xn + 3, ∀n ∈ N∗ . Chứng minh rằng dãy số (xn) có giới hạn hữu hạn. Tìm lim xn. 2 Cho dãy số thực (un) xác định bởi    u1 = 2, u2 = 1 un+2 = r 6 + 1 2 p 3un+1 + 5un + 12, ∀n ∈ N∗ . Tìm lim un. L Câu 2. Trên các cạnh AB, AC của tam giác ABC lần lượt lấy hai điểm C1, B1. Hai đoạn thẳng BB1 và CC1 cắt nhau tại X và hai đoạn thẳng B1C1 và AX cắt nhau tại P. Đường tròn ngoại tiếp các tam giác BXC1, CXB1 cắt nhau tại điểm thứ hai Y và cắt cạnh BC lần lượt tại D và E. 1 Giả sử B1C1 k BC và gọi H, K lần lượt là hình chiếu vuông góc của Y lên AB và AC. Chứng minh rằng Y H AB = Y K AC . 2 Giả sử B1E và C1D cắt nhau tại Q và đường thẳng B1D cắt đường thẳng C1E tại R. Chứng minh ba điểm P, Q và R thẳng hàng. L Câu 3. Cho tập hợp X có 2020 phẩn tử. Bạn An chia tập X thành 2 tập hợp A và B thỏa mãn |A| = |B|; A ∩ B = ∅, bằng k cách khác nhau. Tìm giá trị nhỏ nhất của k sao cho với 2 phần tử bất kỳ của X, luôn có ít nhất 1 cách trong k cách chia mà bạn An chia chúng vào 2 tập hợp khác nhau. L Câu 4. Gọi n là số nguyên dương thỏa mãn điều kiện 2n − 5 | 3(n! + 1). 1 Giả sử tồn tại n 4 thỏa mãn điều kiện trên. Chứng minh rằng 2n − 5 là số nguyên tố. 2 Tìm tất cả các số nguyên dương n thỏa mãn điều kiện trên. 35 Đề thi tỉnh Bình Dương L Câu 1. Giải hệ phương trình sau :      x + 3y = x3 − 12 − y + 4z = y3 − 6 9z + 2x = z3 + 32 . L Câu 2. Với ba số thực a, b, c thỏa mãn các điều kiện abc 6= 0 và 1 |a| + 1 |b| + 1 |c| 6 3. Hãy tìm giá trị nhỏ nhất của biểu thức ® Chinh phục olympic toán 34 h Tạp chí và tư liệu toán học
  • 35. Š Tuyển tập đề chọn đội tuyển các tỉnh trên cả nước VMO 2021 P = [a2 + 2 (b2 + c2 )] [b2 + 2 (c2 + a2 )] [c2 + 2 (a2 + b2 )] L Câu 3. Tìm tất cả các số nguyên dương n và k sao cho (n + 1)n = 2nk + 3n + 1. L Câu 4. Cho dãy số (un) thỏa mãn          u1 = 1 2 un+1 = 1 2 un + r u2 n + 1 4n ! . Tính un theo n và tìm lim n→∞ un. L Câu 5. 1 Cho n ∈ N, n 2. Xét n số thực phân biệt x1, ..., xn. Đặt S = {xi + xj; 1 6 i 6= j 6 n}. Hỏi S có ít nhất bao nhiêu phần tử ? 2 Trong mặt phẳng tọa độ, cho 2021 điểm phân biệt, mỗi điểm có hoành độ và tung độ đều là các số nguyên không âm không vượt quá 807. Chứng minh rằng có thể tìm được 4 điểm tạo thành 4 đỉnh của một hình thang cân (Hình chữ nhật được coi là một hình thang cân). L Câu 6. Cho tam giác ABC có trực tâm H. Điểm P di chuyển trên cạnh BC. Lấy các điểm M, N sao cho PM ⊥ AB, MB ⊥ BC, PN ⊥ AC, NC ⊥ BC. 1 Chứng minh rằng đường thẳng MN đi qua H. 2 Chứng minh rằng đường thẳng đi qua P và vuông góc với MN luôn đi qua một điểm cố định. 36 Đề thi tỉnh Yên Bái. L Câu 1. Tìm tất cả các bộ (x, y, z) gồm ba số thực đôi một phân biệt thỏa mãn hệ phương trình      x2 + y2 = −x + 2y + 2z y2 + z2 = x − y + 3z z2 + x2 = −x + y + 3z L Câu 2. Cho (xn)∞ n=1 là dãy số giảm ngặt và có giới hạn bằng 0. Xét dãy số (yn)∞ n=1, với yn = x1 − x2 + x3 − x4 + ... + (−1)n+1 xn, ∀n ∈ N∗ Chứng minh rằng dãy số (yn)∞ n=1 có giới hạn hữu hạn. L Câu 3. Cho p là một số nguyên tố khác 3 và a, b, c, d là các số nguyên. Với mỗi số nguyên dương n ta đặt Sn = an + bn + cn + dn . Chứng minh rằng nếu S1 và S3 cùng chia hết cho p thì S2021 cũng chia hết cho p. L Câu 4. Cho tứ giác ABCD không là hình thang nội tiếp đường tròn (ω) tâm O. Gọi O1 và O2 là tâm đường tròn nội tiếp các tam giác ABC và ABD. Đường thẳng O1O2 cắt các đoạn thẳng BC và AD lần lượt tại E và F. Đường thẳng qua E vuông góc với BC và đường thẳng qua F vuông góc với AD cắt nhau tại J. Chứng minh rằng 1 Điểm J nằm trên trung trực của đoạn thẳng EF. 2 Đường tròn tâm J bán kính JE tiếp xúc với đường tròn (ω). L Câu 5. Cho một đa giác đều có 2020 đỉnh. Mỗi đỉnh của đa giác được gán một trong các số nguyên dương 1, 2, 3, ..., 1009. Chứng minh rằng đa giác đó có bốn đỉnh A, B, C, D thỏa mãn đồng thời hai điều kiện dưới đây 1 Hai đường thẳng AB và CD song song với nhau. 2 Nếu bốn đỉnh A, B, C, D lần lượt được gán các số a, b, c, d thì a + b = c + d. ® Chinh phục olympic toán 35 h Tạp chí và tư liệu toán học